2ちゃんねる スマホ用 ■掲示板に戻る■ 全部 1- 最新50    

■ このスレッドは過去ログ倉庫に格納されています

■ちょっとした物理の質問はここに書いてね287■

1 :ご冗談でしょう?名無しさん:2024/02/15(木) 11:37:39.86 ID:???.net
★荒らし厳禁、煽りは黙殺
★書き込む前にの注意事項を読んでね
★数式の書き方(参考)はこちら   >>2-3

===質問者へ===

重要 【 丸 投 げ 禁 止 】

・質問する前に
1. 教科書や参考書をよく読む
2. http://www.google.com/ などの検索サイトを利用し、各自で調べる
3. 学生は自分の学年、物理科目の履修具合を書く
4. 宿題を聞くときは、どこまでやってみてどこが分からないのかを書く
・質問に対する回答には返答してね、感謝だけでなく「分からん」とかダメ出しでもOK
・質問するときはage&ID表示推奨
・高度すぎる質問には住人は回答できないかもしれないけれど、了承の上での質問なら大歓迎

===回答者へ===

・丸投げは専用スレに誘導
・不快な質問は無視、構った方が負け
・質問者の理解度に応じた適切な回答をよろしく
・単発質問スレを発見したらこのスレッドへの誘導をよろしくね
・逆に議論が深まりそうなら新スレ立てて移動するのもあり
・板違いの質問は適切な板に誘導を
・不適切な回答は適宜訂正、名回答は素直に賞賛

※前スレ
■ちょっとした物理の質問はここに書いてね285■
https://rio2016.5ch.net/test/read.cgi/sci/1702121891/
■ちょっとした物理の質問はここに書いてね286■
https://rio2016.5ch.net/test/read.cgi/sci/1705558700/

2 :ご冗談でしょう?名無しさん:2024/02/15(木) 11:38:19.82 ID:???.net
●括弧: (), [], {}を適切に入れ子にして分かりやすく書く
●スカラー: a,b,...,z, A,...,Z, α,β,...,ω, Α,Β,...,Ω,...(「ぎりしゃ」「あるふぁ~おめが」で変換)
●ベクトル: V=(v1,v2,...), |V>,V↑, (混同しないならスカラーの記号でいい。通常は縦ベクトル)
●テンソル: T^[i,j,k...]_[p,q,r,...], T[i,j,k,...; p,q,r,...]  (上下付き1成分表示)
●行列: M[i,j], I[i,j]=δ_[i,j] M = [[M[1,1],M[2,1],...], [M[1,2],M[2,2],...],...], I = [[1,0,0,...],[0,1,0,...],...]
 (右は全成分表示。行または列ごとに表示する。例:M=[[1,-1],[3,2]])
●対角行列: diag(a,b) = [[a,0],[0,b]]
●転置行列・随伴行列:M^T, M†("†"は「だがー」で変換可) ●行列式・トレース:|A|=det(A), tr(A)
●複号: a±b("±"は「きごう」で変換可)
●内積・外積: a・b, a×b
●関数・汎関数・数列: f(x), F[x(t)] {a_n}
●平方根: √(a+b) = (a+b)^(1/2) = sqrt(a+b) ("√"は「るーと」で変換可)
●指数関数・対数関数: exp(x+y)=e^(x+y) ln(x)=log_e(x) (底を省略して単にlogと書いたとき多くは自然対数)
 括弧を省略しても意味が容易に分かるときは省略可: sin(x) = sin x
●三角関数、逆三角関数、双曲線関数: sin(a), cos(x+y), tan(x/2), asin(x)=sin^[-1](x), cosh(x)=[e^x+e^(-x)]/2
●絶対値:|x| ●ノルム:||x|| ●共役複素数:z^* = conj(z)

3 :ご冗談でしょう?名無しさん:2024/02/15(木) 11:38:42.24 ID:???.net
a:加速度、昇降演算子 A:振幅、ベクトルポテンシャル B:磁束密度 c:光速 C:定数、熱・電気容量
d:次元、深さ D:領域、電束密度 e:自然対数の底、素電荷 E:エネルギー、電場
f:周波数 f,F:力 F:Helmholtzエネルギー g:重力加速度、伝導度
G:万有引力定数、Gibbsエネルギー、重心 h:高さ、プランク定数 H:エンタルピー、Hamiltonian、磁場
i:虚数単位 i,j,k,l,m:整数のインデックス I:電流、慣性モーメント j:電流密度・流束密度
J:グランドポテンシャル、一般の角運動量 k:バネ定数、波数、Boltzmann定数 K:運動エネルギー
l,L:長さ L:Lagrangian、角運動量、インダクタンス m,M:質量 n:物質量 N:個数、トルク
M:磁化 O:原点 p:双極子モーメント p,P:運動量、圧力 P:分極 q:波数
q,Q:一般化座標、電荷 Q:熱 r:距離 R:抵抗、気体定数 S:エントロピー、面積 t:時間 T:温度
U:ポテンシャル、内部エネルギー v:速度 V:体積、ポテンシャル、電位
W:仕事、状態数 x,y,z:変数、位置 z:複素変数 Z:分配関数

4 :ご冗談でしょう?名無しさん:2024/02/15(木) 11:39:38.52 ID:???.net
https://twitter.com/media_otb/status/1737362000924332417?t=unxeF6Xih4SPebsnb3ADPA&s=19
(deleted an unsolicited ad)

5 :poem:2024/02/15(木) 13:54:54.77 ID:aHf5ZeKy.net
スレ建て乙

偽物スレも皆
使って欲しい
消費お願い
へるぷみー

6 :poem:2024/02/15(木) 13:58:25.14 ID:aHf5ZeKy.net
>>4 は詭弁の分類にもある話で、反論の段階の図の写真だった。図の内容自体を見たことは何回もある

7 :poem:2024/02/15(木) 14:01:45.43 ID:aHf5ZeKy.net
図の内容自体を見たことは何回もあるけど
そういう人を減らす核心的なノウハウは無い
分類だけだから核心的なノウハウ欲しいよね

陰謀論者はソースが循環回帰していて、エコーチャンバー内しか参照してないとか
こちら記事見たことあり、こちらの方がまだ実用的=上の分類より微かだけ核心的な位置
リンクは保存してない

8 :ご冗談でしょう?名無しさん:2024/02/15(木) 18:50:13.74 ID:???.net
エコーチェンバーな

9 :ご冗談でしょう?名無しさん:2024/02/15(木) 19:21:55.25 ID:???.net
マクスウェル方程式の積分形について、積分面が時間変化する場合に両辺の時間微分を計算するにはどうすれば良いのでしょうか?
積分面が時間変化しない場合は、d/dtが積分記号を通り抜けて被積分関数に∂/∂tで作用するのはよく見る話ですが、積分面Sが時間の関数である場合の扱いがわかりません。

10 :ご冗談でしょう?名無しさん:2024/02/15(木) 19:50:34.36 ID:???.net
でぼん

11 :ご冗談でしょう?名無しさん:2024/02/15(木) 21:17:40.98 ID:???.net
d/dt ∫dx [u(t)→v(t)] f(x) = f(v)v' - f(u)u'
xyzの三重積分でこれをする

12 :ご冗談でしょう?名無しさん:2024/02/15(木) 21:34:45.84 ID:???.net
>>11
なるほど!さんくす

13 :ご冗談でしょう?名無しさん:2024/02/15(木) 23:29:46.45 ID:???.net
ちなみに物質部分D/Dtの場合は範囲が時間依存する場合でもレイノルズの輸送定理で積分の中に入れられる
余計な項がついてくるが

14 :ご冗談でしょう?名無しさん:2024/02/16(金) 00:46:41.21 ID:???.net
https://i.imgur.com/clcVv0x.png

15 :ご冗談でしょう?名無しさん:2024/02/16(金) 17:38:03.30 ID:???.net
流体力学の式って電磁気にどのくらい適用できるんだろか?
なんかそんな事をしてるうちに流体力学の教科書の後編を書かずに亡くなった方もいたような・・・

16 :ご冗談でしょう?名無しさん:2024/02/16(金) 18:38:24.47 ID:???.net
適用できる式なんて全然おもい浮かばん

17 :ご冗談でしょう?名無しさん:2024/02/16(金) 18:39:09.48 ID:???.net
静電場,静磁場の式は完全流体の式と同じで線形だから流体力学を学習してれば簡単に理解できる

18 :ご冗談でしょう?名無しさん:2024/02/16(金) 19:19:04.78 ID:???.net
流体力学では質量密度ρがパラメータだけど電磁気学では電荷密度ρがパラメータという違いがあるよね
電磁流体力学では流体力学と電磁気学を両方扱うけど、密度ρはどっちの意味なんだろか?

19 :ご冗談でしょう?名無しさん:2024/02/16(金) 19:25:06.94 ID:???.net
物理量の違いは重要ではない、方程式が同じかどうかだ

20 :ご冗談でしょう?名無しさん:2024/02/16(金) 19:29:00.99 ID:???.net
>>19
プラズマを記述する方程式では、質量密度と電荷密度はどう扱われていますか?

21 :ご冗談でしょう?名無しさん:2024/02/16(金) 19:30:44.04 ID:???.net
完全流体の練習問題が既に解いてるなら、静電場,静磁場の練習問題も簡単に解ける
逆も言える。

22 :ご冗談でしょう?名無しさん:2024/02/16(金) 19:36:13.18 ID:???.net
質問に答えてほしい・・・

23 :ご冗談でしょう?名無しさん:2024/02/16(金) 20:07:11.56 ID:???.net
その教科書みるかググレ

24 :ご冗談でしょう?名無しさん:2024/02/16(金) 20:09:09.05 ID:???.net
>>23
「その教科書」ってどれのことですか?
あと、「ググレ」って何を「ググレ」ばいいんですか?

25 :ご冗談でしょう?名無しさん:2024/02/16(金) 20:10:27.93 ID:???.net
>>11

>>17
って同一人物なの?
ややこしいからコテハンつけてほしいな・・・

26 :ご冗談でしょう?名無しさん:2024/02/16(金) 20:13:19.16 ID:???.net
>>11
は書き込みから賢い人だとわかるけど
>>17
は書き込みからかなりアホだとわかる

27 :ご冗談でしょう?名無しさん:2024/02/16(金) 20:21:54.39 ID:???.net
>>11>>17が別人だった場合、>>17の人はこれから書き込む際は名前欄にアホって書き込んでおいてね。

28 :ご冗談でしょう?名無しさん:2024/02/16(金) 20:26:22.39 ID:???.net
アホでなければ”プラズマ”の式が完全流体などとは似ても似つかぬ複雑な式になる
くらいググらなくても判る

29 :ご冗談でしょう?名無しさん:2024/02/16(金) 20:31:16.84 ID:???.net
電磁場のエネルギーと、プラズマ粒子のエネルギーの比によって完全流体と見なせる場合もあるんじゃないの?
「似ても似つかない」と断言する理由は何?

30 :11:2024/02/16(金) 20:33:24.98 ID:???.net
>>25
別人です

31 :ご冗談でしょう?名無しさん:2024/02/16(金) 20:37:14.12 ID:???.net
>>20 >>22
このアホは 完全流体と静電気の式が同じなら〜という意味を プラズマの式でも同じだ
と勝手に決めつけてる大アホだったということ

32 :ご冗談でしょう?名無しさん:2024/02/16(金) 20:41:13.92 ID:???.net
>>30
さんくす。もしよろしければこのスレ内だけでも11の名前で書き込んで下さい。 名無しのバカ老人の書き込みと区別しやすいから。

33 :ご冗談でしょう?名無しさん:2024/02/16(金) 20:41:15.05 ID:???.net
>>29
オマエの屁理屈は、水でも完全流体と見なせる場合があると言ってるのと変わらん

34 :ご冗談でしょう?名無しさん:2024/02/16(金) 20:43:17.54 ID:???.net
>>33
水でも完全流体と見なせる場合があることが「屁理屈」なんですか?
レイノルズ数の範囲によっては完全流体として扱えますが?

35 :ご冗談でしょう?名無しさん:2024/02/16(金) 20:47:18.36 ID:???.net
>>34
アホ、近似はどうでもいい
基の方程式が違えば異なる現象なんだよ

36 :ご冗談でしょう?名無しさん:2024/02/16(金) 20:48:48.71 ID:???.net
>>35
「基の方程式」ってなんですか?
ナビエストークス方程式のことですか?

37 :ご冗談でしょう?名無しさん:2024/02/16(金) 20:56:24.38 ID:???.net
「近似はどうでもいい」
ってのも激しく頭の悪い書き込みだな
適切な近似をするから数理的に現象を記述できるんだけど

38 :ご冗談でしょう?名無しさん:2024/02/16(金) 20:56:39.73 ID:???.net
でぼーーん

39 :ご冗談でしょう?名無しさん:2024/02/16(金) 21:08:57.86 ID:???.net
流体力学における適切な近似の重要性は、各種の無次元化パラメータの有用性から明らかだ。

https://ja.wikipedia.org/wiki/%E6%B5%81%E4%BD%93%E5%8A%9B%E5%AD%A6%E3%81%AE%E7%84%A1%E6%AC%A1%E5%85%83%E6%95%B0%E4%B8%80%E8%A6%A7

40 :ご冗談でしょう?名無しさん:2024/02/16(金) 21:23:56.65 ID:???.net
>>37
粘着アホか

完全に同じ式か違うかという単純な話と、
オマエの特定の近似なら同じ様になるとかは別の話だ

41 :ご冗談でしょう?名無しさん:2024/02/16(金) 21:27:32.78 ID:???.net
>>40
「完全に同じ式か違うかという単純な話」ってのはどの書き込みのことですか?
「オマエの特定の近似なら同じ様になる」ってのはどの書き込みのことですか?
上記2点が、全く何言ってるのかわからないので、落ち着いて論点を整理して書き込んでね。

42 :ご冗談でしょう?名無しさん:2024/02/16(金) 21:28:42.89 ID:???.net
一応再掲
https://twitter.com/media_otb/status/1737362000924332417?t=unxeF6Xih4SPebsnb3ADPA&s
(deleted an unsolicited ad)

43 :ご冗談でしょう?名無しさん:2024/02/16(金) 21:37:51.03 ID:???.net
>>41
探せんのか
>>17,19
>静電場,静磁場の式は完全流体の式と同じで線形だから流体力学を学習してれば簡単に理解できる
>物理量の違いは重要ではない、方程式が同じかどうかだ
だけだろが、式が同じとは近似の意味など含まないのが明らかだ。

お前らが勝手に膨らませて屁理屈に使ってるだけ

44 :ご冗談でしょう?名無しさん:2024/02/16(金) 21:39:04.69 ID:???.net
>>43
質問(>>41)に答えてほしいのですが、能力的に難しいでしょうか?

45 :ご冗談でしょう?名無しさん:2024/02/16(金) 21:47:38.61 ID:???.net
>>44
>>41 だけで意味は十分だろ

粘着アホの誰かへのレスなど無用

46 :ご冗談でしょう?名無しさん:2024/02/16(金) 21:49:53.98 ID:???.net
>>45
訂正 >>43 だけで意味は十分だろ

47 :ご冗談でしょう?名無しさん:2024/02/16(金) 21:58:19.87 ID:???.net
アホアホ連呼してるだけの人のようなので意思疎通は不可能と判断しました。

48 :ご冗談でしょう?名無しさん:2024/02/16(金) 22:02:10.91 ID:???.net
アホアホ連呼は>>42でいうところの最低辺層の人ね。
同じ層の人同士じゃないと意思疎通は無理だろう。

49 :ご冗談でしょう?名無しさん:2024/02/16(金) 22:08:44.16 ID:???.net
>>43
アホでなければ
係数1として Y = -∇φ と E = -∇U  は同じ式
Y = -∇φ と Y = -∇φ+x  は違う式

>>47の粘着アホにはそれすら分からんらしい。

50 :ご冗談でしょう?名無しさん:2024/02/16(金) 22:12:15.64 ID:???.net
記号の定義もせずに式を書き散らかすのは賢くない証拠でしょうね。

51 :ご冗談でしょう?名無しさん:2024/02/16(金) 22:15:45.48 ID:???.net
>>50
だからオマエはアホなんだよ

52 :ご冗談でしょう?名無しさん:2024/02/16(金) 22:25:44.08 ID:???.net
爺は毎回中身空っぽで出てきて、取り繕うことと虚勢張ることばっかに神経使って虚しくならんのかね

53 :ご冗談でしょう?名無しさん:2024/02/16(金) 23:24:42.46 ID:???.net
侮蔑のためだけに一知半解を振り回す奴など無視が最良

54 :ご冗談でしょう?名無しさん:2024/02/16(金) 23:34:15.87 ID:???.net
>>18
俺のような素人がざっと調べた範囲では、
質量密度をρ、
電荷密度をζ、
として扱うが、両者は数密度に比例するという素朴な仮定でうまくいくようだ。

55 :ご冗談でしょう?名無しさん:2024/02/16(金) 23:35:43.08 ID:???.net
>>54
の理解に至った資料は
https://www-space.eps.s.u-tokyo.ac.jp/~yokoyama/lecture/uchukukan2/plasmabasiceq_v0.pdf

56 :ご冗談でしょう?名無しさん:2024/02/17(土) 01:18:52.91 ID:???.net
アホ草
URLコピペで終わりのワンパターン

57 :ご冗談でしょう?名無しさん:2024/02/17(土) 02:27:27.18 ID:???.net
でぼぼぼ でぼぼぼぼぼ でぼぼぼぼぼぼぼぼ

58 :ご冗談でしょう?名無しさん:2024/02/17(土) 13:16:28.48 ID:???.net
本当にアホしか言えんのだなこいつは

59 :ご冗談でしょう?名無しさん:2024/02/17(土) 18:57:12.53 ID:???.net
URL押す気ないから何も分からんなー

60 :ご冗談でしょう?名無しさん:2024/02/18(日) 12:55:51.14 ID:/UEQgWMs.net
量子力学の質問です。
無限に深い1次元井戸型ポテンシャルでは運動量の不確定性は Δp = nh/2a (aは井戸の幅)で計算できたと思いますが、
無限に深い3次元井戸型ポテンシャルでのΔpはどう計算したらよいのでしょうか?

61 :ご冗談でしょう?名無しさん:2024/02/18(日) 13:24:05.81 ID:???.net
波動関数を井戸の内側の体積で正規化すればオッケー牧場👌

62 :ご冗談でしょう?名無しさん:2024/02/18(日) 13:33:53.57 ID:???.net
>>61
具体的には?

63 :ご冗談でしょう?名無しさん:2024/02/18(日) 14:06:56.45 ID:???.net
ψ(x,y,z)=ψ_1(x)ψ_2(y)ψ_3(z)と変数分離できると仮定します😳
なんやかんやシュレディンガー方程式を解きます😳
そして運動量の2乗の期待値と運動量の期待値の2乗の差を計算して分散を求めます😳

64 :ご冗談でしょう?名無しさん:2024/02/18(日) 14:12:41.05 ID:???.net
>>63
そいで、そいで?

65 :ご冗談でしょう?名無しさん:2024/02/18(日) 16:11:14.39 ID:???.net
ムーニーちゃんしんぷのおじさんはすごいんだぞ

66 :ご冗談でしょう?名無しさん:2024/02/18(日) 18:21:12.05 ID:???.net
井戸型ポテンシャルの運動量て離散的だろ
不確定性て何?

67 :ご冗談でしょう?名無しさん:2024/02/18(日) 18:47:43.00 ID:???.net
>>66
> 井戸型ポテンシャルの運動量て離散的だろ
そうだね

> 不確定性て何?
位置の不確定性Δx、運動量の不確定性Δp の求め方はこのへんを見てくれ
https://batapara.com/archives/infinite-potential-well.html/

68 :ご冗談でしょう?名無しさん:2024/02/18(日) 19:57:02.44 ID:???.net
いうほど離散的か?
エネルギーならわかる

69 :ご冗談でしょう?名無しさん:2024/02/18(日) 20:01:57.66 ID:???.net
エネルギーが離散的なら必然的に運動量も離散的だろ

70 :ご冗談でしょう?名無しさん:2024/02/18(日) 21:55:27.00 ID:???.net
>>69
あ、たしかにw
位置と間違えたわごめんw

71 :ご冗談でしょう?名無しさん:2024/02/19(月) 11:55:14.29 ID:???.net
でもrもpも連続量なのにL=r×pになると必ず離散量になるって不思議だね

72 :ご冗談でしょう?名無しさん:2024/02/19(月) 13:02:19.41 ID:???.net
範囲が有限だと離散量になる
フーリエ変換とフーリエ級数の差だな

73 :ご冗談でしょう?名無しさん:2024/02/21(水) 01:26:56.43 ID:???.net
浮力はなぜ生じるのですか?

74 :ご冗談でしょう?名無しさん:2024/02/21(水) 02:28:48.08 ID:oXLFLeT9.net
水素とか素粒子って個体差ってあるんですか?
猿とか人とかミジンコとかはまったく同一のはないとおもうのですが
それらはDNAは完全コピーであっても食べ物や行動や思考の差で違いが出るとおもうのですが
たとえば体重

75 :ご冗談でしょう?名無しさん:2024/02/21(水) 02:56:49.45 ID:???.net
相対論の光速度一定の話ですが
星 地球の観測者→ 別の星
こういう位置関係と運動をしてる場合、地球の観測者がどうにかして両方の星からくる光の速度を測定できたとしたら両方とも同じ速度になりますか?

76 :ご冗談でしょう?名無しさん:2024/02/21(水) 08:28:34.63 ID:???.net
>>75
慣性系ならそうなりますね
「光速度不変の原理」といいますが、ここで言う「光速度」は向きを含みません

77 :ご冗談でしょう?名無しさん:2024/02/21(水) 09:23:10.71 ID:???.net
でぼんしゃいあー パー

78 :ご冗談でしょう?名無しさん:2024/02/21(水) 09:52:12.83 ID:j+C+kgWL.net
大先生の世紀の大発見!これはノーベル賞!

680名無CCDさん@画素いっぱい (ワッチョイ 2d5f-fe9w)
2022/04/03(日) 03:51:37.39ID:fwRVin6o0
誤) 有限でなフーリエ変換あら無限に伸びる
正) フーリエ変換を無限級数展開すれば無限に伸びる

普通、光学系とデジカメでレンズのフーリエ変換を再現するときは有限離散フーリエ変換が用いられるので
矩形フィルターといえどもその周波数成分は有限の範囲にとどまる。けどまぁすさまじく広い周波数成分を持つのは変わらない
そしてこれが光芒を生み出す

498名無CCDさん@画素いっぱい
2024/02/15(木) 20:42:54.63ID:uNrUjZGf0
フーリエ展開そのものが無限に足し合わせるものだから、別に無限に伸びることはおかしくもなんともない

526名無CCDさん@画素いっぱい
2024/02/16(金) 00:41:22.25ID:oxaKToIe0
何を無限に足し合わせて → 波
何が無限に伸びるの → 波の数

535名無CCDさん@画素いっぱい
2024/02/16(金) 02:03:26.02ID:oxaKToIe0

無限に波を足し合わせるので円形開口のフーリエ変換像である光芒のサイズが数学的には無限のびる

mevius.5ch.net/test/read.cgi/dcamera/1687261331/

79 :ご冗談でしょう?名無しさん:2024/02/21(水) 12:57:46.57 ID:???.net
>>73
なんでググらないの?

80 :ご冗談でしょう?名無しさん:2024/02/21(水) 13:01:15.24 ID:???.net
>>74
素粒子は場の励起に過ぎない
電光掲示板と同じで、その位置が点灯してる状態なだけ
個体差などない

81 :ご冗談でしょう?名無しさん:2024/02/21(水) 13:20:19.92 ID:???.net
>>74
個体差がないことによって引き起こされる統計的に重要な帰結があり、それは確認されている。
つまり個体差はない。

82 :ご冗談でしょう?名無しさん:2024/02/21(水) 15:50:48.61 ID:???.net
>>78
何だこれ
文章どころか言葉になってないぞ

83 :ご冗談でしょう?名無しさん:2024/02/21(水) 17:09:01.90 ID:???.net
浮力は、まわりの流体が物体の下に回り込もうとする力であると言える
潜水艦が海底にぴったりくっつくと浮力が失われるらしい

84 :ご冗談でしょう?名無しさん:2024/02/21(水) 18:16:55.58 ID:???.net
>>73
トランポリンと同じ
水の中に箱を入れるとすると、箱の下の水のほうが重みが掛かってるので跳ね返す力が大きい

85 :ご冗談でしょう?名無しさん:2024/02/21(水) 18:32:38.67 ID:???.net
>>83-84
こう言う出鱈目を信じちゃアカンよ
ググった方が良いからね

86 :ご冗談でしょう?名無しさん:2024/02/21(水) 23:45:51.28 ID:6Aiwsw39.net
浮力は、鉄が沈むとすると、比重8の鉄が1の水を持ち上げ、水も鉄を持ち上げようと力を作用をさせる。従って、鉄はマイナス1の比重7の重さになる。

確かに海底に綺麗に付くなら、浮力なくなるのかも。

滑車に一キロと八キロのものを載せた状況かな。八キロのものは一キロ分重さ減る。水中の重さ計るのは、滑車に載せたこの八キロ側を計ってるようなもんだろね。

87 :ご冗談でしょう?名無しさん:2024/02/21(水) 23:58:18.29 ID:???.net
質問者以外でID出してるの奴の痛文書いてくる率は異常

88 :ご冗談でしょう?名無しさん:2024/02/22(木) 00:02:13.07 ID:???.net
いつもの荒らしでしょ

89 :ご冗談でしょう?名無しさん:2024/02/22(木) 00:08:26.38 ID:FHJJiSBg.net
もし比重が1の物体なら、水に沈むこともなく、浮いて出ることもないはず。平坦になるだろね。

もし比重が0なら、水に接するだけで沈むことはないはず。従って、比重が1より小さい物体はその中間の、ちょっと漬かった状態になるだろう。その物体が比重1になるような漬かり方で。

90 :ご冗談でしょう?名無しさん:2024/02/22(木) 00:30:19.64 ID:???.net
海底にピッタリくっつく場合って吸盤ひっつけたときと同じじゃない?
陰圧で沈んだまま

91 :ご冗談でしょう?名無しさん:2024/02/22(木) 08:05:49.06 ID:JalrONoU.net
>>80
"場の励起に過ぎない状態"が電荷やスピンを有するのは何故ですか?
電荷を与える何か、スピンを与える何かが存在するのでしょうか

92 :ご冗談でしょう?名無しさん:2024/02/22(木) 09:52:38.68 ID:???.net
ゲージ変換や空間回転変換に対する波動関数の変換性、とか言い換えは可能だけど、じゃぁその変換性を与える何かが存在するのかとか言い出したらキリがない。最終的には場の性質がそうなっているからとしか言いようがない

93 :ご冗談でしょう?名無しさん:2024/02/22(木) 10:28:36.15 ID:???.net
浮力はベクトル解析を使わないと記述できないことはあまり知られていない

94 :ご冗談でしょう?名無しさん:2024/02/22(木) 10:58:39.73 ID:???.net
物体が流体を排除している部分を仮想的にその流体で置き換えれば、その流体は浮きも沈みもせずその場にとどまるはずである。つまりこの場合その置き換えた流体にかかる浮力はその流体の重量と釣り合っている。
置き換えた流体を元の物体に戻しても、物体にかかる浮力は変わらない。よって物体が排除している流体の重量と同じ大きさの浮力を受けるというアルキメデスの原理が導かれる

95 :ご冗談でしょう?名無しさん:2024/02/22(木) 12:15:14.87 ID:???.net
>その流体は浮きも沈みもせずその場にとどまるはずである。

ここが間違い
その部分を着色すればその場に留まっていないことは明らかだろう

96 :ご冗談でしょう?名無しさん:2024/02/22(木) 13:07:35.96 ID:???.net
気になるなら無限に薄く軽い仮想の不透膜で囲えばいいだけ

97 :ご冗談でしょう?名無しさん:2024/02/22(木) 13:26:18.70 ID:???.net
物体に及ぼされる流体の圧力を物体の表面全体にわたって積分すれば定量的に出てくる
その際ベクトル解析(というかガウスの発散定理)を用いれば境界面積分が体積分に置き換わって計算が楽になるだろうが、発散定理の証明と同等の計算を地道に行えばいいだけだから、ベクトル解析を使わなければ記述できないわけでもないわな

98 :ご冗談でしょう?名無しさん:2024/02/22(木) 14:28:09.19 ID:???.net
その膜がその場に留まるという根拠がないな
現実の薄くて軽い膜は水の中に入れると動くし

99 :ご冗談でしょう?名無しさん:2024/02/22(木) 15:43:20.82 ID:???.net
>>93
>浮力はベクトル解析を使わないと記述できない

凡人学生ができる正しい選択肢は2つに限られる
1.実験事実を基にしたアルキメデスの(浮力)原理を物理原理として認め適用する

2.流体力学の基礎(ベクトル解析)を学習を学習し、流体静力学からアルキメデスの原理
を導出する。

100 :ご冗談でしょう?名無しさん:2024/02/22(木) 16:12:03.85 ID:???.net
>>98
膜が動く根拠は無いな

101 :ご冗談でしょう?名無しさん:2024/02/22(木) 18:11:22.54 ID:???.net
>>100
根拠は書いてあるが

102 :ご冗談でしょう?名無しさん:2024/02/22(木) 18:36:41.05 ID:???.net
パスカルの原理も知らん馬鹿は黙ってりゃいいのに

103 :ご冗談でしょう?名無しさん:2024/02/22(木) 18:37:17.32 ID:???.net
静止流体が前提で体積0質量0(無限小)の膜は動かん
体積0質量0なら加わる力は圧力のみ
静止流体の圧力は相殺しているから力なし

104 :ご冗談でしょう?名無しさん:2024/02/22(木) 18:41:39.17 ID:???.net
https://www.history.navy.mil/content/dam/museums/psnm/education/documents/kids%20activities/BuildYourOwnSubmarine.pdf

A submarine floating on the ocean’s surface has positive buoyancy. However, when a submarine sinks to the ocean’s floor it has negative buoyancy.

海面に浮かぶ潜水艦は正の浮力を持つ。しかし、潜水艦が海底に沈むと負の浮力を持つ。

105 :ご冗談でしょう?名無しさん:2024/02/22(木) 18:50:52.10 ID:???.net
無限小の膜はわずかな力で動く
圧力揺らぎで動くだろう

106 :ご冗談でしょう?名無しさん:2024/02/22(木) 18:53:11.15 ID:???.net
浮力の問題にブラウン運動持ち込むような馬鹿がいるスレ

107 :ご冗談でしょう?名無しさん:2024/02/22(木) 18:53:11.59 ID:???.net
押しのけた流体の重さに等しい浮力が生じる
というのを馬鹿の一つ覚えで使うと>>104は説明できない

108 :ご冗談でしょう?名無しさん:2024/02/22(木) 18:54:33.37 ID:???.net
アルキメデスの原理を説明できない馬鹿しかいない

109 :ご冗談でしょう?名無しさん:2024/02/22(木) 18:56:57.92 ID:???.net
ベクトル解析でメカニズムが説明されているから、アルキメデスの原理はもはや原理ではない。

110 :ご冗談でしょう?名無しさん:2024/02/22(木) 20:50:07.18 ID:KQupng8d.net
水槽の中にモノを沈めると、水槽の水位は上がるわけで、つまりモノが水に仕事してるわけなんだよね。

水が沈み続けるうちは、モノがより下に動くことで水全体の位置エネルギーが増える。モノが水を押し上げてる。その押し上げる水量がモノの体積に等しいから、その体積分の水の重さ分、モノの落下の加速度が低下する。つまり浮力がある、と見える。

だから、水底に沈んだものはもはや水をそれ以上持ち上げる余地がないので、浮力が生じないかもしれない、と推測できる。

111 :ご冗談でしょう?名無しさん:2024/02/22(木) 21:17:53.32 ID:???.net
モノを海に入れると、海の水位って上がるの?押し上げる水量が浮力の原因なら海では浮力は生じなさそう。

112 :ご冗談でしょう?名無しさん:2024/02/22(木) 21:27:02.68 ID:???.net
地上の空気って1立方メートルで1.2kgあるようだけど、なんでこの浮力は感じないんだろうか?

113 :ご冗談でしょう?名無しさん:2024/02/22(木) 21:35:25.43 ID:???.net
70kgの人間の体積は0.07立方メートル程度だから、浮力は0.08kg重程度か。80gじゃ気づかないな。

114 :ご冗談でしょう?名無しさん:2024/02/22(木) 21:45:23.78 ID:???.net
仮に浮力が8kgだったら気づくんかいな?

115 :ご冗談でしょう?名無しさん:2024/02/22(木) 22:55:51.34 ID:???.net
浮力を感じるって、具体的にどんなふうに感じると?

116 :ご冗談でしょう?名無しさん:2024/02/22(木) 22:58:39.56 ID:???.net
人の感覚は大気圧下に適応しているから感じないだけ

117 :ご冗談でしょう?名無しさん:2024/02/22(木) 23:55:36.85 ID:???.net
80gは1円玉80枚
さすがに気づくよ

118 :ご冗談でしょう?名無しさん:2024/02/23(金) 00:02:15.26 ID:???.net
毎日浮力感じながら生きてる奇特な方がきました

119 :ご冗談でしょう?名無しさん:2024/02/23(金) 00:27:12.36 ID:???.net
水素で満たされた部屋に入って違いがわかるかな?
自分の体重が1kg変わっても分からんから無理

120 :ご冗談でしょう?名無しさん:2024/02/23(金) 00:35:55.15 ID:???.net
>>107
解釈次第でアルキメデスの原理が成り立つとも見なせる

121 :ご冗談でしょう?名無しさん:2024/02/23(金) 00:41:08.86 ID:TqLaGZO+.net
海に入れても多分水位が上がると思うけど、極端な話し、地球の海全体に広がるから、海面の高さの変化を計測するのは難しいと思う。

水槽は一定の範囲で水位上昇を計測可能だけど、海は広大だし、他の要因も関連するわけだからさ。

もし仮に水位が上がらないとすると、水が圧縮されてることになって、これは実験や観測事実と矛盾する。従って、海では水位上昇は起きてるけど認識できてないだけ、とみなすのが妥当でしょ。

実際はわりと局所的な水位上昇が起きて、それが動的だから捉えきれないんだと思うけども。

122 :ご冗談でしょう?名無しさん:2024/02/23(金) 01:02:59.06 ID:TqLaGZO+.net
人間の密度はかなり水に近いんだろね。だから、水に入ると体重が浮力で相殺されてかなり軽くなるはず。これは、ある意味無重力状態みたいに、特定方向に動かされないのに近い。だから、NASAの宇宙飛行士は水中で訓練したりするんだろね。

学校の水泳授業のとき、太り気味の子が飛び込んで風船みたいに沈まずに浮いてクロールしてたのが印象的だったけど、体脂肪多いと、比重1下回る可能性あるのかも。

123 :ご冗談でしょう?名無しさん:2024/02/23(金) 02:47:26.69 ID:???.net
なんだろう、なんとなく喋り方がキモい

124 :ご冗談でしょう?名無しさん:2024/02/23(金) 13:10:47.91 ID:???.net
普通の人は水に浮くけど
鍛えた筋肉質の人は沈む
以上常識

125 :ご冗談でしょう?名無しさん:2024/02/23(金) 17:13:18.45 ID:???.net
いわゆるカナヅチですね

126 :ご冗談でしょう?名無しさん:2024/02/23(金) 18:29:51.87 ID:???.net
でぼーん

127 :ご冗談でしょう?名無しさん:2024/02/24(土) 06:15:47.60 ID:???.net
太ったひーとが
こーんなーにも
浮くとは思わずにーいたー♪

128 :ご冗談でしょう?名無しさん:2024/02/24(土) 11:26:59.94 ID:???.net
でぼーーん

129 :ご冗談でしょう?名無しさん:2024/02/24(土) 16:18:44.38 ID:???.net
液体気体には浮力あるのに固体にはない不思議

130 :ご冗談でしょう?名無しさん:2024/02/24(土) 16:55:30.41 ID:???.net
固体の浮力って何?

131 :ご冗談でしょう?名無しさん:2024/02/24(土) 17:38:45.31 ID:???.net
アルキメデスの原理の導出は固体でも通用する

132 :ご冗談でしょう?名無しさん:2024/02/24(土) 18:00:51.83 ID:???.net
本当?
やってみて

133 :ご冗談でしょう?名無しさん:2024/02/24(土) 18:37:29.97 ID:???.net
固体の「圧力」はゼロだと思うんだけど違うの?

134 :ご冗談でしょう?名無しさん:2024/02/24(土) 18:56:27.03 ID:???.net
固体の地面に圧力がなかったら地球の中心まで自由落下してしまうわ

135 :ご冗談でしょう?名無しさん:2024/02/24(土) 19:04:24.85 ID:???.net
それは重力の反作用でしょ?圧力は別。

136 :ご冗談でしょう?名無しさん:2024/02/24(土) 19:37:56.75 ID:???.net
>>135
> それは重力の反作用でしょ?
違うわ

きみが地面に沈んでいかないのは、足の裏に地面からの圧力がかかってるからやで

137 :ご冗談でしょう?名無しさん:2024/02/24(土) 19:47:34.66 ID:???.net
いやだから、それ圧力項じゃなくて重力の反作用項でしょ
流体の圧力ってナビエストークス方程式でどの項か分からないの?

138 :ご冗談でしょう?名無しさん:2024/02/24(土) 19:49:59.91 ID:???.net
>>137
だから違うよ
「反作用」の意味と「圧力」の意味どちらもわかってないのかいな?

139 :ご冗談でしょう?名無しさん:2024/02/24(土) 19:50:19.47 ID:???.net
力のつり合いと作用・反作用わかってないやん

140 :ご冗談でしょう?名無しさん:2024/02/24(土) 19:51:56.09 ID:???.net
固体の圧力ってナビエストークス方程式で記述できるの?

141 :ご冗談でしょう?名無しさん:2024/02/24(土) 19:52:30.22 ID:???.net
まあ固体に圧力がないと思うなら、裸足で足つぼマットの上歩いてみ
嫌でも実感できるわ

142 :ご冗談でしょう?名無しさん:2024/02/24(土) 19:52:42.37 ID:???.net
というか固体の圧力って何?

143 :ご冗談でしょう?名無しさん:2024/02/24(土) 19:53:32.14 ID:???.net
シリンダの中に固体入れるとするじゃん?
シリンダを固体が押すの?

144 :ご冗談でしょう?名無しさん:2024/02/24(土) 19:58:17.78 ID:???.net
そりゃもちろん
空中に浮いてるわけでもない限りな

145 :ご冗談でしょう?名無しさん:2024/02/24(土) 20:00:23.37 ID:???.net
固体でも分子の熱運動があるから圧力あるじゃん

146 :ご冗談でしょう?名無しさん:2024/02/24(土) 20:03:59.12 ID:???.net
熱運動があるからという理屈がわからんけど、そんなこと考えるまでもなく圧力はあるよね
足つぼマットの上歩いて痛いのは圧力かかってるからじゃないのかい

147 :ご冗談でしょう?名無しさん:2024/02/24(土) 20:04:07.48 ID:???.net
>>145
固体分子の熱運動が「固体の圧力」なんですか?正気ですか?

148 :ご冗談でしょう?名無しさん:2024/02/24(土) 20:11:36.24 ID:???.net
縮退圧のはなし?

149 :ご冗談でしょう?名無しさん:2024/02/24(土) 20:12:05.99 ID:???.net
流体の圧力ってのは、押したら押し返される反作用の力ではなくて
流体と接しているだけで押される力のことを言うんだよ。
|_| ←みたいな形の容器のなかに、
流体を入れると側面に力が発生するけど、
容器と同じ形の固体を入れても側面には力は発生しないでしょ?

150 :ご冗談でしょう?名無しさん:2024/02/24(土) 20:14:01.12 ID:???.net
固体と容器の間に挟まってる気体と固体は力を及ぼし合わないか?

151 :ご冗談でしょう?名無しさん:2024/02/24(土) 20:15:38.33 ID:???.net
それは気体の圧力だね

152 :ご冗談でしょう?名無しさん:2024/02/24(土) 20:19:06.58 ID:???.net
>>149
固体だろうが液体だろうが「圧力」の意味は同じだわ
作用があれば必ず反作用があるのも同じ

>>151
気体の圧力と容器の圧力が作用反作用の関係にあるね
容器と接しているのが固体なら、固体の圧力と容器の圧力が作用反作用の関係にあるはずだね

153 :ご冗談でしょう?名無しさん:2024/02/24(土) 20:20:44.28 ID:???.net
容器側面に働く「圧力」は、流体と固体で全く異なる。

154 :ご冗談でしょう?名無しさん:2024/02/24(土) 20:23:29.38 ID:???.net
>>153
そら接し方が違えば圧力のかかりかたは異なるけど、圧力の意味は変わらんわな

155 :ご冗談でしょう?名無しさん:2024/02/24(土) 20:29:43.93 ID:???.net
ナビエストークス方程式を一度でも見たことがあれば、単純な「圧力項」は存在しないことは誰でもわかる。

156 :ご冗談でしょう?名無しさん:2024/02/24(土) 20:38:52.02 ID:???.net
あと、残念ながらナビエストークス方程式に「熱運動」の項は無いよw

157 :ご冗談でしょう?名無しさん:2024/02/24(土) 20:41:59.28 ID:???.net
そらそうだろう、なにが残念なのか知らんが

158 :ご冗談でしょう?名無しさん:2024/02/24(土) 20:43:06.89 ID:???.net
きみにいってるんじゃなくて

145
sage
02/24(土) 20:00:23.37
固体でも分子の熱運動があるから圧力あるじゃん

↑この人にいってる

159 :ご冗談でしょう?名無しさん:2024/02/24(土) 20:44:22.77 ID:vTVh4Zx/.net
巨大質量星の進化と超高輝度超新星
https://www.asj.or.jp/geppou/archive_open/2014_107_07/107_384.pdf

P385の右上で主系列星時代水素燃焼核が内部に後退していっているけど、
これは赤色巨星に進化直後に中心部に向かってヘリウム濃度が濃くなっていく
勾配が発生しているという事だよな?
だとするとそこで発生する対流はどうなるんだ?
ヘリウム濃度の濃い部分が直接最上位まで上昇できないとすると
多段式の対流が発生することになるのか?

160 :ご冗談でしょう?名無しさん:2024/02/24(土) 20:46:03.91 ID:???.net
流体は本当にムズいよ
流体力学は物理の学部のカリキュラムから除外されている

161 :ご冗談でしょう?名無しさん:2024/02/24(土) 20:47:20.56 ID:???.net
>>158
特定の奴に言ってるんだったらアンカしろよ

162 :ご冗談でしょう?名無しさん:2024/02/24(土) 20:47:55.69 ID:???.net
きみもねw

163 :ご冗談でしょう?名無しさん:2024/02/24(土) 20:49:59.61 ID:???.net
>>162
すぐ上のやつに言ってる場合はアンカしないこともあるけど、そうじゃなかったらアンカするわ

164 :ご冗談でしょう?名無しさん:2024/02/24(土) 21:04:40.77 ID:???.net
項の数だけで見れば
一般相対論のアインシュタイン方程式が4項で
流体のナビエストークス方程式は8項だから
流体のほうが手に負えないと言うこともできる

165 :ご冗談でしょう?名無しさん:2024/02/24(土) 21:12:27.43 ID:???.net
>>164
アインシュタイン方程式は略号で書かれてるから単純に見えるだけ
略号使わずに書き出したら手首折れるわ

166 :ご冗談でしょう?名無しさん:2024/02/24(土) 21:19:03.99 ID:???.net
剛体ならそら圧力は無いやろうけど弾性体ならあるやろ
っていうか剪断力もあるからもっと広く応力があると言える
材料力学やで

167 :ご冗談でしょう?名無しさん:2024/02/24(土) 21:19:51.14 ID:???.net
>>164
アインシュタイン方程式を計量テンソルの要素として書き下したら200項超えたはず

168 :ご冗談でしょう?名無しさん:2024/02/24(土) 21:25:00.49 ID:???.net
>>166
剛体なら圧力無いってどういう理屈なん?
剛体の足つぼマットなら裸足で歩いても痛くないんかいな?

169 :ご冗談でしょう?名無しさん:2024/02/24(土) 21:49:46.27 ID:???.net
アインシュタイン方程式のテンソルってたかだか2階で、対称性も仮定できるんじゃなかったっけ?

170 :ご冗談でしょう?名無しさん:2024/02/24(土) 21:54:45.60 ID:???.net
クレイ研究所のミレニアム問題にナビエストークス方程式の解の問題はあるけど、アインシュタイン方程式は全く触れられていなことからも、どっちが手に負えないかは一目瞭然だろう。

171 :ご冗談でしょう?名無しさん:2024/02/24(土) 22:01:49.74 ID:???.net
アインシュタイン方程式:厳密解がいくつも見つかっている
ナビエストークス方程式:厳密解が存在するかどうかすら不明

172 :ご冗談でしょう?名無しさん:2024/02/24(土) 22:03:48.81 ID:???.net
>>168
剛体の内部には圧力は無いやろ
そもそも剛体は仮想的物体やから種々の物理現象の辻褄が合わん部分も多いな

173 :ご冗談でしょう?名無しさん:2024/02/24(土) 22:13:04.56 ID:???.net
>>172
その理屈を聞いてるのに、なんでただ同じこともう一回いったん?

174 :ご冗談でしょう?名無しさん:2024/02/24(土) 22:29:03.68 ID:???.net
>>149
流体がシリンダーを押す力があるならそれの反作用としてシリンダーが流体を押す力もある
それが圧力

175 :ご冗談でしょう?名無しさん:2024/02/24(土) 22:32:44.87 ID:???.net
>>174
ちがいます

176 :ご冗談でしょう?名無しさん:2024/02/24(土) 22:36:50.65 ID:???.net
ちがわんやろ

177 :ご冗談でしょう?名無しさん:2024/02/24(土) 22:37:43.19 ID:???.net
>>173
剛体は変形せんやん?
せやから引張も圧縮も起こらんやろ?
あるいは剛体が生まれたときから固有で不変な圧力を各々持っとるとも言えるかもしれんが、考えても意味の無いことや
変形せんのやから外部に圧力による仕事をせん

178 :ご冗談でしょう?名無しさん:2024/02/24(土) 22:44:43.67 ID:???.net
応力と圧力の区別ができないとナビエストークス方程式は理解できません

179 :ご冗談でしょう?名無しさん:2024/02/24(土) 22:51:37.88 ID:vTVh4Zx/.net
>>159 の多段式の対流に関して、

普通の対流

→↓
↑↓
↑↓
↑←


なのに対して多段式の対流は、

→↓
↑←
→↓
↑←

と言う、イメージで話していた。

180 :ご冗談でしょう?名無しさん:2024/02/24(土) 22:56:31.30 ID:???.net
>>177
変形しないからといって力が伝わらんことにはならんよ
物体が硬いほど物体の変形が小さくなるが、圧力は同じように伝わるしむしろ硬いほど伝達速度が速くなるだけ
硬さ無限大まで拡張すれば変形は0だが突然圧力が伝わらなくなるという理屈にはならん

181 :ご冗談でしょう?名無しさん:2024/02/24(土) 23:04:10.94 ID:???.net
>>180
なるほど
確かに剛体を介した力の伝達を、剛体内部で圧力が速さ∞で伝播していってると考えれば圧力があると考えられるのか
剛体って条件が特殊だから難しいな

182 :ご冗談でしょう?名無しさん:2024/02/24(土) 23:29:07.91 ID:PrTChFnj.net
圧力の定義って何?

183 :ご冗談でしょう?名無しさん:2024/02/25(日) 00:44:54.25 ID:???.net
ストレステンソルの対角成分

184 :ご冗談でしょう?名無しさん:2024/02/25(日) 06:15:02.35 ID:???.net
でぼん でぼーん

185 :ご冗談でしょう?名無しさん:2024/02/25(日) 12:07:58.53 ID:???.net
>>182
応力の1種

186 :ご冗談でしょう?名無しさん:2024/02/25(日) 13:36:37.09 ID:HmIDFjWx.net
物理板の住人のIQだと以下は全問解けますか?
https://robotzzle.42.fr/index.html

187 :ご冗談でしょう?名無しさん:2024/02/25(日) 13:57:23.62 ID:???.net
Level6まで解いて、あと何問続くのかわからないので飽きた

188 :ご冗談でしょう?名無しさん:2024/02/25(日) 15:46:54.78 ID:???.net
IQ測定サイトって、全問解かせた後に結果を知りたければメールアドレスを登録しろとかいうクソサイトが多いから注意が必要

189 :ご冗談でしょう?名無しさん:2024/02/25(日) 16:03:52.38 ID:???.net
>>186 はただのパズルだからなんも測定せんよ
解くにはプログラミング的なセンスが必要

190 :ご冗談でしょう?名無しさん:2024/02/25(日) 16:30:58.74 ID:???.net
ノーヒントでゲームのルールを把握してLevel8までクリアできたらまあまあ賢いのではなかろうか
なぜなら俺と同レベルだから

191 :ご冗談でしょう?名無しさん:2024/02/25(日) 16:41:01.63 ID:???.net
level10まではクリアした

192 :ご冗談でしょう?名無しさん:2024/02/25(日) 18:44:35.37 ID:???.net
>>159
そのグラフの縦軸は質量目盛りだぞ
燃焼核の質量は減ってるけど温度が上がってるから距離は減ってるかどうか分からんぞ
対流については中心部に対流なんてないのでは?

193 :ご冗談でしょう?名無しさん:2024/02/25(日) 18:50:31.65 ID:???.net
http://hamanako-kankou.com/zukai/buoyancy/index.html

なぜか浜名湖観光局のサイトに浮力の詳しい説明ページがあった

194 :ご冗談でしょう?名無しさん:2024/02/25(日) 18:54:38.45 ID:???.net
相対論前提だと剛体って加速すらできないんだよな
減速もできないから止めることもできない
質量は無限大だね
宇宙ごとブラックホールに吸い込んで破滅だ

195 :ご冗談でしょう?名無しさん:2024/02/25(日) 19:49:34.87 ID:???.net
宇宙が破滅してる時点で、剛体も何もそこに存在してないよね

196 :ご冗談でしょう?名無しさん:2024/02/25(日) 22:05:17.96 ID:???.net
二次元のラプラス方程式Δφ=0は、コーシーリーマン方程式との関連で複素解析の結果を利用できますが
三次元以上のラプラス方程式ではそのような対応は無いのでしょうか?

197 :ご冗談でしょう?名無しさん:2024/02/25(日) 23:23:46.70 ID:q3ZIhXzn.net
ローレンツ力(F=qE+qv×B)はニュートン力学の範疇だがその名の通りすでにローレンツ共変性を具備している。
実際、相対論的運動方程式γdp/dt=γFは、
γF=γqE+qγv×B=qw_μνF^μν
となりローレンツ共変である。

198 :ご冗談でしょう?名無しさん:2024/02/25(日) 23:39:57.02 ID:q3ZIhXzn.net
ニュートンの運動方程式からエネルギー保存則を導出する際の定番操作:速度を掛けて時間積分する、
は相対論的運動方程式でも同様の手法で導出できる。
即ち、vdp/dt=vFの

左辺=vd(mγv)/dt
=mv^2dγ/dt+mvγdv/dt
=mv^2γ^3βdβ/dt+mvγdv/dt
=mγ(β^2γ^2+1)vdv/dt
=mγ^3vdv/dt
=d(mγc^2)/dt

一方、
右辺=vF
=Fdx/dt

辺々をtで積分すれば

mγc^2-∫Fdx=一定

の力学的エネルギー保存則を得る。

199 :ご冗談でしょう?名無しさん:2024/02/26(月) 00:38:53.63 ID:1T4JBF3a.net
ただし、相対論では一般に力場は空間だけでなく時間の関数となるため、
最後の空間積分への変形は留意が必要。
即ち、F=F(t,x)であるならば、
∫Fdx/dt=∫Fdx
とはならない。

200 :ご冗談でしょう?名無しさん:2024/02/26(月) 00:40:32.35 ID:1T4JBF3a.net
×∫Fdx/dt=∫Fdx
〇∫F(dx/dt)dt=∫Fdx

201 :ご冗談でしょう?名無しさん:2024/02/26(月) 11:21:56.70 ID:???.net
でぼんしゃいあー

202 :ご冗談でしょう?名無しさん:2024/02/26(月) 11:48:38.07 ID:???.net
>>197
ローレンツ力 q(E + H×v/c) は相対論と合わんぞ
例えば x 離れた2電荷q1,q2が y 方向に速度 v で動いてる場合の電磁気力は
ローレンツ力だと q1q2√(1-v^2/c^2)/x^2 だが相対論で計算すると q1q2(1-v^2/c^2)/x^2 で√(1-v^2/c^2)だけ違う

203 :ご冗談でしょう?名無しさん:2024/02/26(月) 12:22:00.29 ID:???.net
>>198
>力学的エネルギー保存則を得る。

デタラメ

204 :ご冗談でしょう?名無しさん:2024/02/26(月) 14:35:07.23 ID:???.net
携帯電波で騒ぐも工学者から反論されて黙る

covidで騒ぐもcovid自体が収束したので黙る

あのお方は次は何で騒ぐんだろ

205 :ご冗談でしょう?名無しさん:2024/02/26(月) 14:55:06.57 ID:???.net
https://news.yahoo.co.jp/articles/8f0de29b926e0f0b6ddb0484eeae1c0dbfd793d3

エネルギー保存則がついに破られた模様

206 :ご冗談でしょう?名無しさん:2024/02/26(月) 15:10:22.78 ID:???.net
>>205
アホ

207 :ご冗談でしょう?名無しさん:2024/02/26(月) 15:39:53.12 ID:???.net
>>205
すげえ

208 :ご冗談でしょう?名無しさん:2024/02/26(月) 16:08:59.00 ID:???.net
205のリンク先の技術はすげえが、これをエネルギー保存則の破れだと早合点する205はアホ

209 :ご冗談でしょう?名無しさん:2024/02/26(月) 17:14:04.96 ID:???.net
結局は漕ぐのが大変になる予感

210 :ご冗談でしょう?名無しさん:2024/02/26(月) 18:59:08.64 ID:1T4JBF3a.net
>>202
1つの電荷の話をしているのにいきなりもう一個電荷をもってこられてもな。

>>203
数学的に反証して。

211 :ご冗談でしょう?名無しさん:2024/02/26(月) 20:33:26.67 ID:???.net
>>202
そもそもローレンツ力それじゃなくね?

212 :ご冗談でしょう?名無しさん:2024/02/26(月) 23:15:20.29 ID:???.net
>>205の真相は
https://x.com/sinomoritsukasa/status/1761648624277352610?s=20

213 :ご冗談でしょう?名無しさん:2024/02/27(火) 02:53:37.73 ID:jjHhdekD.net
おまえらが九九を言えない理由

難しいからなあんなもの
覚える方法も解明されていない ばかなやつら

214 :ご冗談でしょう?名無しさん:2024/02/27(火) 02:54:37.73 ID:jjHhdekD.net
馬鹿は死ぬ寸前まで馬鹿でいろ
金次第だ

215 :ご冗談でしょう?名無しさん:2024/02/27(火) 05:04:37.85 ID:???.net
ムニちゃーんポポ

216 :ご冗談でしょう?名無しさん:2024/02/27(火) 13:53:55.78 ID:???.net
F=q(E+v×B)

217 :ご冗談でしょう?名無しさん:2024/02/27(火) 13:55:04.78 ID:???.net
=q(E+βc×B)

218 :ご冗談でしょう?名無しさん:2024/02/27(火) 14:06:55.94 ID:???.net
>>211
ローレンツ力単独じゃ意味がないからな

219 :ご冗談でしょう?名無しさん:2024/02/27(火) 14:36:59.21 ID:???.net
で、>>203の数学的反証はまだ?

220 :ご冗談でしょう?名無しさん:2024/02/27(火) 18:27:54.68 ID:???.net
逃げたんだろ
とは言え相対論的運動方程式なら時間は固有時にして欲しいがな

221 :ご冗談でしょう?名無しさん:2024/02/27(火) 19:41:47.94 ID:???.net
>>217
デタラメ

222 :ご冗談でしょう?名無しさん:2024/02/27(火) 20:34:02.61 ID:NKtfvqOF.net
直線上を運動する物体があります。
その加速度は a(t) です。

なめらかな曲線上を運動する物体があります。
曲線の接線方向の加速度は a(t) です。

このとき、直線上を運動する物体となめらかな曲線上を運動する物体の任意の時刻 t におけるスピードは等しいことを証明せよ。

これはどうやって解くのでしょうか?

223 :ご冗談でしょう?名無しさん:2024/02/27(火) 20:34:46.44 ID:NKtfvqOF.net
時刻 t = 0 において両物体とも静止していたとする。

224 :ご冗談でしょう?名無しさん:2024/02/27(火) 23:19:07.21 ID:???.net
>>222
速度を式で表せば良い

225 :ご冗談でしょう?名無しさん:2024/02/27(火) 23:32:10.56 ID:???.net
表せるもんなら表してみい

226 :ご冗談でしょう?名無しさん:2024/02/28(水) 08:42:59.78 ID:???.net
v=∫adt

227 :ご冗談でしょう?名無しさん:2024/02/28(水) 08:48:44.48 ID:???.net
aが同じで初速一緒ならvも一緒ってことですね

228 :ご冗談でしょう?名無しさん:2024/02/28(水) 10:45:08.34 ID:???.net
>>225
こんな事もできんのか

229 :ご冗談でしょう?名無しさん:2024/02/28(水) 11:48:11.52 ID:???.net
それは速度じゃなくて速さだ
しかもそれは結論としてわかってることで、それを証明するのが問題だろ

230 :ご冗談でしょう?名無しさん:2024/02/28(水) 12:51:13.28 ID:???.net
v=∫adtが証明だろ
aが同じで初速同じなら結果も同じになるんだから

231 :ご冗談でしょう?名無しさん:2024/02/28(水) 12:52:07.82 ID:???.net
ちなみにこれは速度なんだけど

232 :ご冗談でしょう?名無しさん:2024/02/28(水) 12:56:30.42 ID:???.net
証明の意味わかってる?

233 :ご冗談でしょう?名無しさん:2024/02/28(水) 13:12:20.84 ID:???.net
しんぷさんはムーニーちゃんダンスをおどれ
ムニちゃーんダンス
ムニちゃーんしんぷ

234 :ご冗談でしょう?名無しさん:2024/02/28(水) 13:54:26.83 ID:???.net
哀れやな

235 :ご冗談でしょう?名無しさん:2024/02/28(水) 18:20:25.93 ID:???.net
>>232
これ以上なく証明だが

236 :ご冗談でしょう?名無しさん:2024/02/28(水) 18:27:55.78 ID:???.net
曲線を弧長パラメータ表示すれば自明だね

237 :ご冗談でしょう?名無しさん:2024/02/28(水) 18:50:40.50 ID:???.net
積分の中身が同じ関数なんだから同じに決まってる
積分定数は0とする

238 :ご冗談でしょう?名無しさん:2024/02/28(水) 19:45:09.42 ID:???.net
接線方向と垂直の方向の加速度成分って、速さには全く寄与しないんだっけ?

239 :ご冗談でしょう?名無しさん:2024/02/28(水) 20:14:49.65 ID:???.net
そだね、少なくともそれを言わなきゃまったく証明になってないだろうに

240 :ご冗談でしょう?名無しさん:2024/02/28(水) 20:44:56.86 ID:???.net
接線方向の加速度が例えば100a(t)とかだったら速さめちゃくちゃ変わりそう

241 :ご冗談でしょう?名無しさん:2024/02/28(水) 20:46:18.17 ID:???.net
微積分すら分からない馬鹿にはそれも当然含めて証明出来てることが分からないらしい

242 :ご冗談でしょう?名無しさん:2024/02/28(水) 20:48:55.48 ID:???.net
v=∫adtでaと初期条件が同じ場合にvが同じことすら分からないようじゃ物理は無理

243 :ご冗談でしょう?名無しさん:2024/02/28(水) 20:51:27.21 ID:???.net
こんな事で議論があるのが不思議

244 :ご冗談でしょう?名無しさん:2024/02/28(水) 21:11:10.04 ID:???.net
>>241
テストで証明問題出たら全部それ書きや

245 :ご冗談でしょう?名無しさん:2024/02/28(水) 21:29:15.17 ID:???.net
>>222
R^n上を物体O_1,O_2がそれぞれ直線、曲線上を運動するとします
O_1,O_2はそれぞれ接線方向の加速度aを持ち、O_2は接戦と垂直方向に加速度を持つゆえにO_2の加速度ベクトルb↑が
b↑=ae↑+(|b↑|^2-a^2)f↑ (|e↑|=|f↑|=1,e↑·f↑=0)と表されるとします

時刻TにおけるO_1の速さv_1は
v_1=|∫[0,T](dv_1/dt)dt|=|∫[0,T]adt|

対してO_2の速さv_2は、
v_2=|∫[0,T](dv_2/dt)dt|
=|∫[0,T]b↑dt|
=|∫[0,T](ae↑+√(|b↑|^2-a^2)f↑)dt|
ここでe↑とf↑を基底とする平面P上でO_2の位置がr↑=(x,y)|_{x=y=0,y'=0}と表されるとすると、
dr↑/dt=(x',y')=(x',0)
|dr↑/dt|=√(x'^2+y'^2)=|x'|
d(dr↑/dt|)/dt=(x'x''+y'y'')/√(x'^2+y'^2)=x''=a
よってv_2=|∫[0,T]adt|


なんかこれ証明が間違ってる気がするよお🥺

246 :ご冗談でしょう?名無しさん:2024/02/28(水) 23:36:37.14 ID:???.net
曲線上の時間と直線上の時間って同じ?

247 :ご冗談でしょう?名無しさん:2024/02/28(水) 23:37:45.17 ID:???.net
曲線からは拘束力を受けるから時間経過は直線上のそれより遅れそうな気がする

248 :ご冗談でしょう?名無しさん:2024/02/29(木) 00:06:07.61 ID:???.net
一から説明すんのめんどくさいんで、同じとだけ言っとく

249 :ご冗談でしょう?名無しさん:2024/02/29(木) 00:21:26.30 ID:???.net
慣性系において、受ける力を無視すれば固有時に軌道が直線か曲線かは関係無いと思う
それよりは速さの方が関係しそう
ある慣性系において、時刻tのとき速さvで移動する質点における固有時は
c^2dτ^2=c^2dt^2-dr^2=c^2dt^2-v^2dt^2
dτ=dt√(1-v^2/c^2)

受ける力を考慮すれば>>247の言うとおり一般相対論的効果でおくれそうだけど、こっちは具体的な式で書くの相当ムズイでこりゃ

250 :ご冗談でしょう?名無しさん:2024/02/29(木) 00:29:50.93 ID:???.net
嬉々として調子に乗るシッタカ

251 :ご冗談でしょう?名無しさん:2024/02/29(木) 01:20:27.71 ID:???.net
簡単なことを遠回りして難しく見せるだけ

252 :ご冗談でしょう?名無しさん:2024/02/29(木) 01:55:14.10 ID:???.net
え?
何が間違ってるの!

253 :ご冗談でしょう?名無しさん:2024/02/29(木) 01:55:21.56 ID:???.net
どこ?

254 :ご冗談でしょう?名無しさん:2024/02/29(木) 02:11:01.39 ID:???.net
遠回しにしたつもり全く無いんやが!?

255 :ご冗談でしょう?名無しさん:2024/02/29(木) 05:27:54.56 ID:???.net
>>252
速さの方が関係しそうとか、一般相対論的効果で遅れそうとか勘で言わないように
おまえが自分で書いてる式によると、速さ以外なんにも関係ないはずだろ

256 :ご冗談でしょう?名無しさん:2024/02/29(木) 09:28:07.82 ID:???.net
>>255
そら慣性系である時刻における固有時を考えとるしな
君こそ前提無視するのやめてな

257 :ご冗談でしょう?名無しさん:2024/02/29(木) 11:17:18.55 ID:???.net
接線方向の力が与えられてたら相対論絡んでくるけど加速度が与えられてる問題から関係ないよな

258 :ご冗談でしょう?名無しさん:2024/02/29(木) 12:45:23.12 ID:???.net
v=da/dt
これが定義でこれで完結してるからな

259 :ご冗談でしょう?名無しさん:2024/02/29(木) 13:15:03.17 ID:???.net
固有時間なら世界線に依存するけど固有時はその全微分形やし

260 :ご冗談でしょう?名無しさん:2024/02/29(木) 13:35:33.11 ID:???.net
全微分ではないで

261 :ご冗談でしょう?名無しさん:2024/02/29(木) 14:30:54.46 ID:???.net
固有時間と固有時を区別するって何?

262 :ご冗談でしょう?名無しさん:2024/02/29(木) 16:59:30.62 ID:???.net
>>258
vとa逆じゃね?

263 :ご冗談でしょう?名無しさん:2024/02/29(木) 17:03:47.44 ID:23xKLvlI.net
v=∫adtが証明だろ

264 :ご冗談でしょう?名無しさん:2024/02/29(木) 17:05:51.18 ID:23xKLvlI.net
牛乳でつくったやつを、貴世石で、放射する、医療用デバイス放射機
上記 登記

265 :ご冗談でしょう?名無しさん:2024/02/29(木) 17:05:52.12 ID:23xKLvlI.net
牛乳でつくったやつを、貴世石で、放射する、医療用デバイス放射機
上記 登記

266 :ご冗談でしょう?名無しさん:2024/02/29(木) 17:33:56.42 ID:23xKLvlI.net
ポータブル量子コンピュータを
https://www.itmedia.co.jp/news/articles/2212/15/news139.html
大魔道石と賢者の石と賢者の石と賢者の石 8

すべて 上記  登記

267 :ご冗談でしょう?名無しさん:2024/02/29(木) 18:21:31.10 ID:???.net
>>261
「固有時『間』」やから固有時間は時刻の差なのはあったりめーよな
「固有時」は間ではないから各事象で定まる全微分形やね

っていうか固有時間で考えても積分作用素くっつけるだけや
ある事象一点における速さではなく一連の世界線における速さに依存するだけで、結局速さのみに依存しとるな

268 :ご冗談でしょう?名無しさん:2024/02/29(木) 20:27:58.29 ID:???.net
>>267
固有時間も固有時も言い方が違うだけで一緒やっちゅうねん
勝手に自分語作らんといてくれるか

269 :ご冗談でしょう?名無しさん:2024/02/29(木) 20:34:51.46 ID:???.net
>>256
で、おまえの前提って何よ?
力を受けると一般相対論効果で時間が遅れるとかいう、いい加減な結論に行き着いたおまえの前提って何?

270 :ご冗談でしょう?名無しさん:2024/02/29(木) 20:40:50.07 ID:???.net
>>269
重力
電磁気力

これらによる力なら一般相対論的効果で遅れるやん
シュバルツシルト系とかな
一般的な場合の証明ができるんかは知らんが
特殊相対論的な力ってあれ重力や電磁気力とは独立した仮想的な力やろうから厳密に考えたら存在せんわな

271 :ご冗談でしょう?名無しさん:2024/02/29(木) 20:46:39.89 ID:???.net
>>270
重力があるから一般相対論効果が見られるんやで
重力も何にもないところで、速さも変わってないのに力を受けたら時間が遅れるなんてそんな理屈があるかいな

272 :ご冗談でしょう?名無しさん:2024/02/29(木) 20:54:14.50 ID:???.net
平坦な時空で物体がどんな曲線的な動きをしてようと、時間の遅れは特殊相対論の計算どおり速さにしか依存しないってこと

273 :ご冗談でしょう?名無しさん:2024/02/29(木) 20:58:30.46 ID:???.net
平坦な時空っていう前提条件ってあったっけ?

274 :ご冗談でしょう?名無しさん:2024/02/29(木) 21:17:30.70 ID:???.net
平坦じゃない時空って前提条件あったっけ?
だからどうぞ後付けでいくらでも屁理屈こねてくださいと>>269で振ったんだが

275 :ご冗談でしょう?名無しさん:2024/02/29(木) 21:25:36.17 ID:???.net
>>271
>>249
慣性系においてって前提で語っとるで

276 :ご冗談でしょう?名無しさん:2024/02/29(木) 21:28:56.57 ID:???.net
>>275
違ったわwww


>>271
重力や電磁気力以外の力ってなんや?

277 :ご冗談でしょう?名無しさん:2024/02/29(木) 21:28:58.77 ID:???.net
ってことは平坦な時空前提ってことだわな
自分で気付いて偉い

278 :ご冗談でしょう?名無しさん:2024/02/29(木) 21:31:03.37 ID:???.net
どの力も結局は4つの基本相互作用に分解されるわけやろ?
そんで加速度を持つってことは力を受けるってこと

つまり物体が非一様な加速度を持つ系は曲率を持つってことやな!

279 :ご冗談でしょう?名無しさん:2024/02/29(木) 21:31:40.68 ID:???.net
曲がってる時点で慣性系ではない

280 :ご冗談でしょう?名無しさん:2024/02/29(木) 21:41:32.57 ID:???.net
リンドラー座標なら一応速さ以外にも位置にも依存するな!!

281 :ご冗談でしょう?名無しさん:2024/02/29(木) 21:43:27.15 ID:???.net
>>279
あたりまえだけど慣性系にいるのは、物体が曲線運動してると認識してる人、つまり観測者な

282 :ご冗談でしょう?名無しさん:2024/02/29(木) 21:44:51.07 ID:???.net
>>279
それな!!

283 :ご冗談でしょう?名無しさん:2024/02/29(木) 21:45:47.30 ID:???.net
>>280
どうでもいいけど自分で慣性系って言っちゃってるんやで

284 :ご冗談でしょう?名無しさん:2024/02/29(木) 21:47:36.93 ID:???.net
>>283
>>249で「受ける力を考慮すれば」としか書いてないで
慣性系に限定したのは受ける力を無視した場合のみな

285 :ご冗談でしょう?名無しさん:2024/02/29(木) 21:53:09.12 ID:???.net
>>284
わたしは慣性座標系を使いますって最初に宣言してんのよな?
見てる物体が力を受けてようと受けてまいと、決めた座標系が勝手に変わるなんてルールはないぞ

286 :ご冗談でしょう?名無しさん:2024/02/29(木) 22:03:13.00 ID:???.net
>>284
要するに「力を受けてるんだなー」って頭の中で考えただけで、
勝手に座標変換が行われて一般相対論効果で時間が遅れるというファンタジーな主張をしてんの?

287 :ご冗談でしょう?名無しさん:2024/02/29(木) 22:16:56.70 ID:???.net
>>285
力を考慮する場合に慣性系使うなんて言うとらんで
勘違いさせる書き方したのは悪いけど、そもそも「一般相対論的効果」って言ってる時点で慣性系じゃない場合を想定してることくらいわかるやろ?
君がいちいち意地悪な解釈して揚げ足取ろうとしとるだけやな

288 :ご冗談でしょう?名無しさん:2024/02/29(木) 22:57:52.68 ID:???.net
>>287
むしろ最大限に良心的解釈をあれこれ模索しとると思うんだが

一般相対論効果が出てくるなんてどんな座標系想定してんのか言ってみ?

座標系考える上で力を考慮するとかしないとか関係ないこと言い出す意味も
わからんし、
一般相対論効果出るなら、相手が直線運動してようが曲線運動してようが関係なく効果出るはずだからなにが言いたいかまったくわからんのよ

289 :ご冗談でしょう?名無しさん:2024/02/29(木) 23:15:52.39 ID:???.net
>>288
曲線運動するってことは力を受けるわけやろ?
そんで力を受けるってことはその系は厳密には非慣性系なんよ
ほなら一般相対論的効果が現れるやん

290 :ご冗談でしょう?名無しさん:2024/02/29(木) 23:19:02.79 ID:???.net
関係無いけど座標系って座標を張られた系のことやろ?
同じ系でも座標変換で色々な座標を張れるわけやし

291 :ご冗談でしょう?名無しさん:2024/02/29(木) 23:26:57.73 ID:???.net
>>288
非慣性系で直線運動は無理やないか?
測地線に沿った運動はできるが

292 :ご冗談でしょう?名無しさん:2024/02/29(木) 23:27:55.55 ID:???.net
>>289
物体視点の座標系で考えてんの?
ほなら、自分自身の速度は常に0に決まってるし、固有時間も物体自身の時計で測った物体にとっての時間やから遅れも進みもあるはずないわな
なんにも難しいことあらへん

293 :ご冗談でしょう?名無しさん:2024/02/29(木) 23:30:14.83 ID:???.net
>>291
自分で無理な設定ってわかってんなら、きみはいつたい何を語ってたんや?

294 :ご冗談でしょう?名無しさん:2024/02/29(木) 23:35:20.64 ID:???.net
>>292
物体視点に限っとらんが
何を思って物体視点やと思ったんや
>>293
ワイは非慣性系で直線運動する物体の場合なんて一度も述べとらんで
「慣性系で測地線を通る(直線運動する)物体」と「非慣性系で測地線を通る(曲線運動する)物体」の2つにしか言及しとらん
加速度を得るってのは曲がった時空の測地線を通過するってことやからな

295 :ご冗談でしょう?名無しさん:2024/02/29(木) 23:53:32.74 ID:???.net
>>292
あ、慣性系において曲線運動する物体視点の系は非慣性系やから、そういうことか?
まあワイはそもそも慣性系において曲線運動する物体への言及なんかしとらんが

296 :ご冗談でしょう?名無しさん:2024/02/29(木) 23:53:53.69 ID:???.net
>>294
> 物体視点に限っとらんが
> 何を思って物体視点やと思ったんや

> 289 ご冗談でしょう?名無しさん sage 02/29(木) 23:15:52.39
> 曲線運動するってことは力を受けるわけやろ?
> そんで力を受けるってことはその系は厳密には非慣性系なんよ

力を受けて曲線運動してるという物体の座標系に思いっきりスポットして語っとるやないか


> 「慣性系で測地線を通る(直線運動する)物体」と「非慣性系で測地線を通る(曲線運動する)物体」の2つにしか言及しとらん

無茶苦茶後付けで重力による測地線って設定出してきたけどOK
一時期電磁気力とか言ってたのはなんのつもりだったのかの

297 :ご冗談でしょう?名無しさん:2024/03/01(金) 00:02:02.07 ID:???.net
>>296
後出ししたつもりは無いんやがな

298 :ご冗談でしょう?名無しさん:2024/03/01(金) 00:08:12.94 ID:???.net
まとめると、>>249 はこう訂正しなきゃ意味は通らないって話な

× 慣性系において、受ける力を無視すれば
○ 慣性系において、重力以外の力を受けてると考えれば

× 受ける力を考慮すれば
○ 重力を受けてると考えれば

299 :ご冗談でしょう?名無しさん:2024/03/01(金) 00:21:30.31 ID:???.net
>>298
ありがとう

300 :ご冗談でしょう?名無しさん:2024/03/01(金) 00:22:30.90 ID:???.net
加速度あると慣性系でないって言う奴いるよね

301 :ご冗談でしょう?名無しさん:2024/03/01(金) 00:23:38.21 ID:???.net
うむ
>>249は設定の時点で問題がある

302 :ご冗談でしょう?名無しさん:2024/03/01(金) 00:23:42.67 ID:???.net
慣性系では力も加速度も扱えんらしい

303 :ご冗談でしょう?名無しさん:2024/03/01(金) 00:38:39.83 ID:???.net
ストローマン論法

304 :ご冗談でしょう?名無しさん:2024/03/01(金) 08:21:34.01 ID:???.net
でぼん

305 :ご冗談でしょう?名無しさん:2024/03/01(金) 11:24:27.05 ID:???.net
頭悪い奴ばっかだな
(観測者の)座標系が慣性系か非慣性系かの違いであって、物体の加速運動とは関係ない。
ローレンツ力の速度vは慣性系から観測した速度である。

古典物理では地球上は短時間短距離の実験室スケールで慣性系と見なせる
ローレンツ力も、振り子の様な加速度運動も力学計算できる。

306 :ご冗談でしょう?名無しさん:2024/03/01(金) 11:49:48.36 ID:???.net
非相対論に話を限定するべきで、速さvは速度v↑と加速度a↑を使って
v = |v↑| = |∫ a↑ dt|
と書けるが、これが直線上と曲線上で本当に同じと言えるのか?という本来の疑問の答えは結局どうなったんだ?

307 :ご冗談でしょう?名無しさん:2024/03/01(金) 12:25:59.35 ID:???.net
そんなの既に解決してる

308 :ご冗談でしょう?名無しさん:2024/03/01(金) 12:33:35.66 ID:???.net
例えば山手線に乗ると曲がりによる非慣性効果=重力効果で時間が遅れる

309 :ご冗談でしょう?名無しさん:2024/03/01(金) 12:40:49.82 ID:???.net
↑同じ話をしても理解できる奴と理解できないやつがいるという見本

310 :ご冗談でしょう?名無しさん:2024/03/01(金) 12:49:23.33 ID:???.net
>>307
どのレスで?

311 :ご冗談でしょう?名無しさん:2024/03/01(金) 12:54:22.21 ID:???.net
高々300レスを遡れないようでは物理は無理

312 :ご冗談でしょう?名無しさん:2024/03/01(金) 12:55:29.80 ID:???.net
v=∫adt

313 :ご冗談でしょう?名無しさん:2024/03/01(金) 13:14:07.07 ID:???.net
>>298
ローレンツ力を受けて重力を受けないなんて状況無いやろ

314 :ご冗談でしょう?名無しさん:2024/03/01(金) 13:14:50.01 ID:???.net
>>312
速度と垂直方向の加速度を考慮してるか?

315 :ご冗談でしょう?名無しさん:2024/03/01(金) 13:46:44.10 ID:???.net
曲線に沿った積分なんだろ

316 :ご冗談でしょう?名無しさん:2024/03/01(金) 14:43:45.00 ID:???.net
速度も加速度もベクトルだ

317 :ご冗談でしょう?名無しさん:2024/03/01(金) 14:49:42.89 ID:???.net
時間はパラメータということを考えれば自明

318 :ご冗談でしょう?名無しさん:2024/03/01(金) 15:01:16.73 ID:???.net
1つの系で見た速度・加速度の話のはずなのに時間が遅れるとか言ってる人は何の話をしてるんだろうね
時間の遅れ云々は別の系での時間と比較しなければ出て来ない話

319 :ご冗談でしょう?名無しさん:2024/03/01(金) 15:23:02.15 ID:???.net
接線方向の加速度だけで速さは一意に決まるのか?という疑問にはまだ回答がないようだ。

320 :ご冗談でしょう?名無しさん:2024/03/01(金) 16:05:36.50 ID:???.net
既に出てるが

321 :ご冗談でしょう?名無しさん:2024/03/01(金) 16:10:16.26 ID:???.net
だからどれだよ

322 :ご冗談でしょう?名無しさん:2024/03/01(金) 18:25:23.69 ID:???.net
a=dv/dtよりv=∫adtなのか

323 :ご冗談でしょう?名無しさん:2024/03/01(金) 18:36:30.22 ID:???.net
aもvもベクトルだから足し合わせるときに注意が必要

324 :ご冗談でしょう?名無しさん:2024/03/01(金) 18:36:35.06 ID:???.net
そやね

325 :ご冗談でしょう?名無しさん:2024/03/01(金) 18:38:49.96 ID:???.net
速度も加速度も徹頭徹尾接線方向しか考えない前提だから法線方向は関係ないね

326 :ご冗談でしょう?名無しさん:2024/03/01(金) 18:50:44.22 ID:???.net
弧長パラメータ表示で自明

327 :ご冗談でしょう?名無しさん:2024/03/01(金) 19:02:53.27 ID:???.net
式で証明して

328 :ご冗談でしょう?名無しさん:2024/03/01(金) 19:19:02.55 ID:???.net
v=∫adt

329 :ご冗談でしょう?名無しさん:2024/03/01(金) 19:20:51.57 ID:???.net
同じ関数を同じ変数で同じ範囲で積分したら同じ結果になることも納得できないなら付ける薬はない

330 :ご冗談でしょう?名無しさん:2024/03/01(金) 19:25:33.59 ID:???.net
積分とは結局のところ足し算

331 :ご冗談でしょう?名無しさん:2024/03/01(金) 19:42:17.17 ID:???.net
直線は曲率0の曲線でしかない

332 :ご冗談でしょう?名無しさん:2024/03/01(金) 19:49:30.57 ID:???.net
>>318
固有時間は系に依らんが

333 :ご冗談でしょう?名無しさん:2024/03/01(金) 19:51:08.12 ID:???.net
>>298
相対論的ローレンツ力を受けるってことは重力も受けるってことやで

334 :ご冗談でしょう?名無しさん:2024/03/01(金) 19:57:39.86 ID:???.net
|v| = |v↑| = ∫ d|v↑| = { ∫ d ( v↑・v↑) } ^(1/2) = {∫ (1/2)・2v↑・a↑ dt } ^ (1/2)

ここで条件よりv↑とa↑は常に同じ方向だから内積は

335 :ご冗談でしょう?名無しさん:2024/03/01(金) 20:58:58.88 ID:???.net
それぞれのベクトルの大きさの積と等しいため、直線上の運動の場合と同一となる。

336 :ご冗談でしょう?名無しさん:2024/03/01(金) 21:04:34.23 ID:???.net
ってくらい書けばOKだけど、v=∫adtで証明になっていると思ってるやつはちょっと程度が低すぎるね

337 :ご冗談でしょう?名無しさん:2024/03/01(金) 21:05:54.80 ID:???.net
^(1/2)の場所間違ったけど許してね

338 :ご冗談でしょう?名無しさん:2024/03/01(金) 21:07:19.83 ID:???.net
>>242
馬鹿のくせに偉そうだな

339 :ご冗談でしょう?名無しさん:2024/03/01(金) 21:59:52.48 ID:???.net
>>336
それ程度が低い奴のセリフ

340 :ご冗談でしょう?名無しさん:2024/03/01(金) 22:12:36.62 ID:???.net
負け惜しみは教科書どおり

341 :ご冗談でしょう?名無しさん:2024/03/01(金) 22:14:48.29 ID:???.net
結局積分が何やってるかを分かってないから自明と分からないんだよ

342 :ご冗談でしょう?名無しさん:2024/03/01(金) 22:20:35.47 ID:???.net
おまえの教科書には証明問題はすべて自明と回答せよと書いてあるのか

343 :ご冗談でしょう?名無しさん:2024/03/01(金) 22:21:13.22 ID:???.net
>>341
分かってないのはお前

344 :ご冗談でしょう?名無しさん:2024/03/01(金) 22:22:09.21 ID:???.net
>>326
弧長パラメータ表示したところで速度と垂直方向の加速度はゼロにはならん

345 :ご冗談でしょう?名無しさん:2024/03/01(金) 22:50:15.92 ID:???.net
接線の話だろアホ

346 :ご冗談でしょう?名無しさん:2024/03/01(金) 22:51:51.81 ID:???.net
>>345
速度は接線方向と平行だぞバカアホドジ間抜けおたんこなす

347 :ご冗談でしょう?名無しさん:2024/03/01(金) 22:54:20.85 ID:???.net
うわあ程度が高い議論が展開されていますね

348 :ご冗談でしょう?名無しさん:2024/03/01(金) 22:56:47.34 ID:???.net
>>346
だから接線の話だよねアホ

349 :ご冗談でしょう?名無しさん:2024/03/01(金) 22:57:39.85 ID:???.net
弧長パラメータ表示する意味すら分かってないなら自明と気付けないのも仕方ない

350 :ご冗談でしょう?名無しさん:2024/03/01(金) 23:02:03.20 ID:???.net
>>349
バーカ

351 :ご冗談でしょう?名無しさん:2024/03/01(金) 23:02:55.63 ID:???.net
質問者「どう証明すればいいですか?」

バカ「自明」


いや何の意味も無い回答だなw

352 :ご冗談でしょう?名無しさん:2024/03/01(金) 23:02:55.89 ID:???.net
何故馬鹿に付き合って自らのレベルを落とすのか

353 :ご冗談でしょう?名無しさん:2024/03/01(金) 23:03:15.09 ID:???.net
>>349
頭わりーくせにプライド高いどうしようもないやち

354 :ご冗談でしょう?名無しさん:2024/03/01(金) 23:03:47.33 ID:???.net
こいつ弧長パラメータ表示の定義すら知らなそうw

355 :ご冗談でしょう?名無しさん:2024/03/01(金) 23:05:10.53 ID:???.net
発狂して連投w

356 :ご冗談でしょう?名無しさん:2024/03/01(金) 23:08:45.04 ID:???.net
まじでただのv=∫adtなんだよな
低能をここまで拗らせるとこれすら分からない

357 :ご冗談でしょう?名無しさん:2024/03/01(金) 23:22:14.31 ID:???.net
高校レベルw

358 :ご冗談でしょう?名無しさん:2024/03/01(金) 23:30:09.46 ID:???.net
>>356
あーほ

359 :ご冗談でしょう?名無しさん:2024/03/01(金) 23:31:31.39 ID:???.net
>>356
v=∫dt dv/dtと書いてあげないと分からないかもしれない

360 :ご冗談でしょう?名無しさん:2024/03/01(金) 23:33:17.74 ID:???.net
バカ同士のじゃれ合いでスレ埋めんなな見苦しい

361 :ご冗談でしょう?名無しさん:2024/03/01(金) 23:33:26.66 ID:???.net
向心力とか無い世界から来たんかな

362 :ご冗談でしょう?名無しさん:2024/03/01(金) 23:49:18.49 ID:???.net
接線の話というのが理解できないらしい

363 :ご冗談でしょう?名無しさん:2024/03/02(土) 00:12:19.89 ID:???.net
>>362
それが証明?
完全に0点

364 :ご冗談でしょう?名無しさん:2024/03/02(土) 00:16:46.68 ID:???.net
向心力とか頓珍漢なこという採点者は論外

365 :ご冗談でしょう?名無しさん:2024/03/02(土) 00:18:57.58 ID:???.net
おまえの答えは誰が採点しても0点以外ないわな

366 :ご冗談でしょう?名無しさん:2024/03/02(土) 00:20:48.90 ID:???.net
さすがに大学の教授クラスの人ならこの程度の積分は自明だw

367 :ご冗談でしょう?名無しさん:2024/03/02(土) 00:24:41.83 ID:???.net
積分をするんじゃなくて証明をするんやで
証明って義務教育でもやらんかったっけ?

368 :ご冗談でしょう?名無しさん:2024/03/02(土) 00:25:40.59 ID:???.net
https://engineer.fabcross.jp/archeive/240229_reverse-sprinkler.html

ファインマンの逆スプリンクラー問題が解けたみたいだけど、レイノルズ数を変化させたらどうなるんだろうか?

369 :ご冗談でしょう?名無しさん:2024/03/02(土) 01:08:32.81 ID:B8klYcRi.net
加速度ベクトルaの絶対値をkと置いて、それを成分で表記して、それを速度ベクトルvの微分に表記して、そこから積分したとき、速度ベクトルの絶対値が等しいかを確かめる。

俺は計算忘れてすぐできないから、誰か示したら。

370 :ご冗談でしょう?名無しさん:2024/03/02(土) 08:01:45.41 ID:???.net
弧長パラメータ表示しろよ

371 :ご冗談でしょう?名無しさん:2024/03/02(土) 08:12:04.09 ID:???.net
まじでこの程度の計算すら自明レベルに分からない人いるのか

372 :ご冗談でしょう?名無しさん:2024/03/02(土) 08:14:05.08 ID:???.net
速度ベクトルは接ベクトル方向ということが分かってないんだろうな
だから積分ができない

373 :ご冗談でしょう?名無しさん:2024/03/02(土) 10:30:10.98 ID:???.net
物理学について質問です。
物理学とは、みんなでひたすら「私はおまえより理解してる」と主張し続けるだけの競技かなんかなんですか?

374 :ご冗談でしょう?名無しさん:2024/03/02(土) 10:39:30.33 ID:???.net
なんか実体はよくわからんけど「悟りを開いた」と言い張る仏陀みたいなもん

375 :poem:2024/03/02(土) 11:14:36.48 ID:YT3nSYcD.net
でも、皆遊びたいから物理板に来てる

376 :ご冗談でしょう?名無しさん:2024/03/02(土) 12:23:31.97 ID:???.net
>>375
深いな

377 :ご冗談でしょう?名無しさん:2024/03/02(土) 12:24:31.23 ID:???.net
自明ってことは当然ながら公理から導くこともやらないだけでできるんだろうね

378 :ご冗談でしょう?名無しさん:2024/03/02(土) 12:30:10.24 ID:???.net
物理の問題解くのに実数の構成から始める馬鹿

379 :ご冗談でしょう?名無しさん:2024/03/02(土) 12:44:31.15 ID:???.net
任意の滑らかな経路で接線方向の速度を時間微分したものが接線方向の加速度
従って接線方向の加速度を時間で積分すれば接線方向の速度となる
これが経路に関わらず成り立つから直線経路でも成り立つ。
これでも未だ分からん?

380 :ご冗談でしょう?名無しさん:2024/03/02(土) 12:56:43.68 ID:???.net
わかるとかわからんの話じゃなくて、証明なんだからちゃんと数学の形式に則ってなきゃ意味ないだろ
>>222はこれでどう?

加速度ベクトル a> を速度の接線方向 a_t と 法線方向 a_n に分解する
微小時間 dt 秒後の速さの2乗は次のように表される
 (v + dv)^2 = (v + a_t dt)^2 + (a_n dt)^2
式の両辺を展開
 v^2 + 2vdv + (dv)^2 = v^2 + 2va_t dt + (a_t ^2 + a_n ^2) (dt)^2
ここで (dv)^2 と (dt)^2 項は無視できるため
 dv = a_t dt
よって
 v =∫a dt

つまり速さvは、初速度と速度接線方向の加速度のみで決まる

381 :ご冗談でしょう?名無しさん:2024/03/02(土) 12:58:23.75 ID:???.net
> v =∫a dt

ごめん最後
v =∫a_t dt

382 :ご冗談でしょう?名無しさん:2024/03/02(土) 13:16:54.20 ID:???.net
>>379
「速さ」って速度の絶対値のことですよ

383 :ご冗談でしょう?名無しさん:2024/03/02(土) 13:34:07.24 ID:???.net
>>378
できないんだ?

384 :ご冗談でしょう?名無しさん:2024/03/02(土) 14:26:37.24 ID:B8klYcRi.net
そもそも速度ベクトルは位置の変位ベクトルを時間で割ったもの。加速度ベクトルは速度ベクトルの変位ベクトルを時間で割ったもの。

質量同じ初速度同じで、加速度ベクトルが同じなら、同じ経路でないとおかしい。設問は、だから加速度ベクトルの大きさが同じならとの解釈しか意味がない。

変位ベクトルの概念がわからないと、ベクトルの微積分は理解不能と思うな。物理の前段階だけど。

385 :ご冗談でしょう?名無しさん:2024/03/02(土) 15:27:50.35 ID:B8klYcRi.net
位置を表す座標と、スカラー値関数を表す座標、そこにベクトル値関数を表す座標、とかややこしいけど。

ベクトル関数を座標で表すと、変数分の軸が足りないから、コンピュータとかで時間変化するベクトルを動画で出せば視覚的でわかりやすいだろうけど。

386 :ご冗談でしょう?名無しさん:2024/03/02(土) 16:07:08.60 ID:???.net
アホは黙ってろ

387 :ご冗談でしょう?名無しさん:2024/03/02(土) 16:09:12.38 ID:B8klYcRi.net
ベクトルの微分とは何か。それは変位ベクトルの極限を取ること。

ただ速度ベクトルは位置座標上の変位を取るのに対して、加速度ベクトルはベクトル関数の変位を取る。そこが混乱しやすいんだと思う。

位置座標上で加速度ベクトルを求めるとか難しいから。速度ベクトルをベクトル関数として座標表記して、その変位から加速度ベクトルを求めないと視覚的な理解にはさ。

それを数学的に確かめるには、成分でみるしかない。ベクトル解析の勉強だけど。

388 :ご冗談でしょう?名無しさん:2024/03/02(土) 16:15:53.96 ID:???.net
けどけどうるさいわ
誰に対して何のいいわけをしてんだよ

389 :ご冗談でしょう?名無しさん:2024/03/02(土) 16:31:28.99 ID:B8klYcRi.net
速度ベクトルは位置の変位だから、位置座標の二点を狭めるだけで、極限取れる。これは関数座標でのイメージと同じ。

加速度ベクトルは速度ベクトルがどう変化したかを見ないといけないから、平行四辺形の法則になるように始点終点合わせないと、視覚的に変化がわからない。

だから、加速度ベクトルを位置座標上で考えるのはよくない。始点終点合わせることができないから。

390 :ご冗談でしょう?名無しさん:2024/03/02(土) 16:54:42.77 ID:???.net
>>382
速度ベクトルは接ベクトル方向なの分からんの?
加速度の接線方向は速度の大きさの微分なんだがw

391 :ご冗談でしょう?名無しさん:2024/03/02(土) 17:23:58.79 ID:???.net
こいつ何いってんの?
例えば曲率なんかは位置空間内で速度と加速度を使って何の問題もなく定義できるし
ベクトルの加算で始点を一致させないと計算できないなんていう制限もどこにも無い。

392 :ご冗談でしょう?名無しさん:2024/03/02(土) 17:35:01.31 ID:???.net
ID:B8klYcRi のことなら、本人がわからないと言ってるだけだから放っときゃいいんと違いますの
袋に入ってるゴミはわざわざ踏みにいかんでよい

393 :ご冗談でしょう?名無しさん:2024/03/02(土) 17:38:56.64 ID:???.net
でぼん

394 :ご冗談でしょう?名無しさん:2024/03/02(土) 17:46:33.79 ID:???.net
漂う荒し臭

395 :ご冗談でしょう?名無しさん:2024/03/02(土) 18:38:47.94 ID:???.net
ふだんコテハンで書き込んでる荒らしが、場合によって匿名で書き込んでるのでしょうね。

396 :poem:2024/03/02(土) 19:30:14.20 ID:YT3nSYcD.net
名前欄1年以上抜かしたことないよ

397 :poem:2024/03/02(土) 19:32:57.30 ID:YT3nSYcD.net
マジで1年以上poemP○ΘM書き落としがなければ 皆勤賞取れる

398 :poem:2024/03/02(土) 19:34:24.73 ID:YT3nSYcD.net
難しい計算自分できないから
難しい計算は自分じゃないの確実だからね

399 :poem:2024/03/02(土) 19:36:10.17 ID:YT3nSYcD.net
難しい学術雑談は
入れないんだYO

400 :poem:2024/03/02(土) 19:42:47.29 ID:YT3nSYcD.net
速さが
|v|
速度が
v
または
速さが
|v→|
速度が
v→
(書き方一切知らない)
かな?

で、
"加速さ"と"加速度"ね

401 :poem:2024/03/02(土) 19:43:58.05 ID:YT3nSYcD.net
かばやさ?
かぱやさ?

ば?ぱ?

どっちが読みやすい?

402 :poem:2024/03/02(土) 19:47:03.29 ID:YT3nSYcD.net
重箱読みじゃないなら

くわぱやさ
くわばやさ

403 :ご冗談でしょう?名無しさん:2024/03/02(土) 19:49:59.75 ID:???.net
>>390
じゃあそれを証明してくださいね

404 :ご冗談でしょう?名無しさん:2024/03/02(土) 19:51:47.87 ID:???.net
>>380
物理的にはよさそうだけど数学的にはよくなさそう

405 :ご冗談でしょう?名無しさん:2024/03/02(土) 19:52:01.35 ID:???.net
速度ベクトルが曲線の接線方向ってことも知らない馬鹿だったというオチ

406 :ご冗談でしょう?名無しさん:2024/03/02(土) 19:53:56.21 ID:???.net
そのレベルなら物理は無理だろw

407 :ご冗談でしょう?名無しさん:2024/03/02(土) 19:56:56.19 ID:???.net
微分して接線求めるって高校2年の範囲だろ…

408 :ご冗談でしょう?名無しさん:2024/03/02(土) 19:57:05.27 ID:???.net
>>404
よくなさそうとか自分の感覚でしゃべるのは数学的によいの?

409 :ご冗談でしょう?名無しさん:2024/03/02(土) 21:31:12.94 ID:???.net
>>408
感覚っていうか無限小で代数的操作をするのがまずそうってことよ
まあ超準解析とかだと良いらしいけど、ソノヘンハエアプだからよくわからん

410 :ご冗談でしょう?名無しさん:2024/03/02(土) 21:32:25.54 ID:???.net
>>405
そっちじゃなくて「加速度の接線方向は速度の大きさの微分なんだがw」を証明してくださいね
逃げないでくださいね

411 :ご冗談でしょう?名無しさん:2024/03/02(土) 21:40:09.69 ID:???.net
速度ベクトルが接ベクトル方向って分かってたらそこは自明だろw
加速度の定義は速度の時間微分だぞ?w

412 :ご冗談でしょう?名無しさん:2024/03/02(土) 21:44:55.99 ID:???.net
速度ベクトルが接ベクトル方向のときの加速度の接線方向が分からないってまじ?

413 :ご冗談でしょう?名無しさん:2024/03/02(土) 21:50:53.98 ID:???.net
>>411
「加速度は速度の時間微分」ってことが定義だとしても、「加速度ベクトルのうち速度ベクトルと平行な成分の大きさは速度ベクトルの大きさの時間微分と等しい」なんてのは定義じゃないと思いますが

414 :ご冗談でしょう?名無しさん:2024/03/02(土) 21:52:56.98 ID:???.net
御託はいいんで証明してくださいね

415 :ご冗談でしょう?名無しさん:2024/03/02(土) 21:53:21.80 ID:???.net
>>409
よくわからんと言いながら言っちやうことが感覚的発言なんやで

416 :ご冗談でしょう?名無しさん:2024/03/02(土) 21:57:30.32 ID:???.net
>>413
定義ではなくても自明だろ
vは接線方向しかないんだからw

417 :ご冗談でしょう?名無しさん:2024/03/02(土) 21:59:12.41 ID:???.net
a=dv/dt
速度ベクトルは接ベクトル方向
ここから接戦方向の加速度が分からないってどうなん?

418 :ご冗談でしょう?名無しさん:2024/03/02(土) 22:02:16.50 ID:???.net
>>416
自明自明って言葉で逃げるにしてももうちょっと言い方があるよなあ
例えば「質点がある軌道を描くとき、その軌道と垂直な方向に加速度を持っていても質点は軌道上から外れることは無いのでこの加速度は速さには寄与しない」とかさ

419 :ご冗談でしょう?名無しさん:2024/03/02(土) 22:03:24.28 ID:???.net
何もわかってないんじゃないくせに偉そうにしてるようにしか見えんよね

420 :ご冗談でしょう?名無しさん:2024/03/02(土) 22:04:27.90 ID:???.net
>>417
速度ベクトルが接線方向でも、その時間微分が接線方向だとは限らないですよ

421 :ご冗談でしょう?名無しさん:2024/03/02(土) 22:05:59.94 ID:???.net
a=dv/dt
速度ベクトルは接ベクトル方向

このときの加速度の接線方向は速度ベクトルの大きさの時間微分って分からんのまじ?
速度ベクトルが接ベクトル方向ってことは速度ベクトルの接線成分は速度ベクトルの大きさって分からんのまじ?

422 :ご冗談でしょう?名無しさん:2024/03/02(土) 22:06:56.49 ID:???.net
>>420
今考えてるのは加速度の接線成分なんですが…

423 :ご冗談でしょう?名無しさん:2024/03/02(土) 22:10:05.52 ID:???.net
速度ベクトルが接ベクトル方向のとき
v↑=│v↑│e_t
これが自明じゃないってまじ?w

424 :ご冗談でしょう?名無しさん:2024/03/02(土) 22:12:37.01 ID:???.net
ベクトルの向きが単位ベクトル方向ならその成分はベクトルの大きさである
ここから説明が必要だった?

425 :ご冗談でしょう?名無しさん:2024/03/02(土) 22:12:47.42 ID:???.net
dt後の方向は違うだろ

426 :ご冗談でしょう?名無しさん:2024/03/02(土) 22:14:57.44 ID:???.net
あのなそもそも加速度の接線方向の話なんだわ…
問題に与えられてるのは接線成分なのわかる?

427 :ご冗談でしょう?名無しさん:2024/03/02(土) 22:15:52.86 ID:???.net
>>423
見れば見るほど自明だな

428 :ご冗談でしょう?名無しさん:2024/03/02(土) 22:15:54.65 ID:???.net
>>421
分からんなあ
直感的にはそうだとしても数学的に証明してくれないとなあ

429 :ご冗談でしょう?名無しさん:2024/03/02(土) 22:17:03.55 ID:???.net
証明ってどっかに書かれてただろ
いつまでやっとんねん

430 :ご冗談でしょう?名無しさん:2024/03/02(土) 22:17:50.32 ID:???.net
>>428
ベクトルvの向きがe方向のとき
v=│v│eと書けるんだわ…
証明は自分との内積とるだけなんだわ…
まじでここから分からなかったのか…

431 :ご冗談でしょう?名無しさん:2024/03/02(土) 22:19:00.52 ID:???.net
他人がわかろうがわかるまいがどうでもいいだろ
いつまでやっとんねん

432 :ご冗談でしょう?名無しさん:2024/03/02(土) 22:19:32.49 ID:???.net
>>430
途中までしか書いてないじゃん
質問者が理解できるまで書いてね

433 :ご冗談でしょう?名無しさん:2024/03/02(土) 22:20:11.54 ID:???.net
このレベルの数学すら把握してないって物理やる以前に数学復習したほうがいいだろ…
ベクトルは物理の基本だろ…

434 :ご冗談でしょう?名無しさん:2024/03/02(土) 22:21:21.74 ID:???.net
>>432
内積計算すら他人任せじゃ物理の勉強は無理だね…
そうやって全部他人任せにしてテストを乗り越えるがいいさ

435 :ご冗談でしょう?名無しさん:2024/03/02(土) 22:22:05.30 ID:???.net
>>433
まーた「自明」で逃げるのか?
例えば速度と加速度が互いに逆方向のときどうすんの?
速度の大きさの時間微分が加速度の接線方向にならないけど?
ちゃんと証明してくださいよ

436 :ご冗談でしょう?名無しさん:2024/03/02(土) 22:22:08.65 ID:???.net
このレベルだと高校数学からやり直すしかないがそれすらやらないからこんなことになってるんだろうな

437 :ご冗談でしょう?名無しさん:2024/03/02(土) 22:22:41.56 ID:???.net
数学も物理もわかるやつはわかるしわからんやつはわからん
そんなこといつまで言い合ってても無駄

438 :ご冗談でしょう?名無しさん:2024/03/02(土) 22:24:11.94 ID:???.net
>>435
なるんだわ…
まじでこのレベルかよ…

439 :ご冗談でしょう?名無しさん:2024/03/02(土) 22:26:32.75 ID:???.net
そのレベルそのレベル
同じこと何回確認しとんねん

440 :ご冗談でしょう?名無しさん:2024/03/02(土) 22:31:17.17 ID:???.net
>>438
時刻tのとき点PがR上でr=t^2の位置にいるとします
dr/dt=2t
d^2r/dt^2=2

t<0のとき
|dr/dt|=|2t|=-2t
d|dr/dt|/dt=-2

d^2r/dt^2=2

ならないじゃん?

441 :ご冗談でしょう?名無しさん:2024/03/02(土) 22:35:08.70 ID:???.net
vがe方向とはv=keとなるk>0があるということです
eは単位ベクトルなのでe・e=1です
v・v=k^2e・e=k^2です
k=√v・vです
これはvの大きさなので証明完了です

このレベルから説明いるってまじ?w
このレベルならここで質問する前に教科書読み直すレベルだろw

442 :ご冗談でしょう?名無しさん:2024/03/02(土) 22:37:04.43 ID:???.net
>>440
あのー接ベクトルの向き考えて下さい…

443 :ご冗談でしょう?名無しさん:2024/03/02(土) 22:37:45.51 ID:???.net
「速度ベクトルの大きさ」じゃなくて「曲線上大域的な接続を持つ各点での接線内において正規基底表示される速度ベクトルの座標」ってとこですかねー?

444 :ご冗談でしょう?名無しさん:2024/03/02(土) 22:39:57.22 ID:???.net
嘘ついちゃいかんですよ

445 :ご冗談でしょう?名無しさん:2024/03/02(土) 22:40:59.35 ID:???.net
>>442
ほんとな
向き考えてほしいわ

446 :ご冗談でしょう?名無しさん:2024/03/02(土) 22:42:21.41 ID:???.net
>>440
まじでこのレベルかよ…
t^2なんだからよく知ってるただの放物運動考えりゃいいだろ
ちゃんと対応してることも分かるだろ

447 :ご冗談でしょう?名無しさん:2024/03/02(土) 22:43:12.13 ID:???.net
>>446
自分は嘘をついてしまったと認めたほうがいいですよ

448 :ご冗談でしょう?名無しさん:2024/03/02(土) 22:45:47.44 ID:???.net
t=0で単位接ベクトルが変わることを理解していないから間違えてしまったようだ
まじでこのレベルだと物理は無理だろ…

449 :ご冗談でしょう?名無しさん:2024/03/02(土) 22:46:50.34 ID:???.net
放物線は中央軸で対称な形してるだろw

450 :ご冗談でしょう?名無しさん:2024/03/02(土) 22:49:47.16 ID:???.net
「速度ベクトルの大きさ」じゃなくて「曲線上大域的な接続を持つ各点での接線内において正規基底表示される速度ベクトルの座標」ってとこですかねー?

451 :ご冗談でしょう?名無しさん:2024/03/02(土) 22:50:39.07 ID:???.net
>>441
このレベルからやり直す程度の数学レベルじゃ物理は無理
ランダウも物理を見失わないように最低限の数学は身につけておくようにと言っていたようだ

452 :ご冗談でしょう?名無しさん:2024/03/02(土) 22:51:40.89 ID:???.net
単一光子の電場と磁場は記述できますか?

453 :ご冗談でしょう?名無しさん:2024/03/02(土) 22:52:12.67 ID:???.net
>>451
で?
向きどうすんの

454 :ご冗談でしょう?名無しさん:2024/03/02(土) 22:53:57.74 ID:???.net
このレベルまで巻き戻って
a=dv/dt
v=│v│e_t
ここまで巻き戻れば猿でも分かるだろ…

455 :ご冗談でしょう?名無しさん:2024/03/02(土) 22:54:20.08 ID:???.net
>>454
で?向きどうすんの

456 :ご冗談でしょう?名無しさん:2024/03/02(土) 22:55:14.65 ID:???.net
「自明」とか言ってるからこうなる

457 :ご冗談でしょう?名無しさん:2024/03/02(土) 22:55:48.31 ID:???.net
>>455
ああ、e_tは単位接ベクトルと書かないと分からないレベルだった?w
幼稚園児の相手してるんじゃねぇんだぞw

458 :ご冗談でしょう?名無しさん:2024/03/02(土) 22:57:53.62 ID:???.net
>>457
「状況次第で向き変えまーす」ってか?w
おもろw

459 :ご冗談でしょう?名無しさん:2024/03/02(土) 22:57:59.18 ID:???.net
vがe方向のときの成分がその大きさであることの証明は441で書いてるからよw
ガチで高校以下で草

460 :ご冗談でしょう?名無しさん:2024/03/02(土) 22:59:21.75 ID:???.net
>>458
単位接ベクトルの向きが変わらないってどんな思考回路だよ…
まじで物理する土台に立てないレベルの数学力だな

461 :ご冗談でしょう?名無しさん:2024/03/02(土) 23:00:16.78 ID:???.net
>>460
「状況次第で向き変えまーす」なんてしなくても接続を導入すれば一意的に向きが定まるんだけどなー

462 :ご冗談でしょう?名無しさん:2024/03/02(土) 23:00:21.11 ID:???.net
今の高校だとデカルト座標しかやらないのか?
単位ベクトルはex、eyで固定しか知らないのか?

463 :ご冗談でしょう?名無しさん:2024/03/02(土) 23:01:19.41 ID:???.net
単位接ベクトルの向きが一意に決まらない…?

464 :ご冗談でしょう?名無しさん:2024/03/02(土) 23:01:55.64 ID:???.net
そんで状況次第で変えるでもいいけど、どういう条件でどう帰るかは明確に述べておいてね

465 :ご冗談でしょう?名無しさん:2024/03/02(土) 23:02:16.70 ID:???.net
>>463
2方向ありますけど…)

466 :ご冗談でしょう?名無しさん:2024/03/02(土) 23:03:11.53 ID:???.net
>>465
単位接ベクトルは一意に決まりますけど…w

467 :ご冗談でしょう?名無しさん:2024/03/02(土) 23:03:57.44 ID:???.net
接ベクトルは時間の関数だから時間で微分をすれば法線方向のベクトルになるんじゃね?

468 :ご冗談でしょう?名無しさん:2024/03/02(土) 23:04:01.22 ID:???.net
dr/dsが一意に決まらないらしい…w

469 :ご冗談でしょう?名無しさん:2024/03/02(土) 23:04:35.15 ID:???.net
>>466
じゃあその決め方を書いてねー
また「自明」かな?

470 :ご冗談でしょう?名無しさん:2024/03/02(土) 23:05:04.46 ID:???.net
>>467
そもそもベクトル表示すら分かってなかったのにその微分なんか分かるわけもなかったってことだ…w

471 :ご冗談でしょう?名無しさん:2024/03/02(土) 23:05:17.63 ID:???.net
>>468
誰もそんなこと書いてないですよーw

472 :ご冗談でしょう?名無しさん:2024/03/02(土) 23:05:27.94 ID:???.net
>>469
上に書いてますよ…w

473 :ご冗談でしょう?名無しさん:2024/03/02(土) 23:05:52.14 ID:???.net
>>472
レス番は?

474 :ご冗談でしょう?名無しさん:2024/03/02(土) 23:06:24.53 ID:???.net
上のレスが見えないらしいw

475 :ご冗談でしょう?名無しさん:2024/03/02(土) 23:06:55.90 ID:???.net
>>474
逃げちゃったかー……
接続とかも知らないんだろうなー……

476 :ご冗談でしょう?名無しさん:2024/03/02(土) 23:07:52.93 ID:???.net
あっdr/dsの大きさが1になることも知らないのか…w
そりゃそうだよなベクトルの大きさが成分であることも判ってなかったもんなw

477 :ご冗談でしょう?名無しさん:2024/03/02(土) 23:09:01.13 ID:???.net
上のレスを見ないふりw

478 :ご冗談でしょう?名無しさん:2024/03/02(土) 23:09:56.65 ID:???.net
いろいろ書き方はあるだろうけど
>>334
でいいんじゃないの

479 :ご冗談でしょう?名無しさん:2024/03/02(土) 23:10:06.38 ID:???.net
v=│v│e_tも知りません
│dr/ds│=1も知りません
やばすぎ…

480 :ご冗談でしょう?名無しさん:2024/03/02(土) 23:11:50.55 ID:???.net
v=dr/dtだからdr/dsと同じ向きであることは自明
あっこれも自明って言っちゃ駄目なんだっけ…w

481 :ご冗談でしょう?名無しさん:2024/03/02(土) 23:17:57.06 ID:???.net
ベクトルとスカラーは書き分けましょう

482 :ご冗談でしょう?名無しさん:2024/03/02(土) 23:17:57.15 ID:???.net
猿だと分からないことが判明してしまってけど>>454から求めるのがいいだろう
あとは接線成分を積分するだけで解答になる

483 :ご冗談でしょう?名無しさん:2024/03/02(土) 23:20:50.79 ID:???.net
うむ
代入して積分して終わりだ

484 :ご冗談でしょう?名無しさん:2024/03/02(土) 23:22:17.34 ID:???.net
>>476
急に弧長持ち出してきてどうしたん?

485 :ご冗談でしょう?名無しさん:2024/03/02(土) 23:23:15.15 ID:???.net
自明君、相手がベクトルも理解してない馬鹿だと思ってたら足元をすくわれちゃったね

486 :ご冗談でしょう?名無しさん:2024/03/02(土) 23:25:41.19 ID:???.net
>>484
弧長パラメータ表示なんて昨日の段階で既に言及してるがw
単位接ベクトルの求め方分かってよかったねw

487 :ご冗談でしょう?名無しさん:2024/03/02(土) 23:26:38.54 ID:???.net
ベクトルの微積分が出来ない馬鹿かと思ってたら単位ベクトルのレベルから分かってない馬鹿でずっこけたよ

488 :ご冗談でしょう?名無しさん:2024/03/02(土) 23:27:00.05 ID:???.net
・加速度の接線方向は速度の大きさの微分なんだがw
・単位接ベクトルは一意に決まりますけど…w

自明君、これらを同時に満足する解決策ってなに?

489 :ご冗談でしょう?名無しさん:2024/03/02(土) 23:28:07.43 ID:???.net
>>486
なんでdr/ds=1を知らないと思ったの?

490 :ご冗談でしょう?名無しさん:2024/03/02(土) 23:28:59.13 ID:???.net
>>488
これ求む

491 :ご冗談でしょう?名無しさん:2024/03/02(土) 23:30:49.02 ID:???.net
>>441
このレベルの話をする必要があるやつ相手にしてるからさw

492 :ご冗談でしょう?名無しさん:2024/03/02(土) 23:31:39.51 ID:???.net
同時に満足しない例ってなに…w

493 :ご冗談でしょう?名無しさん:2024/03/02(土) 23:32:02.69 ID:???.net
>>492
はいどうぞ
>>440

494 :ご冗談でしょう?名無しさん:2024/03/02(土) 23:32:36.88 ID:???.net
>>493
同時に満足してて草

495 :ご冗談でしょう?名無しさん:2024/03/02(土) 23:39:29.27 ID:???.net
>>494
ベクトルの向き逆ですよ

496 :ご冗談でしょう?名無しさん:2024/03/02(土) 23:40:27.72 ID:???.net
単位接ベクトルe_t=dr/ds
速度ベクトルv=│v│e_t
加速度ベクトルa=dv/dt
全部決まるやないかーいw
ここまで書かないと猿には分からなかったらしい

497 :ご冗談でしょう?名無しさん:2024/03/02(土) 23:41:10.36 ID:???.net
>>496
それだと向き逆ですよw

498 :ご冗談でしょう?名無しさん:2024/03/02(土) 23:41:13.51 ID:???.net
>>495
単位接ベクトルは一意に決まってますけど…w

499 :ご冗談でしょう?名無しさん:2024/03/02(土) 23:42:02.12 ID:???.net
マジで何が問題なのか理解してないの?

500 :ご冗談でしょう?名無しさん:2024/03/02(土) 23:43:09.27 ID:???.net
単位接ベクトルが一意に決まって加速度の接成分が速度の大きさの時間微分になってて草
完全論破とはこのことかw

501 :ご冗談でしょう?名無しさん:2024/03/02(土) 23:45:10.88 ID:???.net
あーあ完全論破されちゃったねぇ

502 :ご冗談でしょう?名無しさん:2024/03/02(土) 23:45:22.24 ID:???.net
>>500
加速度の接線方向の成分の向きが速度ベクトルと逆になりますよー

503 :ご冗談でしょう?名無しさん:2024/03/02(土) 23:45:42.04 ID:???.net
>>501
見下してた相手に負けてて可哀相

504 :ご冗談でしょう?名無しさん:2024/03/02(土) 23:46:46.08 ID:???.net
「ベクトル場に沿った微積分」ってやったことない?

505 :ご冗談でしょう?名無しさん:2024/03/02(土) 23:47:13.38 ID:???.net
>>502
成分の向きってなんすかw
加速度の接線成分はスカラーですよ…w
ベクトルは単位ベクトルと成分の積なんすよ…w

506 :ご冗談でしょう?名無しさん:2024/03/02(土) 23:48:08.77 ID:???.net
>>505
ベクトルを数ベクトルとして表示したら各成分がスカラーになるだけですよー
そしてスカラーでも正負はありますよー

507 :ご冗談でしょう?名無しさん:2024/03/02(土) 23:49:04.64 ID:???.net
「ベクトル場に沿った微積分」ってやったことある?

508 :ご冗談でしょう?名無しさん:2024/03/02(土) 23:52:55.03 ID:???.net
「ベクトルの大きさ」って向きの情報が失われてるからそりゃそうなるわなって感じ

509 :ご冗談でしょう?名無しさん:2024/03/02(土) 23:56:53.73 ID:???.net
他人を見下した態度取るからこうなるんやで
>>241←これとか

510 :ご冗談でしょう?名無しさん:2024/03/02(土) 23:58:19.09 ID:???.net
>>506
そうだねその正負が微分に内包されてるのを理解してねw

511 :ご冗談でしょう?名無しさん:2024/03/02(土) 23:59:09.96 ID:???.net
>>509
ベクトルの基本すら出来てないやつが見下されるのは当然だろ…

512 :ご冗談でしょう?名無しさん:2024/03/02(土) 23:59:39.12 ID:???.net
まだ負けてないつもりなのか

もう相手できんわ

513 :ご冗談でしょう?名無しさん:2024/03/03(日) 00:02:51.53 ID:???.net
結局>>496で猿でも分かる証明になったろw
単位接ベクトルは一意に決まります
加速度の接線方向は速度の大きさの時間微分です
逆に速度の大きさは加速度の接線方向の積分です
>>222の猿でも分かる証明w

514 :ご冗談でしょう?名無しさん:2024/03/03(日) 00:03:36.28 ID:???.net
逃げてて草
まあ数学で完全論破するとそうするしかないよな

515 :ご冗談でしょう?名無しさん:2024/03/03(日) 00:06:52.54 ID:???.net
単位接ベクトルe_t=dr/ds
速度ベクトルv=│v│e_t
加速度ベクトルa=dv/dt
この三つは正直自明レベルの話なんだがそれすら分からない人がいたということ
これが自明とすれば後はv=∫adtだけなのでこれが答え

516 :ご冗談でしょう?名無しさん:2024/03/03(日) 00:12:46.62 ID:???.net
>>513
>>440の例でそれやってみるよ

単位接ベクトルe_t=dr/ds=1
速度ベクトルv=│v│e_t←vで循環してね??

まあ譲歩して意図を汲んであげるとすれば、
接加速度ベクトル
a_t=(d|v|/dt)e_t
=(-2)*(1)
=-2

しかしa=a_t=d^2r/dt^2=2

517 :ご冗談でしょう?名無しさん:2024/03/03(日) 00:16:06.04 ID:???.net
>>516
逆にこれe_t=dr/ds=-1とすると、t>0で今度は合わなくなる
tの値で場合分けすればいいと思うかもしれないが、そんなことができるのは今考えてる例が単純だから
例えば
(sin(πcost)+cos(2t),cosh(exp(t))+t)みたいな出鱈目な速度ベクトルの場合どうするか
いちいち正負を調べるのは非合理

518 :ご冗談でしょう?名無しさん:2024/03/03(日) 00:18:26.50 ID:???.net
tの値で場合分けするんじゃなくてそもそもe_tはtの関数なんだよ

519 :ご冗談でしょう?名無しさん:2024/03/03(日) 00:19:02.18 ID:???.net
>>518
いや実質同じことだけど

520 :ご冗談でしょう?名無しさん:2024/03/03(日) 00:19:26.70 ID:???.net
接続を導入すればそんな煩わしさからは解放されますよ

521 :ご冗談でしょう?名無しさん:2024/03/03(日) 00:19:50.71 ID:???.net
もしかしてtangentのtじゃなくてtimeのt!?

522 :ご冗談でしょう?名無しさん:2024/03/03(日) 00:21:01.04 ID:???.net
そもそもrはtの関数なんだから面倒もクソもない

523 :ご冗談でしょう?名無しさん:2024/03/03(日) 00:22:41.28 ID:???.net
>>520
おまえがぐだぐだ言ってんのがいちばん煩わしいと思うよ

524 :ご冗談でしょう?名無しさん:2024/03/03(日) 00:23:00.16 ID:???.net
>>517
滑らかであれば別にe_t(t)=dr(t)/ds(t)で各tの単位接ベクトルは一意に定まる

525 :ご冗談でしょう?名無しさん:2024/03/03(日) 00:23:14.43 ID:???.net
>>523
多様体論を勉強しよう!!

526 :ご冗談でしょう?名無しさん:2024/03/03(日) 00:25:19.68 ID:???.net
>>524
|v|=f(t)とすると

e_t=(sgn((f^(-1))'(t))dr/ds
これならいけるけど、逆関数を含むね

527 :ご冗談でしょう?名無しさん:2024/03/03(日) 00:25:34.06 ID:???.net
そもそも元の問題が一般の滑らかな曲線という条件なのだからそれで通用する一般形式でいい

528 :ご冗談でしょう?名無しさん:2024/03/03(日) 00:27:21.39 ID:???.net
>>525
そうやって会話にならない話しかけられ方するのが煩わしいと言わずになんといおう

529 :ご冗談でしょう?名無しさん:2024/03/03(日) 00:27:27.21 ID:???.net
>>526
いや逆関数含まないわ
バカ

530 :ご冗談でしょう?名無しさん:2024/03/03(日) 00:28:06.28 ID:???.net
>>526
これこうだわ
e_t=(sgn((f'(t))dr/ds

隙を見せちまった!!

531 :ご冗談でしょう?名無しさん:2024/03/03(日) 00:28:43.64 ID:???.net
まあsgnとかいう非解析的関数が入る時点で似たようなもんか

532 :ご冗談でしょう?名無しさん:2024/03/03(日) 00:29:24.38 ID:???.net
いやまだ違った

あーあ
勝ち確の雰囲気が壊れたわ

533 :ご冗談でしょう?名無しさん:2024/03/03(日) 00:30:44.21 ID:???.net
|v|じゃなくてvに関するsgn関数みたいなやつが要るわ
まあどっちにしろ非解析的関数ですよ

534 :ご冗談でしょう?名無しさん:2024/03/03(日) 00:32:33.09 ID:???.net
v=dr/dt
a=dv/dt
e_t=dr/ds

535 :ご冗談でしょう?名無しさん:2024/03/03(日) 00:33:13.75 ID:???.net
理解する気のない奴を相手にするのは無駄
放置が最適

536 :ご冗談でしょう?名無しさん:2024/03/03(日) 00:36:15.54 ID:???.net
v=dr/dt
a=dv/dt
e_t=dr/ds=dr/dt/ds/dt

537 :ご冗談でしょう?名無しさん:2024/03/03(日) 00:39:06.00 ID:???.net
つまんねー言い合いに終始してるってことは両方ともポンコツなんだからお開きにしろよ

538 :ご冗談でしょう?名無しさん:2024/03/03(日) 00:42:52.29 ID:???.net
ポンコツならやめどきわからず続けるでしょ

539 :ご冗談でしょう?名無しさん:2024/03/03(日) 00:45:42.47 ID:???.net
んなこと知らんわ
数学的に証明せい

540 :ご冗談でしょう?名無しさん:2024/03/03(日) 00:55:19.00 ID:???.net
レスバしてたうちの一人です
荒らしてすみませんでした

541 :ご冗談でしょう?名無しさん:2024/03/03(日) 01:00:18.28 ID:???.net
いえいえお気になさらずに

542 :ご冗談でしょう?名無しさん:2024/03/03(日) 01:16:13.80 ID:???.net
>>541
ありがとうございます
反省します

543 :ご冗談でしょう?名無しさん:2024/03/03(日) 09:35:54.42 ID:???.net
>>539
>>515でどうでしょうか

544 :ご冗談でしょう?名無しさん:2024/03/03(日) 09:43:16.27 ID:???.net
>>539が求めてるのは「ポンコツならやめどきわからず続けるでしょ」に対する証明やで?

545 :ご冗談でしょう?名無しさん:2024/03/03(日) 10:49:07.61 ID:???.net
圧力について質問です

水が流れている同一の配管上で部屋の大きさが違う場合、入り口と出口、各部屋の圧力の関係はどうなりますか?

イメージだとP>PG2>PG1>Rになるのかなと思いますが分からないのでよろしくおねがいします。

https://i.imgur.com/veaCjiq.jpg

546 :ご冗談でしょう?名無しさん:2024/03/03(日) 11:26:23.86 ID:???.net
それは上から見た図?横から見た図?
重力がどちらに働いてるかによって変わると思うけど

547 :ご冗談でしょう?名無しさん:2024/03/03(日) 11:49:39.20 ID:LLI5W3JE.net
群と物理(佐藤 光 著)を読み初めて早速躓きました
群論に関しては全くの初心者です
設問1.と2.について教えて下さい

正六角形を中心の周りに反時計方向に2π/6回転する操作をθで表す
この操作を続けてn回行えば(2π/6)n回転することになりこれをθ^nと表す
何も操作をしない恒等変換をeと表す
また、6本の対称軸に関する鏡像変換を
σ_i(i=1,2,3,4,5,6)で表す
正六角形を自分自身に移す全ての合同変換は次の12の操作である
{e,θ,θ^2,θ^3,θ^4,θ^5,σ_1,σ_2,σ_3,σ_4,σ_5,σ_6}

1.上記の部分集合{e,θ^2,θ^4,σ_1,σ_2,σ_3}は元の群の部分群になることを示せ
2.またこれ以外の部分群をすべてもとめよ

特に2.の解答は{e,σ_1}{e,σ_2}{e,σ_3}
{e,θ^3}{e,θ^2,θ^4}{e,θ,θ^2,θ^3,θ^4,θ^5}
だけなのですがその理由を教えて下さい

548 :ご冗談でしょう?名無しさん:2024/03/03(日) 11:54:36.50 ID:???.net
>>546
横から見た図です

549 :ご冗談でしょう?名無しさん:2024/03/03(日) 12:25:57.78 ID:???.net
>>545
P=R(PとRは高さも流速も等しいため)
PG2>PG1 (高所の方が圧力が低いため)
それ以外の関係は一概にはいえん

550 :ご冗談でしょう?名無しさん:2024/03/03(日) 13:46:54.84 ID:???.net
>>547
何が知りたいのかよくわからんな

551 :ご冗談でしょう?名無しさん:2024/03/03(日) 15:00:59.12 ID:???.net
>>547
場合分けをして全部調べれば良い
σの有無で分けろよ

552 :ご冗談でしょう?名無しさん:2024/03/03(日) 15:28:32.49 ID:???.net
1.は定義に従って確かめるだけ

553 :ご冗談でしょう?名無しさん:2024/03/03(日) 16:02:31.17 ID:LLI5W3JE.net
正六角形はどう角度を変えてもどうひっくり返しても正六角形というのではなく、全ての角を角1,角2・・・角5と区別して考えなきゃいけないということでしょうか

だとしたら{e,θ^2,θ^4}は部分群なのに
{e,θ^1,θ^5}は部分群でないのは何故でしょうか

554 :ご冗談でしょう?名無しさん:2024/03/03(日) 16:27:20.60 ID:X9pUOmU8.net
θ が部分群 H に入っているならば、 θ, θ^2, θ^3, θ^4, θ^5, θ^6 = e のすべてが H に入っていなければなりません。
部分群は2項演算について閉じているからです。

555 :ご冗談でしょう?名無しさん:2024/03/03(日) 16:28:51.53 ID:X9pUOmU8.net
θ^2 が部分群 H に入っているならば、 θ^2, θ^4, θ^6 = e のすべてが H に入っていなければなりません。
部分群は2項演算について閉じているからです。

そして、 {θ^2, θ^4, θ^6 = e} はそれ自体で群になっています。

556 :ご冗談でしょう?名無しさん:2024/03/03(日) 16:30:24.75 ID:X9pUOmU8.net
>{e,θ^1,θ^5}は部分群でないのは何故でしょうか

例えば、 θ^1 が入っているのに、 θ^1 * θ^1 = θ^2 が入っていません。

557 :ご冗談でしょう?名無しさん:2024/03/03(日) 16:37:23.28 ID:X9pUOmU8.net
以下では G の部分群で θ を含む部分群をすべて求めています。
こんな感じで、しらみつぶしにやっていけばいいんです。

G :={e,θ,θ^2,θ^3,θ^4,θ^5,σ_1,σ_2,σ_3,σ_4,σ_5,σ_6} とする。 


G の部分群が θ を含む場合を考える。
{e, θ, θ^2, θ^3, θ^4, θ^5} はその部分群の部分集合でなければならない。
{e, θ, θ^2, θ^3, θ^4, θ^5} 自体既に G の部分群である。

G の部分群が θ の他に σ_i を含む場合を考える。

{e, θ, θ^2, θ^3, θ^4, θ^5} ∪ {e, σ_i} をその部分群は最低でも含まなければならない。
まだその部分群は積について閉じているから {θ*σ_i, θ^2*σ_i, θ^3*σ_i, θ^4*σ_i, θ^5*σ_i} を含まなければならない。
{σ_i, θ*σ_i, θ^2*σ_i, θ^3*σ_i, θ^4*σ_i, θ^5*σ_i} = {σ_1, σ_2, σ_3, σ_4, σ_5, σ_6} であることは操作の定義から容易に分かる。
ここまでをまとめると、その部分群は、 {e, θ, θ^2, θ^3, θ^4, θ^5} ∪ {σ_1, σ_2, σ_3, σ_4, σ_5, σ_6} をその部分群は最低でも含まなければならない。
ところがこれは G である。
以上から、 G の部分群が θ を含む場合ならば、その部分群は、 {e, θ, θ^2, θ^3, θ^4, θ^5} または G でなければならない。
これで、 G の部分群が θ を含む場合についてはすべて考えた。

558 :ご冗談でしょう?名無しさん:2024/03/03(日) 16:47:21.78 ID:???.net
θ_0=e
θ_n*θ_m=θ_(n+m mod 6)
(θ_n)^-1=θ_(6-n) @n=1,2,3,4,5

{e, θ_1, θ_5}
θ_1*(θ_5)^-1=θ_1*θ_1=θ_2 ∉ {e, θ_1, θ_5}

{e, θ_2, θ_4}
θ_2*(θ_4)^-1=θ_2*θ_2=θ_4 ∈ {e, θ_2, θ_4}

559 :ご冗談でしょう?名無しさん:2024/03/03(日) 17:10:51.34 ID:???.net
群って物理で何の役に立つの?

560 :ご冗談でしょう?名無しさん:2024/03/03(日) 17:21:43.32 ID:LLI5W3JE.net
皆さん親切な解答ありがとうございます
結構しらみ潰しに考えないといけないんですね

あと自分は部分群の定義だけしか考えてなく元の群の定義を考慮していなかったようです

561 :ご冗談でしょう?名無しさん:2024/03/03(日) 17:23:19.84 ID:???.net
σ_n*σ*n=e
(σ_n)^-1=σ_n

{e, σ_n}
σ_n*(σ_n)^-1=e ∈{e, σ_n}

562 :ご冗談でしょう?名無しさん:2024/03/03(日) 19:37:18.20 ID:???.net
>>559
直交変換群とかで回転を表せりゅ

563 :ご冗談でしょう?名無しさん:2024/03/03(日) 19:52:26.01 ID:???.net
>>562
鏡映やパンティ変換表わせりゅ!!

564 :ご冗談でしょう?名無しさん:2024/03/03(日) 19:58:47.66 ID:???.net
>>549
ありがとうございます。
同一の経路であれば内部の形状が膨らんだり縮んだりしても圧力は入り口から出口に向けて下がっていくですね

565 :ご冗談でしょう?名無しさん:2024/03/03(日) 20:07:56.17 ID:???.net
>>564
なにを聞いとったんじゃおまえは

566 :ご冗談でしょう?名無しさん:2024/03/03(日) 20:19:13.81 ID:???.net
流体に粘性がれば下流のほうが圧力が下がるというのは正しい

567 :ご冗談でしょう?名無しさん:2024/03/03(日) 20:21:12.89 ID:???.net
山口人生とビルゲイツはどっちのほうがすごいですか?

568 :ご冗談でしょう?名無しさん:2024/03/03(日) 20:28:09.12 ID:???.net
馬鹿は繰り返す

569 :ご冗談でしょう?名無しさん:2024/03/03(日) 23:18:15.96 ID:???.net
ポアンカレの回帰定理によれば、力学系のある状態を出発点としたときに、その時間発展は出発点といくらでも近い状態に無限回戻ってくることを主張するらしいです。
https://ja.wikipedia.org/wiki/%E3%83%9D%E3%82%A2%E3%83%B3%E3%82%AB%E3%83%AC%E3%81%AE%E5%9B%9E%E5%B8%B0%E5%AE%9A%E7%90%86

ということは、真空の部屋2つを穴でつないで、最初に左側だけに気体を詰めた力学系は、
時間発展させると何度でも左側だけに気体が集まるということでしょうか?

570 :ご冗談でしょう?名無しさん:2024/03/03(日) 23:37:09.84 ID:???.net
物理量の次元(レベルは除く)はISQ基本次元{L,M,T,I,Θ,N,J}の整数乗の積として表されることが多いですね(組立次元)
これは素因数分解に似てると思います
物理的に存在する全ての次元を組立次元の表現として一意に表せるような次元の集合というものはどうなるのでしょう
言わば素数ではなく「素次元」となるのでしょうか

ISQ基本次元は素次元なのでしょうか?

571 :ご冗談でしょう?名無しさん:2024/03/04(月) 00:06:04.17 ID:???.net
>>569
すごく長い時間スケールで待てば、理屈としてはそだろうね
どれくらいかかるかは見当つかんけど

572 :ご冗談でしょう?名無しさん:2024/03/04(月) 00:22:17.53 ID:???.net
>>570
きみの言葉遊びの話を他人に聞いてどうすんの
他人が答え持ってるわけないでしょう

573 :ご冗談でしょう?名無しさん:2024/03/04(月) 00:36:29.79 ID:???.net
>>572
言葉遊びの部分は正直どうでもいいのです
ISQ基本次元がこの世の物理次元を全て一意に表せるかが訊きたいのです

574 :ご冗談でしょう?名無しさん:2024/03/04(月) 11:01:53.66 ID:???.net
でぼーん

575 :ご冗談でしょう?名無しさん:2024/03/04(月) 11:07:29.81 ID:???.net
コイルの巻数とかトポロジカルな量は物理の単位系で表現できないのはなにか気持ちが悪い

576 :ご冗談でしょう?名無しさん:2024/03/04(月) 11:30:24.00 ID:???.net
ISQって歯医者の話だよね
なんで物理板に書くのかな?

577 :ご冗談でしょう?名無しさん:2024/03/04(月) 11:51:13.74 ID:???.net
正直カンデラは基本次元に含めなくていい気がする

578 :ご冗談でしょう?名無しさん:2024/03/04(月) 13:52:54.14 ID:???.net
人間の感覚だもんな

579 :ご冗談でしょう?名無しさん:2024/03/04(月) 18:51:35.53 ID:???.net
>>570
素数は線形独立なスペクトルの基底だと思うほうが物理学的だと思う。

>>575
整数で示してる周波数みたいなもんだからなあ。

580 :ご冗談でしょう?名無しさん:2024/03/04(月) 20:09:40.83 ID:???.net
でーぼんしゃいあーたーろ?

たろたーろ たろたーろ たろたーろ

でーぼんしゃいあーたろたーろ

581 :ご冗談でしょう?名無しさん:2024/03/04(月) 22:12:17.60 ID:???.net
>>575
ディラックの量子化条件あたりが量子物理学とトポロジカルな離散量の関係性の際たるものってとこか

582 :ご冗談でしょう?名無しさん:2024/03/05(火) 01:02:36.69 ID:uafRNAbs.net
四本脚と二本脚の体重は同じように計れるのか??

583 :ご冗談でしょう?名無しさん:2024/03/05(火) 01:07:19.05 ID:uafRNAbs.net
同量のタワーと平屋は同じ数値を示すか?

584 :ご冗談でしょう?名無しさん:2024/03/05(火) 09:43:30.56 ID:???.net
体重計に両足で乗った場合と片足で乗った場合では測定結果は違う

585 :ご冗談でしょう?名無しさん:2024/03/05(火) 10:25:22.40 ID:???.net
チカイモンは歌って踊ってみんなを楽しませるのが得意なのだ。

「チーカイモンー」
♪ ∧  ∧
⎛c=*•ヮ•=⎞♪
⎛⊃     ⊃⎞

「チーカイモンー」
  ∧  ∧ ♪
♪⎛=*•ヮ•=ↄ⎞
⎛⊂     ⊂⎞

「ホンワカパッパホンワカパッパ」
  ∧  ∧
⎛     ⎞クルッ
⎛      ⎞

「チーカイモンー」
  ∧  ∧
⎛c=*•ヮ•=⎞クルリンパ♪
⎛⊂     ⊃⎞

586 :ご冗談でしょう?名無しさん:2024/03/05(火) 12:38:43.91 ID:???.net
>>579
対数取ってってことですか?

587 :ご冗談でしょう?名無しさん:2024/03/05(火) 13:09:58.64 ID:???.net
でぼぼぼ でぼぼぼぼぼ でぼぼぼぼぼぼぼぼ

588 :ご冗談でしょう?名無しさん:2024/03/05(火) 13:36:20.79 ID:???.net
哀れやな

589 :ご冗談でしょう?名無しさん:2024/03/05(火) 14:59:06.59 ID:uafRNAbs.net
お菓子のグラム数はでたらめということだな?

590 :ご冗談でしょう?名無しさん:2024/03/05(火) 18:36:45.57 ID:???.net
表記から2%の誤差は許容されてるので問題ない

591 :ご冗談でしょう?名無しさん:2024/03/05(火) 21:49:33.93 ID:???.net
>>268
固有時、固有時間の使い分けは一般的やで

592 :ご冗談でしょう?名無しさん:2024/03/05(火) 21:51:37.79 ID:???.net
おまえの中でわな

593 :ご冗談でしょう?名無しさん:2024/03/05(火) 22:03:04.00 ID:???.net
少なくとも「一般相対性理論を一歩一歩数式で理解する」って本では使い分けられてるし、以下のページでは固有時間の微分形を固有時と呼んでる
微分形を「固有時」と呼ぶなら、同時に「固有時『間』」とは呼ばないでしょ
https://ja.wikipedia.org/wiki/%E5%9B%BA%E6%9C%89%E6%99%82
https://eman-physics.net/relativity/proper.html
https://butsurimemo.com/world-interval/

594 :ご冗談でしょう?名無しさん:2024/03/05(火) 22:18:45.22 ID:???.net
どつちでもいいから表現がわかりやすいように勝手に使い分けることもあるけど、本来どっちも同じ意味やで

595 :poem:2024/03/06(水) 00:04:39.38 ID:M1Jz7Q29.net
ちんちんかいかいモン
だって
なんだこの呪文

596 :ご冗談でしょう?名無しさん:2024/03/06(水) 00:28:00.37 ID:???.net
電力需要が増えると商用電源の周波数が低下するというのは本当ですか?

597 :ご冗談でしょう?名無しさん:2024/03/06(水) 00:31:58.38 ID:???.net
嘘です

598 :ご冗談でしょう?名無しさん:2024/03/06(水) 01:46:39.48 ID:???.net
>>596
北朝鮮は慢性的な電力不足で夜真っ暗、電圧も周波数も低下してるだろな

599 :ご冗談でしょう?名無しさん:2024/03/06(水) 01:53:09.70 ID:???.net
日本では電圧、周波数が低下する前に重要でない地域から強制停電させる
自分の住んでる地域が過去に強制停電されてれば重要地域でない証拠だ

600 :ご冗談でしょう?名無しさん:2024/03/06(水) 09:16:17.99 ID:???.net
周波数変動を測定している人たちがいる
https://tomono.tokyo/2021/01/11/9106/

601 :ご冗談でしょう?名無しさん:2024/03/06(水) 09:18:22.44 ID:???.net
リアルタイムグラフ
https://www2.zxvf.jp/freqWatch/

602 :ご冗談でしょう?名無しさん:2024/03/06(水) 09:28:20.74 ID:???.net
株価のようにランダムに変動してるように見える

603 :ご冗談でしょう?名無しさん:2024/03/06(水) 10:00:51.87 ID:???.net
>>602
>ランダムに変動してるように見える

それが電源のエントロピーに対応している

604 :ご冗談でしょう?名無しさん:2024/03/06(水) 10:12:11.92 ID:???.net
バカは黙ってろ

605 :ご冗談でしょう?名無しさん:2024/03/06(水) 10:18:16.01 ID:???.net
ランダムに変動してるように見える->電源のエントロピーに対応している
確率統計で定義できる物理的現象だ、お前らが無知なだけ

606 :ご冗談でしょう?名無しさん:2024/03/06(水) 10:23:31.84 ID:???.net
sustainable energy -without the hto air
という本の196ページに、周波数変動を検知して使用者側が需要をコントロールして供給側の負荷を減らすアイディアが書いてある。
https://www.withouthotair.com/c26/page_196.shtml

607 :ご冗談でしょう?名無しさん:2024/03/06(水) 10:33:41.45 ID:???.net
インドの周波数変動すごすぎ
https://posoco.in/reports/frequency-profile/

608 :ご冗談でしょう?名無しさん:2024/03/06(水) 10:38:53.94 ID:???.net
ムーニーちゃんしんぷのおじさんがすごいってことは知ってるだろ?

609 :ご冗談でしょう?名無しさん:2024/03/06(水) 12:43:25.36 ID:???.net
tepcoよりkepcoのほうが若干安定しているように見える

610 :ご冗談でしょう?名無しさん:2024/03/06(水) 13:25:28.96 ID:???.net
>>593
固有時間の微分形は固有時ではない
それは省略してるだけ

611 :ご冗談でしょう?名無しさん:2024/03/06(水) 13:30:03.72 ID:???.net
>>596
商用電源の周波数は発電機の回転数なので負荷が増えれば一時的に低下する
しかし周波数を保持するシステムがあるので発電力に余裕があれば回復する

612 :ご冗談でしょう?名無しさん:2024/03/06(水) 13:33:15.01 ID:???.net
>>605
変動元は電力需要だ
無知はお前

613 :ご冗談でしょう?名無しさん:2024/03/06(水) 13:35:25.53 ID:???.net
>>609
50Hzより60Hzの方が良いね

614 :ご冗談でしょう?名無しさん:2024/03/06(水) 13:42:18.92 ID:???.net
>>612
頭悪い奴だな
変動元が何だろうが周波数がランダム変動することで成立する熱現象と同様

615 :ご冗談でしょう?名無しさん:2024/03/06(水) 13:47:01.28 ID:???.net
「電源のエントロピー」と言う造語にケチつけてるだけ

616 :ご冗談でしょう?名無しさん:2024/03/06(水) 14:15:52.69 ID:???.net
需要変動が熱現象???????????????

617 :ご冗談でしょう?名無しさん:2024/03/06(水) 14:45:53.74 ID:???.net
お前はランダムの意味が理解できない低脳だな

618 :ご冗談でしょう?名無しさん:2024/03/06(水) 14:54:25.60 ID:???.net
人間が個々の理由で電気をより多く使うか少なく使おうが電圧・周波数はランダムに変動している

619 :ご冗談でしょう?名無しさん:2024/03/06(水) 14:56:29.84 ID:???.net
熱雑音のことを言ってるの?

620 :ご冗談でしょう?名無しさん:2024/03/06(水) 15:02:40.84 ID:???.net
電源のエントロピーが大きい国の電源はクォリティー(品質)が悪いということだ

621 :ご冗談でしょう?名無しさん:2024/03/06(水) 15:22:05.13 ID:???.net
>>618は、熱雑音のことを指しているの?

622 :ご冗談でしょう?名無しさん:2024/03/06(水) 15:24:40.03 ID:???.net
電源のエントロピーってどう定義するの?

623 :ご冗談でしょう?名無しさん:2024/03/06(水) 15:31:16.63 ID:???.net
>>621
お前は一か所だけが電力を使うと思い込んでるのか
非常に多くの家庭や事業所が個々の都合で使えば基の電源電圧と周波数がランダムに変動する
それすら分からんのか

624 :ご冗談でしょう?名無しさん:2024/03/06(水) 15:34:47.54 ID:???.net
>>622
このスレか何処かで式を書いてやってるから探せ

625 :ご冗談でしょう?名無しさん:2024/03/06(水) 15:35:51.67 ID:???.net
>>623
それが熱とエントロピーと何の関係があるの?

626 :ご冗談でしょう?名無しさん:2024/03/06(水) 15:39:49.36 ID:???.net
>>624
書いてないようなのでもう一度数式で書いてください。へんな文章による定義は要りません。

627 :ご冗談でしょう?名無しさん:2024/03/06(水) 15:42:05.63 ID:???.net
そのエントロピーはエントロピーの性質を満たしてないですね

628 :ご冗談でしょう?名無しさん:2024/03/06(水) 15:45:31.60 ID:???.net
>>625
熱現象にはエントロピーがあるだろが
情報通信には(負の)エントロピーがある
電源のエントロピーを定義すれば電源のクォリティー(品質)が定量的に比較できる
めでたし、めでたし

629 :ご冗談でしょう?名無しさん:2024/03/06(水) 16:05:49.54 ID:???.net
>>628
おまけ
ハイクォリティー(高品質)の電源を創るにはどうすればいいか?
答え
つまり、電源のエントロピーを小さくするには情報の負のエントロピーで制御すれば
よい。

630 :ご冗談でしょう?名無しさん:2024/03/06(水) 16:05:58.19 ID:???.net
気が狂ってやがる

631 :ご冗談でしょう?名無しさん:2024/03/06(水) 18:21:14.15 ID:???.net
物理にウリジナル定義は不要

632 :ご冗談でしょう?名無しさん:2024/03/06(水) 18:30:55.22 ID:???.net
>>629
>電源のエントロピーを小さくするには情報の負のエントロピーで制御すればよい。

現実に日本の電力会社はその発電システムでハイクォリティー(高品質)の電源を供給している

633 :ご冗談でしょう?名無しさん:2024/03/06(水) 18:42:57.24 ID:???.net
このバカはほっとくとして
https://www.withouthotair.com/c26/page_198.shtml
ここでは電気自動車のバッテリーがエネルギーのバッファとして機能すると書いてあるけど
自動車メーカーが軒並みEVから撤退しているのを見るとそれも厳しそうだ

634 :ご冗談でしょう?名無しさん:2024/03/06(水) 18:43:51.43 ID:???.net
俺様定義は無視が吉

635 :ご冗談でしょう?名無しさん:2024/03/06(水) 19:05:02.42 ID:???.net
>>632
頭の悪い村人はエントロピーの概念から制御工学と情報理論が結び付いてるのが理解出来ないらしい。

636 :ご冗談でしょう?名無しさん:2024/03/06(水) 19:22:13.86 ID:???.net
重力蓄電おもしろそう
https://spaceshipearth.jp/gravity-storage/

637 :ご冗談でしょう?名無しさん:2024/03/06(水) 21:50:43.81 ID:???.net
商用電源の安定性は、電圧サージの有無や電圧変動、周波数変動といった特徴量で判定できるようだ。当然、エントロピーなんかは絶対に出て来ない。

638 :ご冗談でしょう?名無しさん:2024/03/07(木) 03:15:47.56 ID:???.net
>>637
それな古い電気技術屋の発想、俺も電気業界だったから物理の理論一貫性など考えない

639 :ご冗談でしょう?名無しさん:2024/03/07(木) 05:21:47.61 ID:???.net
ぼくのかんがえた新ぶつり学の話は他所でやれって

640 :ご冗談でしょう?名無しさん:2024/03/07(木) 08:33:00.66 ID:???.net
電気屋の末路がこれか

641 :ご冗談でしょう?名無しさん:2024/03/07(木) 09:23:54.79 ID:???.net
ムニちゃーんポポ

642 :ご冗談でしょう?名無しさん:2024/03/07(木) 12:49:53.50 ID:???.net
何のエントロピーを計算するのかは知らんが、エントロピーは分布全体を順序なく計算するから時系列データの異常判定にはどのみち機能しないのは明らか

643 :ご冗談でしょう?名無しさん:2024/03/07(木) 13:37:24.01 ID:???.net
電源エントロピー教じゃね?

644 :ご冗談でしょう?名無しさん:2024/03/07(木) 20:44:03.49 ID:???.net
>>569

Arnol'd, V.I.. Mathematical Methods of Classical Mechanics (Graduate Texts in Mathematics)

D Poincaré’s recurrence theorem
E Applications of Poincaré’s theorem

に解説があった。この本おもしろい。

645 :ご冗談でしょう?名無しさん:2024/03/08(金) 10:10:27.62 ID:???.net
しんぷさんもムーニーちゃん プー

646 :ご冗談でしょう?名無しさん:2024/03/08(金) 10:24:08.87 ID:???.net
https://nazology.net/archives/146332

0.43ミリグラムの物体が発する「重力」の測定に成功!

すごすぎる

647 :ご冗談でしょう?名無しさん:2024/03/08(金) 13:02:07.79 ID:???.net
>物体が発する「重力」
発するとか意味不明
重力は距離の2乗に反比例するから質量同士の距離が非常に小さければ測定可能か

648 :ご冗談でしょう?名無しさん:2024/03/08(金) 14:26:06.35 ID:???.net
言葉尻にしか反応できんのか

649 :ご冗談でしょう?名無しさん:2024/03/08(金) 14:44:05.20 ID:???.net
下校中の小学生をジロジロ眺めていたけど、今の小学生女子のランドセルって紫が多いんだな
プリキュアでも紫が人気だから、幼女は紫色が好きなのかね
これがあと10年もしたらいやらしい紫色の下着をつけて男を誘惑するわけだから、俺、嫌になっちゃうよ

650 :ご冗談でしょう?名無しさん:2024/03/09(土) 01:03:33.25 ID:WWI0otbj.net
風呂につかりながら、栓を抜いて手を当てると手が吸い込まれるけど、一見圧力で上から押されるように頭じゃイメージするのに、やっぱり手は押されるより吸い込まれてる。

手は気圧とつりあうように外側に膨張圧力があるのかな。負圧になったとき、そっちに膨張変形して手が引っ張られてるような。

宇宙船で生物が外に出ると爆発する説あるけど、実際のとこはどうなんだろう。

651 :ご冗談でしょう?名無しさん:2024/03/09(土) 01:11:06.65 ID:???.net
>>650
手が変形する分は手の内部の圧力が大きい方から小さい方へ力がかかった結果じゃないかな
圧力勾配の負ベクトル

652 :ご冗談でしょう?名無しさん:2024/03/09(土) 11:35:57.74 ID:???.net
>>650
人間の皮膚は丈夫だから爆発なんかしないよ
せいぜい内出血

653 :ご冗談でしょう?名無しさん:2024/03/09(土) 17:29:06.40 ID:???.net
仰向けに寝て、左右の尻肉の下にそれぞれ左右の両手を敷いてから手のひらの中心を持ち上げるようにすると
尻肉の重みが乗って、まるで吸引力が下方に加わるような感じを受ける。

654 :ご冗談でしょう?名無しさん:2024/03/09(土) 17:43:14.31 ID:???.net
余計なお世話かもしれんけど、他にやることないんかいな

655 :ご冗談でしょう?名無しさん:2024/03/09(土) 20:31:40.35 ID:???.net
何か意味のある考察しようかと思ったけど
思いつかんかった(未だ未だだね)

656 :ご冗談でしょう?名無しさん:2024/03/09(土) 21:54:25.15 ID:???.net
量とは加法完全族の要素のことですか

657 :ご冗談でしょう?名無しさん:2024/03/09(土) 23:40:28.41 ID:???.net
統計力学と熱力学は同じく平衡系を記述する論理体系ですが、どちらがより基本的な理論なのでしょうか?

658 :ご冗談でしょう?名無しさん:2024/03/10(日) 00:06:03.01 ID:???.net
基本的という日本語がどちらのニュアンスか曖昧だけど、
基礎的という意味合いでは熱力学、根本的とかより掘り下げた内容という意味合いなら統計力学かな

659 :ご冗談でしょう?名無しさん:2024/03/10(日) 13:12:32.55 ID:???.net
平衡系に限らんし

660 :ご冗談でしょう?名無しさん:2024/03/10(日) 13:15:54.04 ID:???.net
>>656
加法完全族て何?
関係あるか知らんけど量に完全加法性はないよ

661 :ご冗談でしょう?名無しさん:2024/03/10(日) 13:39:30.65 ID:???.net
でぼんしゃいあー パー

662 :ご冗談でしょう?名無しさん:2024/03/10(日) 15:12:22.51 ID:???.net
相対論による計算でも真円を描く質点の運動速度は
v=√(GM/r)
でニュートン力学の結果とまったくの同一になる
(但し、vはシュバルツシルト計量の時間による測定、rはシュバルツシルト計量の動径)

相対的補正が入ると思ってたけどそんなことは全然なかった
これは何を意味するのだろうか

思うにrはニュートン力学の距離とは異なり、その意味を図ることは難しい
寧ろニュートン力学で上記の速度の軽算式から逆算した距離をシュバルツシルト計量の動径方向測定値と定義すると考えるのが自然であろう

663 :ご冗談でしょう?名無しさん:2024/03/10(日) 16:02:01.30 ID:???.net
>>662
なにが難しいのかしらんが、
・当事者視点ではニュートン力学より重力が強く速く回っている
・観測者視点ではそれがゆっくりに見える
両方の効果が相殺されてニュートン力学と同じ計算になるだけ

664 :ご冗談でしょう?名無しさん:2024/03/10(日) 17:34:04.36 ID:7vmRRQmw.net
なかなか奥深さが知れないとは思うが、シュバルツシルト計量では物差しで測るような実際の距離はrではなく、
ds=dr/√(1-a/r)
で表される。

円運動の速さと半径の関係がこのようなニュートン力学の実測距離ではなくシュバルツシルト計量の動径rに置き換わって成立しているというのが趣深い

665 :ご冗談でしょう?名無しさん:2024/03/10(日) 17:35:51.25 ID:7vmRRQmw.net
また速度vも固有時間τで測定した速度ではなく、シュバルツシルト計量の時間tでこの法則が成立しているところも味わいがある。

666 :ご冗談でしょう?名無しさん:2024/03/10(日) 17:55:58.68 ID:???.net
普通は計量を等方化した座標を使うから円軌道速度は
v = √(GM/r) / (1+GM/(2c^2 r))^3 だぞ
等方化したシュバルツシルト計量は
(ds)^2 = (1-GM/(2c^2 r))^2(cdt)^2/(1+GM/(2c^2 r))^2 - (1+GM/(2c^2 r))^4 ((dr)^2+(rdφ)^2+(r cosφ dθ)^2)

667 :ご冗談でしょう?名無しさん:2024/03/10(日) 18:08:47.15 ID:???.net
きみら、趣とか、味わいとか、普通はとか、なにか言ってるようで結局ほとんどなにも言ってないの多いよな

668 :ご冗談でしょう?名無しさん:2024/03/10(日) 18:09:38.45 ID:7vmRRQmw.net
ちなみにこのシュバルツシルト計量の時間tで円周方向の光の速さを測定するとcではなく
c√(1-a/r)
となる。
r=3a/2の地点で円運動できる速さは
c/√3
であるが、この地点の光速と一致するため、これより半径が小さい場所ではもはやいかなる物体も円運動することができない。

669 :ご冗談でしょう?名無しさん:2024/03/10(日) 18:11:04.17 ID:7vmRRQmw.net
>>666
そのvは等方計量(≠シュバルツシルト計量)の時間で測定した速度だよね。
シュバルツシルト計量の時間で測定するとニュートン力学の結果と形式が一致するのが深い

670 :ご冗談でしょう?名無しさん:2024/03/10(日) 18:13:01.59 ID:???.net
>>667
物理は感じるんだよ
例えばディラック方程式は美しい
物理学者はディラック方程式を見ると涙を流します

671 :ご冗談でしょう?名無しさん:2024/03/10(日) 18:22:42.01 ID:???.net
物理に人間の感情入れたらダメだろ
そういうのをとことん排除するのが自然科学の手法なんだから

672 :ご冗談でしょう?名無しさん:2024/03/10(日) 18:41:01.74 ID:???.net
感動は発見の原動力だぞ

673 :ご冗談でしょう?名無しさん:2024/03/10(日) 19:04:52.19 ID:???.net
ここ20~30年の超弦理論の停滞は物理学者が感動を忘れかけているせいだと思っている。
どうせ実験では測定不可能だからと数学的な観念の遊戯に陥っているのだ

674 :ご冗談でしょう?名無しさん:2024/03/10(日) 19:16:32.60 ID:???.net
停滞してる分野ってのは、将来的に見込みが薄いから停滞してるんだと思うんだが

675 :ご冗談でしょう?名無しさん:2024/03/10(日) 19:18:47.83 ID:???.net
賢者は苦痛の中で発見に至り、
愚者は感動の中で発見を安易に創作する

676 :ご冗談でしょう?名無しさん:2024/03/10(日) 19:29:34.26 ID:???.net
>>674
超弦理論が真実から遠いということだよね
ならみんな超弦理論から撤退して他の分野に注力する方が生産的だよねえ

677 :ご冗談でしょう?名無しさん:2024/03/10(日) 20:40:41.84 ID:???.net
んだんだ

678 :ご冗談でしょう?名無しさん:2024/03/10(日) 21:35:12.28 ID:???.net
>>662
シュバルツシルト系における動径座標は時刻座標ctおよび角度座標θ,φに対して直交かつ

∮ds=∫[0,2π]f(r)dφ=2πr

となるように計量g_{φφ}=f(r)を定められた座標なのでしゅ
言ってしまえば特異点に対して静止かつ無限遠の点から見た系で、ニュートン力学的極限と一致しましゅ
それ故に動径座標もニュートン力学と一致しましゅが、これはあくまでも座標動径でしゅ
実際の特異点からの距離、いわゆる固有距離で考えると当然ながら話は変わってくるので注意でしゅ

679 :ご冗談でしょう?名無しさん:2024/03/10(日) 21:37:30.64 ID:???.net
>>678
あ、円周に沿った積分でしゅ
周回積分ならなんでもいいわけじゃないでしゅ……

680 :ご冗談でしょう?名無しさん:2024/03/10(日) 21:56:25.00 ID:???.net
リリィ・シュシュのすべて

681 :ご冗談でしょう?名無しさん:2024/03/10(日) 22:02:58.42 ID:???.net
>>678
> 実際の特異点からの距離、いわゆる固有距離で考えると当然ながら話は変わってくるので注意でしゅ

じゃあ、実際の特異点からの距離? 固有距離? とやらを計算してみ
そんなものがあるのならな

682 :ご冗談でしょう?名無しさん:2024/03/10(日) 22:30:24.87 ID:???.net
>>681
>>664に書いてあるdsを積分すればいいだろ。
だれが計算してもどの軽量で計算しても同じ値になる。
ローレンツ不変量だからな

683 :ご冗談でしょう?名無しさん:2024/03/10(日) 22:39:42.41 ID:???.net
超弦理論が停滞て何を言ってるやら

684 :ご冗談でしょう?名無しさん:2024/03/10(日) 22:42:03.60 ID:???.net
>>682
なら単純に a=1とし、始点を特異点、終点をa=2としよう
さてどう積分し、その結果答えはどうなった?

685 :ご冗談でしょう?名無しさん:2024/03/10(日) 22:46:29.57 ID:???.net
>>681
固有距離ってのは∫dsのことでしゅ
>>684
動かすのはaじゃなくてrでしゅ

686 :ご冗談でしょう?名無しさん:2024/03/10(日) 22:46:32.88 ID:???.net
a=1を跨いだらあかんやろ

687 :ご冗談でしょう?名無しさん:2024/03/10(日) 22:51:08.74 ID:???.net
>>686
そう、まず一つ目のツッコミとして、a=1 を跨いだら実数解は出ない

688 :ご冗談でしょう?名無しさん:2024/03/10(日) 22:52:11.30 ID:???.net
シュバルツシルト計量の限界だな
事象の地平面で正則な計量をとればいい感じになる

689 :ご冗談でしょう?名無しさん:2024/03/10(日) 23:08:21.75 ID:???.net
いい感じになるってどういうことだよ

>>664 の式に対してもう一つツッコミもしておくが
> ds=dr/√(1-a/r)

t の項はどこいった?
おそらくシュヴァルツシルト座標系で始点と終点の時刻座標どちらもt=0として計算してるんだと思うが、
別の座標系で見れば t 座標なんてどうとでも変わるぜ。
特定の座標系で同時刻線を引いてみて、その長さを測ってみたところで、それを「実際の距離」と言うことなんてできないってこと。

690 :ご冗談でしょう?名無しさん:2024/03/10(日) 23:14:59.81 ID:???.net
相対論で「距離」といったら時空間上の距離を指す
物差しで測るような距離など測り方によってどうとでも変わるから、それを「実際の距離」と呼ぶことはできないってことね。

691 :ご冗談でしょう?名無しさん:2024/03/10(日) 23:21:50.55 ID:???.net
>>689
固有距離は世界線に沿って移動する粒子が移動する道のりのことなので、たしかに世界線に依存する以上「実際の距離」って表現はおかしいでしゅね
しかし今考えてるのはシュバルツシルト系単体で、空間座標の変換の話なので多分セーフでしゅ

692 :ご冗談でしょう?名無しさん:2024/03/10(日) 23:29:21.50 ID:???.net
>>691
つまりシュバルツシルト系においてdt=0かつ特異点からの測地線となるような世界線のみを考えてるのでセーフだと思うでしゅ
他の系からみてdt'≠0で「実際の距離」じゃなくても大丈夫でしゅ

693 :ご冗談でしょう?名無しさん:2024/03/10(日) 23:30:24.19 ID:???.net
しゅしゅ

694 :ご冗談でしょう?名無しさん:2024/03/10(日) 23:30:30.60 ID:???.net
そういうことでしゅ

695 :ご冗談でしょう?名無しさん:2024/03/11(月) 00:01:41.35 ID:???.net
>>683
理論は進展してるの?
無矛盾に4つの力を統一できるの?

696 :ご冗談でしょう?名無しさん:2024/03/11(月) 00:06:59.89 ID:???.net
シュバルツシルト計量で測定した公式がニュートン力学と一致するのは不思議だけど固有時で測定した円運動速度
rdφ/dτ=√(GM/r)
ではなく、ローカル時間で測定した円運動速度
rdφ/dt=√(GM/r)
と公式が一致するのが味わい深い

697 :ご冗談でしょう?名無しさん:2024/03/11(月) 00:20:54.02 ID:???.net
>>692
じゃあそれで求めた特異点からの距離ってのに何の意味があるんだい?
事象の地平線の外側では静止した宇宙船からメジャーを垂らして測れる長さといえるかもしれんが、内側まで垂らせば切れてしまい測ることはできないし、内側では計算上も実数ではない値になる

698 :ご冗談でしょう?名無しさん:2024/03/11(月) 01:46:46.16 ID:???.net
特異点にこだわる必要ないじゃん。
a>1では実測距離になるんだから実用的な範囲でつかえばよろし

699 :ご冗談でしょう?名無しさん:2024/03/11(月) 04:37:32.68 ID:???.net
>>697
当方座標を使えばよろしいでしゅ

700 :ご冗談でしょう?名無しさん:2024/03/11(月) 06:16:56.42 ID:???.net
>>698
本人が特異点からの距離を考えると話してたからなんだが、だからそんなものは考えられないでしょうという話

>>699
何座標だろうがそれで何が求まっているわけでもない、本質的に意味のない計算だということに変わりないでしょう

701 :ご冗談でしょう?名無しさん:2024/03/11(月) 09:26:02.12 ID:???.net
>>700
等方座標を使えば事象の地平面は特異点にならないでしゅ

702 :ご冗談でしょう?名無しさん:2024/03/11(月) 09:47:36.67 ID:???.net
だから何?

703 :ご冗談でしょう?名無しさん:2024/03/11(月) 12:21:05.18 ID:???.net
>>702
特異点にならないので固有長が求まりましゅ
等方座標において計量は
ds^2=-((1+a/4R)/(1-a/4R))^2 dw^2+(1+a/4R)^4(dx^2+dy^2+dz^2)
と表されるらしいでしゅ
dw=dy=dz=0として、
s=∫_C ds=∫[0,r] (1+a/4R)^2 dx
この積分は被積分関数、積分範囲ともに実数なので、結果も実数でしゅ
よってw=y=z=0における固有長が求まりましゅ
また、シュバルツシルト時空は球対称かつ静的な時空なので、時刻座標における並進変換および空間座標における回転変換を用いることにより一般の事象における固有長sが求まりましゅ

704 :ご冗談でしょう?名無しさん:2024/03/11(月) 12:31:50.06 ID:???.net
きみは、どんな座標系を選ぶかによってころころ変わるような値を固有長と呼ぶの?

705 :ご冗談でしょう?名無しさん:2024/03/11(月) 12:40:55.73 ID:???.net
シュバルツシルト解の特異点には空間距離が存在しない。時間距離のみ
地平線内の全ての点から未来にあるから必ず特異点に行くことになる
逃れることはできない

706 :ご冗談でしょう?名無しさん:2024/03/11(月) 12:40:58.45 ID:???.net
>>704
変わらないでしゅよ

707 :ご冗談でしょう?名無しさん:2024/03/11(月) 12:43:26.99 ID:???.net
ムーニーちゃんダンスをおどるしんぷ
ムニポポダンスのおじさんだい

708 :ご冗談でしょう?名無しさん:2024/03/11(月) 15:58:43.32 ID:???.net
>>703
いややっぱ求まらないかもしれないでしゅね……
lim[x→+0] R=∞なら、広義積分の結果はRの形次第でしゅね……
Rの具体的な形はよくわからないでしゅし……
ちょっと考え直しましゅ

709 :ご冗談でしょう?名無しさん:2024/03/11(月) 22:01:39.17 ID:???.net
>>708
等方座標系のRは
R^2=x^2+y^2+z^2
でしょ

710 :ご冗談でしょう?名無しさん:2024/03/11(月) 22:14:15.47 ID:???.net
ちゃうわ
r=R(1+a/4R)^2
を満たすRだ

711 :ご冗談でしょう?名無しさん:2024/03/11(月) 22:20:22.42 ID:???.net
rとRの関係にtもwも介入しないからシュバルツシルト計量と同じ固有距離を定義できる。

712 :ご冗談でしょう?名無しさん:2024/03/11(月) 22:37:09.56 ID:???.net
>>709
lim[x→+0]R=0でしゅた
間違えましゅた

713 :ご冗談でしょう?名無しさん:2024/03/11(月) 22:49:03.05 ID:???.net
>>711
そうでしゅね

714 :ご冗談でしょう?名無しさん:2024/03/11(月) 23:05:25.25 ID:???.net
定義して遊ぶのは勝手だけど、それってなんかちょっとでも物理的に意味があるの?

715 :ご冗談でしょう?名無しさん:2024/03/12(火) 00:01:32.98 ID:???.net
シュバルツシルト計量も等方座標系もクルスカル=セケレシュ座標系も意味がなければ固有名詞はつかないよ

716 :ご冗談でしょう?名無しさん:2024/03/12(火) 00:06:19.38 ID:???.net
ちな、漏れが好きな座標系はルメートル座標系。
無限遠から特異点に向かって真っすぐに自由落下する観測者からみた座標系。

717 :ご冗談でしょう?名無しさん:2024/03/12(火) 00:22:33.95 ID:???.net
>>715
座標系には意味があるに決まってるだろ
意味がないのはそれらに対しておおよそ意味がない計算を勝手に定義するお遊び

718 :ご冗談でしょう?名無しさん:2024/03/12(火) 00:41:54.47 ID:???.net
等方でない座標系の意味は?

719 :ご冗談でしょう?名無しさん:2024/03/12(火) 00:52:01.00 ID:???.net
等方である座標系の対義語

720 :ご冗談でしょう?名無しさん:2024/03/12(火) 01:12:44.47 ID:???.net
>>717
意味があるんだからそれを解釈するためにさまざまな面から検討を加えるのも当然意味がある。

721 :ご冗談でしょう?名無しさん:2024/03/12(火) 05:50:38.25 ID:???.net
>>720
あなたのいう解釈って、無意味な計算をしてみて出てくる値に適当な名前をつけてみる遊びのことをいうんですか?ずいぶんお手軽な遊びですね

722 :ご冗談でしょう?名無しさん:2024/03/12(火) 10:44:21.78 ID:???.net
ダンシングムーニー

723 :ご冗談でしょう?名無しさん:2024/03/12(火) 11:38:53.54 ID:???.net
で、等方でない座標系の意味は?

724 :ご冗談でしょう?名無しさん:2024/03/12(火) 12:06:02.49 ID:???.net
>>721
dr/dtの代わりにds/dτを考えるのがそこまで無意味だとは思えないでしゅ

725 :ご冗談でしょう?名無しさん:2024/03/12(火) 12:32:57.13 ID:???.net
>>724
あ、rは座標位置のつもりでしゅたが動径と紛らわしいでしゅね
xの方がいいかもでしゅ

726 :ご冗談でしょう?名無しさん:2024/03/12(火) 16:46:36.47 ID:???.net
>>723
>>716みたいな意味の座標系があるお
自由落下してるから等位置線の延長線上に特異点があるんだお
同じ場所に留まってても終わりがくるんだお

727 :ご冗談でしょう?名無しさん:2024/03/12(火) 16:47:25.29 ID:???.net
>>721
だから無意味じゃないんだってば
計量の意味するところを正しく知る意義があるのこと

728 :ご冗談でしょう?名無しさん:2024/03/12(火) 18:52:04.76 ID:???.net
でぼぼぼ でぼぼぼぼぼ でぼぼぼぼぼぼぼ

729 :ご冗談でしょう?名無しさん:2024/03/12(火) 20:48:46.33 ID:???.net
ここ最近荒らしが減ったかわりにキモオタばっかり増えてきてね?

730 :ご冗談でしょう?名無しさん:2024/03/12(火) 20:58:28.56 ID:???.net
>>726
等方座標系で問題ないのに何の意味があるんだ?

731 :ご冗談でしょう?名無しさん:2024/03/12(火) 20:59:40.13 ID:???.net
>>729
落ちこぼれの劣等感は邪魔するしか救いがないのさ

732 :ご冗談でしょう?名無しさん:2024/03/12(火) 21:08:57.62 ID:???.net
だおだおでしゅでしゅでぼぼぼぼとか言いながらキモさにプライド持ってんの?

733 :ご冗談でしょう?名無しさん:2024/03/12(火) 21:13:16.08 ID:???.net
>>730
え!?

734 :ご冗談でしょう?名無しさん:2024/03/12(火) 21:13:59.92 ID:???.net
>>731
「さ」とかキモス(オエッ

735 :ご冗談でしょう?名無しさん:2024/03/12(火) 21:17:33.47 ID:???.net
>>721
計量を導出してからが大事。
計量の意味するところを吟味するのだ。

計量が出るだけで満足な香具師は計量を額縁に入れてうやうやしく拝んでいるがよい。
計量を吟味しなければ人類は光の湾曲も水星の近日点移動の知見も得ることはできないのだよ。

736 :ご冗談でしょう?名無しさん:2024/03/12(火) 21:24:57.16 ID:???.net
>>735
吟味って、きみら趣があるとか味わい深いとかいい感じとかソムリエみたいな感想しか出てこねーだろ

737 :ご冗談でしょう?名無しさん:2024/03/12(火) 21:44:28.45 ID:???.net
感情を失ったらそれは人間ではないのだ
それなら物理はAIに任せればいいのだ

738 :ご冗談でしょう?名無しさん:2024/03/12(火) 22:09:14.78 ID:???.net
隣り合う質点同士がバネに繋がれてる状況で縦波について考えるのは結構見るけど
横波についての理論とか考察ってありますか?
もしそういう本とかWebページがあれば教えて欲しいです

739 :ご冗談でしょう?名無しさん:2024/03/12(火) 22:25:39.50 ID:???.net
>>737
うん、だからAIでできそうなことは勝手にAI使っとったらええがな
なんで自分でしようとすんねん

740 :ご冗談でしょう?名無しさん:2024/03/13(水) 01:12:34.79 ID:???.net
>>739
そう
そして人は職を失い、労働生産性は向上する

741 :ご冗談でしょう?名無しさん:2024/03/13(水) 01:22:23.33 ID:???.net
社会不安で崩壊する世界

742 :ご冗談でしょう?名無しさん:2024/03/13(水) 06:20:11.21 ID:???.net
そもそも無職のおまえらが心配することじゃない

743 :ご冗談でしょう?名無しさん:2024/03/13(水) 08:46:28.08 ID:???.net
かじる親のスネが細くなるんだろ

744 :ご冗談でしょう?名無しさん:2024/03/13(水) 10:59:24.62 ID:???.net
絶望してる奴は心配しない
無差別殺人に走るだけ

745 :ご冗談でしょう?名無しさん:2024/03/13(水) 13:42:03.64 ID:???.net
身近にある机や本などにゆっくりと手を伸ばし、手が「触れた」と感じる位置と目が認識する位置は同じである。つまり、物体の大きさや形は人間の目に見える可視光に合致している。このことはそれらの物体を人間が自分の目で見て作ったのだから理解できる。しかし、自然物、例えば人類が生まれる以前から存在した岩や大樹、昆虫なども人間の可視光による認識と合致しているのが不思議でならない。人間の可視光なんて宇宙を飛び交う電磁波の中のごく一部に過ぎないのに。

746 :ご冗談でしょう?名無しさん:2024/03/13(水) 17:19:46.88 ID:Wjn5cj1u.net
蝙蝠🦇「超音波使ってます」
鮭「川産まれ海育ちですけど地磁気理解してるんで」

747 :ご冗談でしょう?名無しさん:2024/03/13(水) 17:25:59.15 ID:???.net
シュバルツシルト計量では動径方向がdr^2ではなく
dr^2/√(1-a/r)
となっているから、線素で動径方向の距離を実測すると円周距離の変化よりも大きく変化することが分かる。
こうする空間の歪みが実際に計算できて趣がある。

748 :ご冗談でしょう?名無しさん:2024/03/13(水) 17:26:52.45 ID:???.net
✕dr^2
◯dr

749 :ご冗談でしょう?名無しさん:2024/03/13(水) 19:43:15.34 ID:???.net
>>745
太陽光のスペクトルの内で地球大気を通過する範囲に合わせて生物が進化したから

750 :ご冗談でしょう?名無しさん:2024/03/13(水) 20:01:39.55 ID:???.net
シュバルツシルト計量で使われる等方でない座標系は計量の式が簡単になると言う意味しかない
物理的な意味のある結果を出したいなら等方座標系で表す必要がある
昔、海上保安庁水路部で発行していた天体位置表は等方化されたEIH計量で計算されていた
(EIHはEinstein, Infelt, Hoffmann)

751 :ご冗談でしょう?名無しさん:2024/03/13(水) 20:18:18.61 ID:???.net
でぼん でぼーん

752 :ご冗談でしょう?名無しさん:2024/03/13(水) 20:25:39.23 ID:???.net
>>750
物理的な意味のある結果って具体的に何?

753 :ご冗談でしょう?名無しさん:2024/03/13(水) 20:33:13.82 ID:???.net
>>749
岩とか水などは生物ですらないよ

754 :ご冗談でしょう?名無しさん:2024/03/13(水) 20:36:19.79 ID:???.net
>>750
実用面では実用的な計量を使えば良い
計量の取り方によっては新しい発見もある

755 :ご冗談でしょう?名無しさん:2024/03/13(水) 20:47:10.31 ID:???.net
>>753
岩とか水も太陽光に照らされてんでしょう
生物は太陽光の色が見えるように進化したんだからそりゃ見えるわ

756 :ご冗談でしょう?名無しさん:2024/03/13(水) 21:04:54.98 ID:???.net
>>755
夜にライターの火で岩を見ても同じことやね
ライターの火の光も可視光
電灯も可視光
なんでこんなに人間に都合が良いの?

757 :ご冗談でしょう?名無しさん:2024/03/13(水) 21:31:11.33 ID:???.net
>>756
ライターの火も可視光成分出てるけどだいぶ見にくいよね?
街灯も月明りもない夜だと暗視カメラがなきゃ何も見えないよね?
赤外線なんてそこらじゅうに飛び交ってるはずなのに
こんな不便なのにどこが都合が良いんでしょう?

758 :ご冗談でしょう?名無しさん:2024/03/13(水) 21:35:30.89 ID:???.net
電灯は人間に見やすいように作ってるから見えるのはあたりまえだわな

759 :ご冗談でしょう?名無しさん:2024/03/13(水) 21:38:02.75 ID:???.net
コタツなんかも見えないと売れないんで、わざと色付けてる

760 :ご冗談でしょう?名無しさん:2024/03/13(水) 22:07:43.28 ID:???.net
去年だったかバーベキューでアルコール燃料投入して焼死した事件あったけど、周りの人に炎見えないから火だるまになってるのわからなかったらしいな

761 :ご冗談でしょう?名無しさん:2024/03/14(木) 00:09:09.29 ID:DglTN68s.net
昆虫には紫外線が見えるものがいるらしい。紫外線で見ると、植物が人間の見た姿とは違ってるんだったか。昆虫の視覚に適応してるらしい。

視覚に限らず、光合成も可視光域を利用するから、光化学的な理由なのかな。でも面白いのは、植物が隣接植物を認識するのは近赤外線とか。うろ覚えだけど。

低エネルギーの電磁波だと、化学的な反応を誘導しにくいのかもね。

762 :ご冗談でしょう?名無しさん:2024/03/14(木) 11:43:00.84 ID:???.net
>>752
天体位置表と書いてある

763 :ご冗談でしょう?名無しさん:2024/03/14(木) 11:48:06.75 ID:???.net
ライターも安全のため見える火にしたんだろうなー

764 :ご冗談でしょう?名無しさん:2024/03/14(木) 11:53:08.43 ID:???.net
>>761
赤外線を使う光合成もあるそうだ

765 :ご冗談でしょう?名無しさん:2024/03/14(木) 12:29:15.20 ID:???.net
>>762
天体位置表なら別の座標系でも書けるでしょう
等方座標系を使う意味は?

766 :ご冗談でしょう?名無しさん:2024/03/14(木) 12:50:43.64 ID:???.net
太陽の黒体放射エネルギー最大の電磁波波長は可視光領域に近い
さらに(快晴の)地球大気は可視光中心領域の電磁波を透過する

地球生物が進化する過程で視覚や物質合成に適応しないはずがない
物理環境が生物を進化させ滅亡させる、全知全能の神は必要ない。

767 :ご冗談でしょう?名無しさん:2024/03/14(木) 13:07:23.14 ID:???.net
今でも大部分の地球人は全知全能の神を信じており違う教義で闘争を続ける
宗教は物理数学の学習も実験も必要ないその教義を信じればいいだけだから

768 :ご冗談でしょう?名無しさん:2024/03/14(木) 13:38:04.68 ID:???.net
>>765
天体位置表は実用のためにある
等方座標系でなきゃ誰も使わん
使わん理由は使ってた人(漁船やアマチュア天文)に聞け
君が使うなら海上保安庁に要望してみたら?
現在は本じゃなくネットで流してるから要望に応えやすいぞ

769 :ご冗談でしょう?名無しさん:2024/03/14(木) 13:41:53.66 ID:???.net
ペンローズ図の座標は宇宙全体が一望できてヌル世界線が直線だから便利だぜ
ワームホールが他宇宙に繋がってるのも見れる

770 :ご冗談でしょう?名無しさん:2024/03/14(木) 14:07:14.92 ID:???.net
>>768
そりゃ物理的な話じゃなくて用途限定の話だろ
物理的には座標系を用途によって使い分けりゃいいってだけの話

771 :ご冗談でしょう?名無しさん:2024/03/14(木) 16:30:48.30 ID:???.net
んだんだ

見栄え的にはシュバルツシルト計量が一番分かり易く見通しがいい。
実測距離dsが直感的に分かり易く計算できるのも良き。

772 :ご冗談でしょう?名無しさん:2024/03/14(木) 18:45:20.75 ID:???.net
シュワルツシルト計量は外側しか記述出来ない時点で不便だ

773 :ご冗談でしょう?名無しさん:2024/03/14(木) 22:08:52.05 ID:???.net
内側も普通に記述できるけど、内と外の境目が発散するってだけだけどな

774 :ご冗談でしょう?名無しさん:2024/03/15(金) 11:31:38.85 ID:???.net
>>771
ホントに距離だと思ってんの?

775 :ご冗談でしょう?名無しさん:2024/03/15(金) 11:33:51.48 ID:???.net
>>470
お前の物理的は測定観測が入らんのか

776 :ご冗談でしょう?名無しさん:2024/03/15(金) 12:08:01.89 ID:???.net
どこにレスつけてんのw

777 :ご冗談でしょう?名無しさん:2024/03/15(金) 12:16:04.36 ID:???.net
ぷっちーポポ ぷちプラーイ

778 :ご冗談でしょう?名無しさん:2024/03/15(金) 12:43:26.78 ID:???.net
ニュートン力学に比べて相対論的力学や量子力学は式が複雑な傾向にあると思います
これは人間の大きさ的にニュートン力学が最も生きる上で重要なので、本能的にニュートン力学を理解しやすいように脳や体が作られているのでしょうか

779 :ご冗談でしょう?名無しさん:2024/03/15(金) 17:08:36.66 ID:F6zqCCL/.net
>>774
正確には擬距離空間か

780 :ご冗談でしょう?名無しさん:2024/03/15(金) 17:12:46.45 ID:F6zqCCL/.net
ミンコフスキー計量の動径rは円周距離から逆算した(2πで除した)距離であり、実測距離とは異なる。
実測距離は
ds=dr/√(1-a/r)
であり円周距離より伸びている。
これがまさに空間が歪んでいる証左。

781 :ご冗談でしょう?名無しさん:2024/03/15(金) 17:46:49.83 ID:???.net
実測距離は時空を定める段階で既に決まっているのに対し、シュバルツシルト座標及び計量はその導出の時点で交叉項0かつg_{φφ}がrg_{rr}に比例するという仮定を課してるからね

782 :ご冗談でしょう?名無しさん:2024/03/15(金) 17:53:49.75 ID:???.net
>>781
r^2g_{rr}だった

783 :ご冗談でしょう?名無しさん:2024/03/15(金) 18:18:12.15 ID:???.net
円が縮むってこと?

784 :ご冗談でしょう?名無しさん:2024/03/15(金) 18:46:40.43 ID:???.net
だとすれば円が潰れるときに破綻するね

785 :ご冗談でしょう?名無しさん:2024/03/15(金) 20:34:33.46 ID:???.net
>>780
距離と実測距離ってなんか違うん?

786 :ご冗談でしょう?名無しさん:2024/03/16(土) 09:20:21.60 ID:???.net
相対論ではそもそも空間的な距離は測り方によって変わるから、
座標の目盛りrをそのまま使うか、そうじゃないならちゃんと測り方を指定してやらないとだめよ

787 :ご冗談でしょう?名無しさん:2024/03/16(土) 11:17:44.57 ID:???.net
>>772
外側しか記述出来ないのは等方座標系だ
シュバルツシルトブラックホールを等方座標で書いてみたら、ブラックホールの内側は座標上のどこにも対応してないのな

788 :ご冗談でしょう?名無しさん:2024/03/16(土) 13:10:21.76 ID:???.net
>>784
現実的にあり得るから、持ってる円預金はある程度外貨や株式や金資産に変えておくとよい

789 :ご冗談でしょう?名無しさん:2024/03/16(土) 13:23:35.33 ID:???.net
でぼん しゃいあー

790 :ご冗談でしょう?名無しさん:2024/03/16(土) 13:52:30.35 ID:???.net
結構前から円をドルに変えておいたから円安で儲かった

791 :ご冗談でしょう?名無しさん:2024/03/16(土) 22:53:45.17 ID:KR/H+4V5.net
tps://i.imgur.com/VfvgoLs.jpeg

写真は講談社基礎物理学シリーズの統計力学からです。関係ないところは隠しました。ボルツマン分布で、速度空間をV_1, V_2, ...と分割し、N個の分子をN_1, N_2, ..個に分けた微視的状態の数Wの変分を取って、δlogWを考えるとき、δlogWのうちNlogNの項の偏微分がなくなる理由はなんですか。
N=N_1+N_2+... という数だから、N_iのどれでも偏微分できるように思います。

792 :ご冗談でしょう?名無しさん:2024/03/16(土) 23:00:22.10 ID:???.net
Nは定数

793 :ご冗談でしょう?名無しさん:2024/03/16(土) 23:07:38.43 ID:KR/H+4V5.net
わかりました。ありがとうございます。偏微分できるような気がしましたが、確かにN_iを変化させてもNlogNの値は変わらないですね。
実際に偏微分すると0にならないのが不思議です

794 :ご冗談でしょう?名無しさん:2024/03/16(土) 23:15:47.84 ID:???.net
x+y=1
∂x/∂x +∂y/∂x =∂1/∂x
1=0

795 :ご冗談でしょう?名無しさん:2024/03/16(土) 23:33:25.87 ID:KR/H+4V5.net
いくら変数が入ってても全体で変化しないなら定数だから、微分するのがおかしいってことですかね
ありがとうございます。

796 :ご冗談でしょう?名無しさん:2024/03/17(日) 11:40:19.52 ID:???.net
おかしいこたないがな
定数を微分したら0になるだけ

797 :ご冗談でしょう?名無しさん:2024/03/17(日) 12:25:34.11 ID:???.net
>>795
dy/dx=-1やで

798 :ご冗談でしょう?名無しさん:2024/03/17(日) 12:25:48.67 ID:???.net
>>797
>>794

799 :ご冗談でしょう?名無しさん:2024/03/17(日) 12:52:38.10 ID:0VgvqqGX.net
>>791
分子の数N個をV_1, V_2, ...という速度空間で分割し、それぞれの速度空間に含まれる分子の数をN_1, N_2, ..個とします。このとき、微視的状態の数Wは、以下のように表されます。

W = N! / (N_1! N_2! ...)

ここで、N_1, N_2, ...は、それぞれの速度空間に含まれる分子の数であり、N = N_1 + N_2 + ...です。この式の対数を取ると、

logW = log(N!) - log(N_1!) - log(N_2!) - ...

となります。この式の変分を取ると、

δlogW = δlog(N!) - δlog(N_1!) - δlog(N_2!) - ...

となります。ここで、N_1, N_2, ...を微小量dN_1, dN_2, ...だけ変化させたとき、N_1 + dN_1, N_2 + dN_2, ...になるため、Nも微小量dNだけ変化します。つまり、δN = dNとなります。したがって、

δlogW = δlog(N!) - δlog(N_1!) - δlog(N_2!) - ...
   = δ[Nlog(N/e)] - δ[N_1log(N_1/e)] - δ[N_2log(N_2/e)] - ...
   = δ[NlogN - N - N_1logN_1 + N_1 - N_2logN_2 + N_2 - ...]

ここで、N_1, N_2, ...を微小量dN_1, dN_2, ...だけ変化させたとき、N_1 + dN_1, N_2 + dN_2, ...になるため、以下の式が成り立ちます。

δN_1 + δN_2 + ... = 0

したがって、δN_1, δN_2, ...の中には、NlogNの項に対応するものはありません。そのため、δlogWのうちNlogNの項の偏微分がなくなるのです。

800 :ご冗談でしょう?名無しさん:2024/03/17(日) 13:08:10.66 ID:???.net
N2を固定してN1で微分するのとN=N1+N2は両立しないというだけ
N1を増やせばN2は減らすしかない

801 :ご冗談でしょう?名無しさん:2024/03/17(日) 20:02:42.03 ID:???.net
ムーニーちゃんしんぷはダンスを始めてからすごくなったんだ

802 :ご冗談でしょう?名無しさん:2024/03/17(日) 21:33:35.03 ID:???.net
哀れやな

803 :ご冗談でしょう?名無しさん:2024/03/19(火) 10:47:11.56 ID:???.net
機械学習の深層学習を基礎から調べているのですが
深層学習の学習部分は、深層学習の多層ネットワークの重みを
最急降下法で求めることで、重みを推定しています。
深層学習は膨大な数万個の重みを一度に求める線形最適化なのです。
この最適化ですが量子コンピュータの用途の一つの量子アニーリングで
高精度にできれば膨大な量のGPUは不要だと思うのです。
実現可能でしょうか?

804 :ご冗談でしょう?名無しさん:2024/03/19(火) 12:50:39.53 ID:???.net
可能でも意味ないだろうなー

805 :ご冗談でしょう?名無しさん:2024/03/19(火) 12:52:29.63 ID:???.net
でぼぼぼ でぼぼぼぼぼ でぼぼぼぼぼぼぼ でぼん

806 :ご冗談でしょう?名無しさん:2024/03/19(火) 13:12:48.23 ID:???.net
>>804
あるよ

807 :ご冗談でしょう?名無しさん:2024/03/19(火) 17:53:44.86 ID:???.net
本物のAIに達しないのに?

808 :ご冗談でしょう?名無しさん:2024/03/19(火) 19:14:04.20 ID:???.net
まあ量子コンピュータの使い道ができてよかったじゃない
「生成AIの学習を量子コンピュータならわずか1ミリ秒で完了する!
 では、そのプロセスをもう一度見てみよう」

809 :ご冗談でしょう?名無しさん:2024/03/19(火) 19:21:49.08 ID:???.net
ディープラーニングは学習データが重要であって、今は計算を高速化する意味は無いと言っても良い。

810 :ご冗談でしょう?名無しさん:2024/03/19(火) 19:57:08.99 ID:???.net
量子アニーリングじゃ損失関数の最小化はできないよ

811 :ご冗談でしょう?名無しさん:2024/03/19(火) 20:00:14.44 ID:???.net
>>809
デタラメ
計算時間を短くする手法は常に探られている

812 :ご冗談でしょう?名無しさん:2024/03/19(火) 20:24:26.11 ID:???.net
エントロピーは統計力学ではミクロな状態数から定義されて、マクロでは熱の出入りから定義されていますが
それらの全く異なる定義の量は一致するのでしょうか?

813 :ご冗談でしょう?名無しさん:2024/03/19(火) 20:29:51.01 ID:???.net
そもそもLLMで計算量に対するスケーリング測が成り立ってるから超重要

814 :ご冗談でしょう?名無しさん:2024/03/19(火) 20:32:57.24 ID:???.net
機械学習は、「ゴミを入れればゴミしか出てこない」ということが一番大切な指導原理になっている。些細な高速化にこだわるのは末端の自称研究者だけ。

815 :ご冗談でしょう?名無しさん:2024/03/19(火) 20:46:54.94 ID:???.net
物理しか知らないやつは、計算量と計算速度とアルゴリズムとコンピュータアーキテクチャの定量的評価の方法を全く知らないからトンチンカンな妄言を吐いてバカにされる。
2003年でCPUの電力・サイズ効率の限界に達したので、それ以降のMPUはコア数を増やす戦略に変わったとか、そういう基本的な事実すら知らない。

816 :ご冗談でしょう?名無しさん:2024/03/19(火) 20:54:26.27 ID:???.net
>>813
LLMで計算量に対するスケーリング測が成り立ってる
っていうのはどういうことですか?
証拠を示せますか?

817 :ご冗談でしょう?名無しさん:2024/03/19(火) 20:58:41.21 ID:???.net
そのままの意味ですけど…

818 :ご冗談でしょう?名無しさん:2024/03/19(火) 21:00:26.23 ID:???.net
>>812
特定の条件で極限とれば概ね一致する

819 :ご冗談でしょう?名無しさん:2024/03/19(火) 21:02:59.62 ID:???.net
>>818
ということは、ある特定の条件でしか一致しない、ということでしょうか?

820 :ご冗談でしょう?名無しさん:2024/03/19(火) 21:11:45.65 ID:???.net
熱力学と統計力学って別の学問なの?

821 :ご冗談でしょう?名無しさん:2024/03/19(火) 21:32:42.44 ID:???.net
日本人の物理研究者が書いた熱力学の本で、「この本で新しい熱力学の論理体系を整理した」みたいなのがいくつかあるけど
その手の本を見てもエントロピーの説明はグズグズでなんらの新しさも有用性もない。
その一方で、情報理論の構築過程で明らかになった、「負の情報量の期待値」がいろいろな場面で役に立つ、
という経験則を、物理のエントロピーの説明に接続しようとする試みは、情報理論の研究者側からしつこく発表されている。
ギブスエントロピーとか、シャノンエントロピーとか、名前はどうでもいいけど、それらは同一の量を定式化している。

822 :ご冗談でしょう?名無しさん:2024/03/19(火) 21:40:09.23 ID:???.net
>>820
もちろん別だけど力学ってなんやろな
熱力学、統計力学、量子力学とはいうけど、相対性力学、電磁気力学とはいわんし
力学ってどういう意味の言葉なんだろうか

823 :ご冗談でしょう?名無しさん:2024/03/19(火) 21:41:47.52 ID:???.net
>>822
申し訳ないけど、>>820はそういう程度の低い話ではない。

824 :ご冗談でしょう?名無しさん:2024/03/19(火) 21:45:09.93 ID:???.net
>>823
話の程度とかどうでもいいけど、>>822 の疑問に対する答えはないの?

825 :ご冗談でしょう?名無しさん:2024/03/19(火) 21:47:01.03 ID:???.net
>>824
ない。

826 :ご冗談でしょう?名無しさん:2024/03/19(火) 21:48:08.05 ID:???.net
>>825
だいぶ程度低い人なんですね

827 :ご冗談でしょう?名無しさん:2024/03/19(火) 21:51:30.58 ID:???.net
疑問ってのは最底辺層のバカでも作ろうと思えばいくらでも作れる。
それらの最底辺層のバカが作った大量の疑問にいちいち全部答えられるのは、その最底辺層の人間だけだろう。

828 :ご冗談でしょう?名無しさん:2024/03/19(火) 21:53:57.50 ID:???.net
答えんでいいけど、ただ煽るだけなら出てこないでよい

829 :ご冗談でしょう?名無しさん:2024/03/19(火) 21:54:09.22 ID:???.net
>>822
dynamicsやから物体の運動についての学問なんかな?って思ったけど静的な系とかも考えるしようわからんw
運動じゃなくてエネルギーについてとか?

830 :ご冗談でしょう?名無しさん:2024/03/19(火) 21:54:49.75 ID:???.net
dynamicsとkinimaticsの違いは何ですか?みたいな最底辺層の疑問と回答は英語でも死ぬほど見つかるから自分で見つけると良いよ

831 :ご冗談でしょう?名無しさん:2024/03/19(火) 22:01:01.47 ID:???.net
力学と付く分野も付かない分野も、dynamics も kinimatics も扱うと思うんだが
となるとわかんないよね

832 :ご冗談でしょう?名無しさん:2024/03/20(水) 01:14:06.06 ID:???.net
ムニちゃーんポポ

833 :ご冗談でしょう?名無しさん:2024/03/20(水) 11:01:07.73 ID:???.net
>>822
相対論的力学, 電磁力学と言うぞ

834 :ご冗談でしょう?名無しさん:2024/03/20(水) 11:03:30.85 ID:???.net
dynamics 【名】 《物理》動力学◇物体や物体の系の、動作および平衡状態に影響を与える力を扱う学問分野

835 :ご冗談でしょう?名無しさん:2024/03/20(水) 11:15:35.89 ID:???.net
>>833
それは相対性理論や電磁気学の一分野だよね
一方、熱力学、統計力学、量子力学にははじめからデフォで力学と付くのはなんでという疑問

836 :ご冗談でしょう?名無しさん:2024/03/20(水) 11:38:44.27 ID:???.net
場をメインにするか物体をメインにするかの違いだよ
量子力学も場をメインにすると場の量子論と言う

837 :ご冗談でしょう?名無しさん:2024/03/20(水) 11:50:32.22 ID:???.net
なるほど

838 :ご冗談でしょう?名無しさん:2024/03/20(水) 13:08:07.34 ID:???.net
相対性理論は力学の一分野だろ
量子電磁力学は電磁気学を全部含んでんじゃねーの?

839 :ご冗談でしょう?名無しさん:2024/03/20(水) 13:22:20.18 ID:???.net
相対性理論は時空の理論だよ
場の古典論の範疇

840 :ご冗談でしょう?名無しさん:2024/03/20(水) 13:24:31.92 ID:???.net
正直な印象としては、「力学」に厳密な定義は無いんだろうなって感じ
みんななんとなく感覚で付けたり外したりしてそう

841 :ご冗談でしょう?名無しさん:2024/03/20(水) 14:29:28.10 ID:???.net
幾何学の幾何は、GeometryのGeoと中国読みの幾何(ジーフ)が似ていたから

●●的の的は、日本で生み出されたが、romanticのticがどうやっても訳せず、
日本語の音の「的」に置き換えた
romanも訳せず「浪漫」にした、邦訳の敗北みたいな言葉だ

842 :ご冗談でしょう?名無しさん:2024/03/20(水) 18:32:28.62 ID:???.net
>>840
dynamicならdynamicsだろ

843 :ご冗談でしょう?名無しさん:2024/03/20(水) 20:16:57.81 ID:???.net
>>841
中国語の「〇〇的」は?
あれと日本語の「〇〇的」は同じ由来かと思ってた

844 :ご冗談でしょう?名無しさん:2024/03/20(水) 20:44:55.54 ID:???.net
雷が光ったり音を出したりする原因は、電気が流れた部分の空気が激しく振動するからだと教わりました。一方で、放射線の中にはベータ線というものがあって、それは電子だと教わりました。雷は放射線なのでしょうか。

845 :ご冗談でしょう?名無しさん:2024/03/20(水) 21:06:56.66 ID:???.net
>>843
なので「的」は3つ意味がある
1 まと
2 の
3 ticの置き換え

2と3は中国が逆輸入した

846 :ご冗談でしょう?名無しさん:2024/03/20(水) 22:00:03.94 ID:???.net
>>845
はえー

847 :ご冗談でしょう?名無しさん:2024/03/20(水) 22:48:00.44 ID:???.net
重力は、英語のgravityではなく
オランダ語のzwaartekrachtつまり、zwaarte=重い、kracht=力の直訳
これも中国は逆輸入した
量子も日本で作られたと思う

848 :ご冗談でしょう?名無しさん:2024/03/20(水) 23:56:55.34 ID:???.net
>>844
電子とういう意味では同じだけど、β線はベータ崩壊が原因で雷は電磁気力によるものだからメカニズムが違うね
β線は空気中では数m、体内では数mmしか飛ばないけど、雷は電磁気力により人体を貫通していくし威力も桁違いなので人体への被害は甚大

849 :ご冗談でしょう?名無しさん:2024/03/21(木) 01:08:34.02 ID:???.net
>>844 なんて、いかにもエセ質問じゃん

850 :ご冗談でしょう?名無しさん:2024/03/21(木) 01:09:59.36 ID:???.net
>>849
エセ質問ってなに
どうみても質問だ

851 :ご冗談でしょう?名無しさん:2024/03/21(木) 01:41:57.07 ID:???.net
自分が分からない質問はエセ質問

852 :ご冗談でしょう?名無しさん:2024/03/21(木) 07:34:39.76 ID:???.net
>>848
ありがとうございます!
メカニズムが違うと名前が変わるんですね!
後で調べてたら他にも陰極線というものもみつけました

853 :ご冗談でしょう?名無しさん:2024/03/21(木) 11:49:39.89 ID:???.net
ムーニーちゃんしんぷのおじさんをバカにするな

854 :ご冗談でしょう?名無しさん:2024/03/21(木) 12:48:36.28 ID:???.net
大航海時代の船
約1850mごとに結び目をつけたロープのついている丸太を投げ込み、
一定時間進んで、流したロープを回収して結び目の数を数えて
結び目の数×1850m ÷ 時間 = 平均速度
を求めて、それにl航海時間と方位から、出発地からの位置を求めたそうだけど
GPS無かったのか

その丸太がログの語源で、結び目がノットの語源らしいけど

でもアートディンクの「アトラス」では船は陸伝いに進むのが原則で
陸もない所を進むと、幻の大陸みたいのに行ってしまってやべーことになった記憶

昼間なら太陽の南中時間で正午がわかるけど黄道十二宮の星座が見えない
夜中は太陽が無いので時間がわからない

855 :ご冗談でしょう?名無しさん:2024/03/21(木) 18:44:12.22 ID:???.net
「1850mごと」てのは何で分かるんだ?

856 :ご冗談でしょう?名無しさん:2024/03/21(木) 20:53:26.24 ID:???.net
国語って大事だな

857 :ご冗談でしょう?名無しさん:2024/03/21(木) 20:58:05.63 ID:???.net
1850mは緯度1分の1/60の長さ
40000m÷360=111.111
111.111÷60=1852m

ロープに1850mごとに結び目(ノット)をつけておく
その長さを測る原器は何かがあったんだろう

858 :ご冗談でしょう?名無しさん:2024/03/21(木) 21:48:12.55 ID:???.net
大航海時代にメートルなんて単位はまだなかったろ

859 :ご冗談でしょう?名無しさん:2024/03/22(金) 11:32:56.29 ID:???.net
海流は無視か

860 :ご冗談でしょう?名無しさん:2024/03/22(金) 12:00:22.93 ID:???.net
北極星で緯度はわかるが経度がわからない
月とそのほかの星座の方位と仰角の関係から、緯度とローカルタイム=経度がわかるようになるには
航海年鑑が整備されてから。18世紀

861 :ご冗談でしょう?名無しさん:2024/03/22(金) 12:34:13.75 ID:???.net
16世紀頃だろ
その頃航海術の必要性に迫られて天体の位置を求めるためにコペルニクスらが地動説を確立させていったという側面もある

862 :ご冗談でしょう?名無しさん:2024/03/22(金) 13:22:51.82 ID:???.net
それは後から作られた偽りの歴史

863 :ご冗談でしょう?名無しさん:2024/03/22(金) 13:49:53.18 ID:???.net
宇宙戦艦ヤマトはどうやって自艦の位置を計算したか

ボイジャーでさえどうなってるのかわからんが、恒星間どころか
銀河間航海で位置を決めるのは難しいね

864 :ご冗談でしょう?名無しさん:2024/03/22(金) 22:16:47.93 ID:???.net
銀河は個性的だから簡単に識別できる

865 :ご冗談でしょう?名無しさん:2024/03/23(土) 14:13:32.46 ID:???.net
宇宙には暗黒物質は存在せず年齢は267億歳だったようだ
https://nazology.net/archives/147436

866 :ご冗談でしょう?名無しさん:2024/03/23(土) 16:25:35.70 ID:???.net
しんぷさんもムーニーちゃん プー

867 :ご冗談でしょう?名無しさん:2024/03/23(土) 17:33:14.24 ID:???.net
哀れやな

868 :ご冗談でしょう?名無しさん:2024/03/24(日) 00:46:22.51 ID:???.net
>>836
4光年先のアルファケンタウリに亜光速宇宙船で行く場合、
出発地の太陽と、アルファ星は観測できるから、どこに行けばいいのかという問題は無いとして
どこを航行しているのかは、近傍の他の恒星を観測する必要がある

869 :ご冗談でしょう?名無しさん:2024/03/24(日) 01:47:01.76 ID:???.net
アンカ間違えるなよ

870 :ご冗談でしょう?名無しさん:2024/03/24(日) 14:52:38.48 ID:???.net
でぼん でぼーん

871 :ご冗談でしょう?名無しさん:2024/03/24(日) 20:19:28.39 ID:???.net
サスキンドが、ノイマンの本の主張に間違いを見つけて新しい理論を作った話って何だっけ?

872 :ご冗談でしょう?名無しさん:2024/03/24(日) 21:52:28.27 ID:WY4UTmAc.net
コリオリ力は上昇気流や下降気流を妨害する・・・とすると、
北半球の竜巻は北方向に傾いて空に伸びていて、
逆に南半球の竜巻は南方向に傾いて空に伸びているんですか?

873 :ご冗談でしょう?名無しさん:2024/03/24(日) 22:31:48.26 ID:7ZjWISQ5.net
ゴミを燃やしたりすると煙が巻き上がる。あれも自転しないと巻かないのかな。

台風は渦巻きに自転の明らかな影響あるけど、竜巻や渦潮とか諸々の渦にはあるとは耳にしないね。

874 :ご冗談でしょう?名無しさん:2024/03/24(日) 22:58:11.76 ID:???.net
計算すればわかるけどコリオリの力は小さいので、台風や海流などの大規模構造でしかその効果は現れない。
風呂の渦巻きが赤道をまたぐと反対になる、とかいうデモは全部ウソ。

875 :ご冗談でしょう?名無しさん:2024/03/24(日) 23:00:40.04 ID:???.net
一応補足:自転によるコリオリの力と書くべきだった

876 :ご冗談でしょう?名無しさん:2024/03/24(日) 23:12:25.47 ID:WY4UTmAc.net
コリオリの力で吸い込む時に発生する渦は自転と同じ方向だから遠心力を強化し吸い込みを妨害する。
逆にコリオリの力で押し出す時に発生する渦は自転と逆方向だから遠心力を打ち消し押し出しを妨害する。
だから渦の軸は、自転軸と平行になろうとして竜巻は北や南に倒れ込むような姿勢になるのでは?

877 :ご冗談でしょう?名無しさん:2024/03/24(日) 23:24:47.14 ID:7ZjWISQ5.net
見かけの力を本来の力と区別するために偽力とか呼ぶ方がわかりやすいよね。コリオリの力ではなく、コリオリの偽力みたいに。慣性力とか、名称に疑問。慣性の力って何なの?ってなるし。

878 :ご冗談でしょう?名無しさん:2024/03/24(日) 23:38:09.97 ID:???.net
一般相対論的には慣性力は偽ではない
非一般相対論的にも慣性力が実在か否か等は判別できないし

879 :ご冗談でしょう?名無しさん:2024/03/25(月) 00:13:14.13 ID:???.net
コリオリの偽力のほうが100倍何なの?ふざけてんの?ってなるわ

880 :ご冗談でしょう?名無しさん:2024/03/25(月) 00:13:42.93 ID:???.net
>>874
実際に実験すると反対になるということはYouTubeでも上がってる

881 :ご冗談でしょう?名無しさん:2024/03/25(月) 00:15:10.85 ID:???.net
>>880
だからあれはウソ
あらかじめかき混ぜてから表面が落ち着くまで待ってから線を抜いてる

882 :ご冗談でしょう?名無しさん:2024/03/25(月) 00:23:52.51 ID:8N1ARXW6.net
コリオリの力って言っても自転による見かけの力なだけだし、その点の理解のために相対論とか必要ないし。力の分類から考えるなら、偽の力なんだし。相対論でそのカテゴリーの移動をするにしても、偽の力のカテゴリ自体は必要でしょ。

883 :ご冗談でしょう?名無しさん:2024/03/25(月) 00:32:14.84 ID:???.net
>>882
だから偽の力ってなんやねん
勝手に偽にすんな

884 :ご冗談でしょう?名無しさん:2024/03/25(月) 00:33:04.87 ID:???.net
漂う荒し臭

885 :ご冗談でしょう?名無しさん:2024/03/25(月) 00:39:41.85 ID:8N1ARXW6.net
>>876
洗濯機の渦が洗濯機を傾けたらどうなるか観察してみたらどうか。実際は動きながら傾けないと、実態に近づかないように思うけど。複雑な現象は実験観察しかないと思うし。

886 :ご冗談でしょう?名無しさん:2024/03/25(月) 00:41:30.98 ID:???.net
>>882
そもそも何を持って偽とするんだ?

887 :ご冗談でしょう?名無しさん:2024/03/25(月) 00:51:10.13 ID:???.net
複数人でまったく同じことをかわるがわる聞かんでよい

888 :ご冗談でしょう?名無しさん:2024/03/25(月) 01:06:03.22 ID:8N1ARXW6.net
>>886
相対論っぽくなって難しいね。見かけの力の言い換え程度だね。

889 :ご冗談でしょう?名無しさん:2024/03/25(月) 01:21:07.90 ID:8N1ARXW6.net
宇宙のあらゆる銀河や銀河系とかが測量できるなら、本当の力かどうかわからないことないはず。

本当の力を識別する方法がないとは限らないと思う。

890 :ご冗談でしょう?名無しさん:2024/03/25(月) 02:03:43.16 ID:???.net
無視推奨

891 :ご冗談でしょう?名無しさん:2024/03/25(月) 05:18:30.82 ID:???.net
なんでやねんおかしいやろというツッコミのつもりでも、なんか聴衆から質問が来たわみたいに捉えられるから、
どうしても馬鹿と絡みたい場合はとにかく直接的な表現を心がけましょう

892 :ご冗談でしょう?名無しさん:2024/03/25(月) 09:52:20.20 ID:???.net
>>885
洗濯機本体の排水口はドラムの向こう側で観察しづらいと思うぞ

893 :ご冗談でしょう?名無しさん:2024/03/25(月) 10:04:54.37 ID:???.net
というか相対時間クソもあるかよ。
宇宙なんて時間滅茶苦茶だ。
相対して損したわ

894 :ご冗談でしょう?名無しさん:2024/03/25(月) 12:46:51.68 ID:???.net
>>878
大域的には区別可能
つまり重力と慣性力は区別可能

895 :ご冗談でしょう?名無しさん:2024/03/25(月) 13:09:14.76 ID:???.net
>>894
大域的に重力と慣性力が区別可能なのはそのとおりだけど局所的には重力と慣性力は区別不能
完全な別物ではない以上偽と弾ずるには早計かと

896 :ご冗談でしょう?名無しさん:2024/03/25(月) 13:45:49.64 ID:???.net
断ずる

897 :ご冗談でしょう?名無しさん:2024/03/25(月) 14:21:44.28 ID:???.net
弾ずると跳ね返りそうだな

898 :ご冗談でしょう?名無しさん:2024/03/25(月) 14:52:44.42 ID:???.net
多時間論

899 :ご冗談でしょう?名無しさん:2024/03/25(月) 16:44:06.29 ID:LI8eCUnp.net
>>895
馬鹿なことを言うな。重力と慣性力は完全に別物だ。重力は質量によって引き起こされる引力であり、慣性力は物体が静止または一定速度で直線運動を続ける性質だ。局所的にも大域的にも、この違いははっきりしている。それを理解できないのはお前の頭が悪いからだろう。

900 :ご冗談でしょう?名無しさん:2024/03/25(月) 16:45:02.81 ID:???.net
>>888
というか「力」はすべてゲージ作用

901 :ご冗談でしょう?名無しさん:2024/03/25(月) 16:47:09.23 ID:???.net
>>889
局所座標系が補正を受けること自体が「力」の定義
相対性原理もゲージ原理の系。

902 :ご冗談でしょう?名無しさん:2024/03/25(月) 20:40:08.02 ID:???.net
量子はエネルギー場を持ってる。
スカラー群の作用で力場を生じる

電荷そもそもが力場を持つ

903 :ご冗談でしょう?名無しさん:2024/03/25(月) 20:41:01.28 ID:???.net
>>899
局所的には区別できんやろ
区別できない例があるんやからなおさら慣性力を偽物扱いするのは早計や
無くても早計やしな

904 :ご冗談でしょう?名無しさん:2024/03/25(月) 21:00:36.94 ID:???.net
>>902
マクスウェルの応力テンソルがその例だな

905 :ご冗談でしょう?名無しさん:2024/03/25(月) 21:12:27.84 ID:???.net
重力はゲージ原理に従ってるが
まだゲージ場として確立してないんだよな
スピンも重力源にしようと幾つか理論は作られてるが結論は遠い

906 :ご冗談でしょう?名無しさん:2024/03/25(月) 22:53:55.11 ID:???.net
>>904
マクスウエルの応力といえばビーフェルトブラウン効果かな?
アメリカが風洞実験して、その結果は機密扱いになっている。

907 :ご冗談でしょう?名無しさん:2024/03/25(月) 23:24:29.19 ID:???.net
エントロピック重力

908 :ご冗談でしょう?名無しさん:2024/03/26(火) 00:01:25.37 ID:???.net
原子核に中性子を当てると聞きますがどうやって当てるんですか?

909 :ご冗談でしょう?名無しさん:2024/03/26(火) 00:26:36.46 ID:???.net
>>908
頑張る

910 :ご冗談でしょう?名無しさん:2024/03/26(火) 00:31:32.20 ID:???.net
>>908
数撃ちゃ当たる

911 :ご冗談でしょう?名無しさん:2024/03/26(火) 00:35:25.68 ID:???.net
どういう機械を使って当てるんですか?

912 :ご冗談でしょう?名無しさん:2024/03/26(火) 01:18:10.37 ID:???.net
>>911
そもそも原子核物理は中性子を原子に当てることで始まった
ラジウムから出るアルファー線をベリリウム箔に当てると中性子が出てくるのでそれを原子に照射した
アルファー線ではエネルギーを変えられないので、サイクロトロン加速器で陽子(水素イオン)を加速して
ベリリウムに照射していろんなエネルギーの中性子を作り、いろんな原子に照射した
中性子をウランに照射すると核分裂が観測された

913 :ご冗談でしょう?名無しさん:2024/03/26(火) 02:09:40.29 ID:???.net
>そもそも原子核物理は中性子を原子に当てることで始まった
中性子の発見まで原子核物理はなかったとでも?

914 :ご冗談でしょう?名無しさん:2024/03/26(火) 06:01:40.48 ID:???.net
でぼぼぼ でぼぼぼぼぼ でぼぼぼぼぼぼぼ

915 :ご冗談でしょう?名無しさん:2024/03/26(火) 10:19:54.06 ID:???.net
>>912
アルファ線源としてポロニウム210が発見されて
それで強い中性子を作れるようになったと教科書に書いてあったが
なんでかは知らない

916 :ご冗談でしょう?名無しさん:2024/03/26(火) 13:11:28.97 ID:???.net
ポロニウムのアルファ線が大エネルギーだからでしょ

917 :ご冗談でしょう?名無しさん:2024/03/26(火) 14:29:30.02 ID:???.net
物凄い高速でぶつける必要があると言うけどなんでその為に高温にする必要があるの?
例えば核融合なんかは核爆発レベルの熱が必要だというけどスピードとなんの関係があるの?

918 :ご冗談でしょう?名無しさん:2024/03/26(火) 14:32:11.19 ID:???.net
陽子同士のクーロン斥力に打ち勝つ速度が必要

919 :ご冗談でしょう?名無しさん:2024/03/26(火) 15:19:50.18 ID:???.net
>>917
例えで言えば、強力なネオジム磁石のN極同士で引っ付けようとしても手の力じゃ無理だろ

核融合を起こす距離(10^-15m)の陽子同士の斥力をクーロン力の式から計算して見ればよい

920 :ご冗談でしょう?名無しさん:2024/03/26(火) 15:24:39.18 ID:???.net
ぷっちーポポ
ムニちゃーんポポ

921 :ご冗談でしょう?名無しさん:2024/03/26(火) 15:47:32.62 ID:???.net
>>917
温度は粒子運動のスピードを決めるパラメーター
高温になると原子のポテンシャル障壁を超えて融合できる高速の粒子が増える

922 :ご冗談でしょう?名無しさん:2024/03/26(火) 16:19:32.15 ID:???.net
>>917
統計力学によると運動エネルギーmv^2/2がkT/2に対応しているらしい
つまりv=√(KT/m)で温度の√に比例することになる

923 :ご冗談でしょう?名無しさん:2024/03/26(火) 16:49:13.42 ID:???.net
>>922
その式は粒子速度の平均値

核融合に1億度が必要ならば、実際にポテンシャル障壁を超えて融合できる粒子は
速度分布の平均値よりかなり高速の少数の粒子になる。

924 :ご冗談でしょう?名無しさん:2024/03/26(火) 17:51:47.93 ID:???.net
トンネル効果もあるし

925 :ご冗談でしょう?名無しさん:2024/03/26(火) 21:01:59.84 ID:XGtA2QAC.net
1億度の陽子線の波長は陽子の直径よりずっと大きいのに衝突するの?

926 :ご冗談でしょう?名無しさん:2024/03/26(火) 22:00:55.40 ID:XGtA2QAC.net
粘度
https://ja.wikipedia.org/wiki/%E7%B2%98%E5%BA%A6
>気体の粘度は温度が上昇すると上昇する。

温度が上昇すると粘性が上昇するって事は、
気体が粘性を生み出す仕組みは流体の横ズレ運動量を気体粒子が運ぶ事が原因か?
って事は、亜光速で色々な物をすり抜けられるニュートリノガスは非常に粘性が高いという事か?
ニュートリノは亜光速で流体内部をすり抜け遠くまで横ズレ運動量を輸送できる。

927 :ご冗談でしょう?名無しさん:2024/03/26(火) 22:04:01.38 ID:???.net
陽子線の波長が大きいと反応しないと思う根拠は何?

928 :ご冗談でしょう?名無しさん:2024/03/26(火) 22:13:50.83 ID:???.net
周りの粒子に運動量が移らないってことは粘性が無いってことだね

929 :ご冗談でしょう?名無しさん:2024/03/26(火) 22:37:07.04 ID:XGtA2QAC.net
>>928
中性子星の中ではニュートリノもまっすぐ進めないらしい。
これはニュートリノが色々な物質と運動量をやり取りしてるって事だろ?
ただ、中性子星は密度が高すぎて相対的に粘性の力がちっぽけになってしまうかもな・・・


>>927
↓の人がそれっぽい事言ってた。

■ちょっとした物理の質問277■蛸壺予選勝抜戦1
https://rio2016.5ch.net/test/read.cgi/sci/1680509525/362
2024/03/03(日) 00:58:06.83ID:???
1eV = 1.6×10^(-19) J
E = 250MeV = 4×10^(-11) J
E = hν, ν = E/h = 6*10^22/ s, λ ≒ c/ν = 5*10^(-13) m
原子核サイズ = 10^(-14) m < λ
波長より小さいから衝突しない

930 :P○ΘM:2024/03/26(火) 22:50:32.27 ID:K0AyddwN.net
お!

931 :ご冗談でしょう?名無しさん:2024/03/26(火) 22:50:42.92 ID:???.net
原子のサイズ(0.1 nmオーダー)は可視光の波長(400-800 nmくらい)より圧倒的に小さい。
波長より小さいと衝突しないのなら、可視光は原子と衝突しないことになるがそんなことはない

932 :ご冗談でしょう?名無しさん:2024/03/26(火) 23:05:33.37 ID:XGtA2QAC.net
原子に可視光線を当てられるのは共鳴じゃないの?
マイクロ波が金網を通れないのに水分子を加熱させたれるような・・・

933 :ご冗談でしょう?名無しさん:2024/03/26(火) 23:14:40.45 ID:???.net
最近ここんとこ毎日来てるヤツだろ?
なんというかいつも発言が当てずっぽうすぎ

934 :ご冗談でしょう?名無しさん:2024/03/27(水) 11:05:25.44 ID:???.net
>>919
剛体球モデルとポテンシャルモデルがあると思うけど
核融合だとけっこう違うものかな、臨界温度とか

935 :ご冗談でしょう?名無しさん:2024/03/27(水) 11:06:53.58 ID:???.net
>>931
それは原子が1個の場合
そもそも可視光は原子と衝突しないがな
原子の周りの電磁場と相互作用するだけ

936 :ご冗談でしょう?名無しさん:2024/03/27(水) 11:36:55.72 ID:???.net
それを衝突という

937 :ご冗談でしょう?名無しさん:2024/03/27(水) 14:28:12.89 ID:???.net
光の波長が原子より遥かに長いから
光によるほぼ一定の電磁場の中で原子が動く
古典的な現象だぜ
光子の衝突なんて量子現象を考えるのは牛刀

938 :ご冗談でしょう?名無しさん:2024/03/27(水) 14:33:54.91 ID:???.net
>>919
どっちかと言うとクーロンポテンシャルのエネルギーを計算して温度に換算すれば良いのでは

939 :P○ΘM:2024/03/27(水) 14:44:16.38 ID:U0QU4w/o.net
IDテスト兼
ksk加速

940 :ご冗談でしょう?名無しさん:2024/03/27(水) 14:50:51.21 ID:???.net
でぼぼぼぼぼぼ しゃいあー パー

941 :ご冗談でしょう?名無しさん:2024/03/27(水) 17:59:44.05 ID:???.net
>>937
散乱もするし吸収もする。古典論だけで片付くと思うなよ

942 :ご冗談でしょう?名無しさん:2024/03/27(水) 21:20:38.46 ID:???.net
口だけじゃなく具体的に

943 :P○ΘM:2024/03/28(木) 00:50:45.30 ID:JhIWkBOt.net
今日もテスト

944 :P○ΘM:2024/03/28(木) 00:52:32.54 ID:JhIWkBOt.net
あれ?読み込みができない

945 :P○ΘM:2024/03/28(木) 00:53:57.02 ID:JhIWkBOt.net
書いた後の画面切り替わりができない
戻って切り替わりできないのID変わるかなと2回目

946 :poem:2024/03/28(木) 00:55:38.93 ID:JhIWkBOt.net
書いた後やっと読み込んだ
ID変わってないじゃん
あと暗号絶対解けんわ前々回は解けて前回は解けなかったけどまだ見込みあったけど

947 :poem:2024/03/28(木) 01:01:50.98 ID:JhIWkBOt.net
あれ?なんとか少し解けた。半分未満、少し

948 :poem:2024/03/28(木) 01:02:36.02 ID:JhIWkBOt.net
この感じ…不正解?
じゃあ少しも解けてないな

949 :ご冗談でしょう?名無しさん:2024/03/28(木) 14:26:41.79 ID:???.net
poemが出てくると皆は去る

950 :ご冗談でしょう?名無しさん:2024/03/28(木) 15:17:04.01 ID:???.net
ムニちゃーんポポ

951 :ご冗談でしょう?名無しさん:2024/03/28(木) 16:03:44.46 ID:???.net
ペロ、これは反物質

952 :P○ΘM:2024/03/28(木) 16:33:27.11 ID:JhIWkBOt.net
>>951 加速器に頭突っ込んで陽子一粒頭を貫通した作業員が昔外国にいたらしいけど
陽子一粒の道筋に焼けた?みたいだけど、反粒子できなったということになるから、なんで反粒子できなかったの?
反粒子できたら、脳一定範囲焼けた可能性ありえる?

953 :ご冗談でしょう?名無しさん:2024/03/28(木) 20:39:08.78 ID:???.net
>>950
次スレ立ててね

954 :ご冗談でしょう?名無しさん:2024/03/28(木) 20:54:45.23 ID:???.net
クソワロタw
そいつもキリ番適用されるんかw

955 :P○ΘM:2024/03/28(木) 21:04:11.06 ID:JhIWkBOt.net
自分が建てよう
→偽物スレになる
自分建てるのよしとこう
→本物スレよろし

956 :ご冗談でしょう?名無しさん:2024/03/28(木) 21:25:49.92 ID:???.net
脳異常者お断り、巣スレから出るな

957 :P○ΘM:2024/03/28(木) 22:04:20.77 ID:JhIWkBOt.net
建てる人は偽物スレの番号とURL間違えないようにね

958 :poem:2024/03/28(木) 22:06:18.58 ID:JhIWkBOt.net
>>956
自分頭悪いから
ハト時計みたいなもの
時報レベルで頻繁に巣を出ちゃう

959 :poem:2024/03/28(木) 22:08:39.34 ID:JhIWkBOt.net
ハトも人間の表現単語駆使したパントマイム的喋り理解できるから
頭は実は易しい日本語理解できるレベルにはいいんだけどね

960 :poem:2024/03/28(木) 22:12:07.05 ID:JhIWkBOt.net
動物は割と
パントマイム的易しい喋りで
経験積ませれば
薄ら以上ちゃんと理解してくれる
まあ動物により頭の善し悪しはあるけど
頭良さそうな動物はいつもパントマイム的単語と手や絵使って理解できるようになる

961 :poem:2024/03/28(木) 22:13:43.80 ID:JhIWkBOt.net
パントマイム的表現駆使単語
これが重要だからね

962 :ご冗談でしょう?名無しさん:2024/03/28(木) 22:17:14.85 ID:???.net
スレ荒らしより動物のほうがマトモ

963 :P○ΘM:2024/03/28(木) 22:18:40.99 ID:JhIWkBOt.net
>>962 それは言える
poemさん←見本

964 :ご冗談でしょう?名無しさん:2024/03/28(木) 23:55:40.68 ID:niUfRd/I.net
最低20万余裕じゃね
完全に野菜に目覚めた
悔しい。

965 :ご冗談でしょう?名無しさん:2024/03/28(木) 23:57:11.77 ID:tCaIlaEV.net
乗らないのはその辺もある

966 :ご冗談でしょう?名無しさん:2024/03/28(木) 23:57:11.70 ID:xDI2sK/d.net
そうそう助かりそうになってらか味わえ
警察予備隊に過ぎん

967 :ご冗談でしょう?名無しさん:2024/03/28(木) 23:57:58.67 ID:???.net
あの動画見てたから6月まではないとは
詐欺師のやり取りの中からインチキジャンプって言われてるようなプリペイドカードしか登録できないのかなw頑張れ
まだかなまだかな~

968 :ご冗談でしょう?名無しさん:2024/03/29(金) 00:02:23.07 ID:2aOpri1o.net
なんでわざわざ嫌われるようなことするんだろ
そういった層は増えたソースある?

969 :ご冗談でしょう?名無しさん:2024/03/29(金) 00:05:48.86 ID:Ar2j9M6W.net
業界?

970 :poem:2024/03/29(金) 00:09:00.08 ID:cqU8OVlx.net
専スレ破壊荒らし来た

ヤバイ

971 :poem:2024/03/29(金) 00:09:43.68 ID:cqU8OVlx.net
これから、壊される

972 :ご冗談でしょう?名無しさん:2024/03/29(金) 00:11:15.55 ID:???.net
次スレあるのが悪いのは全然含み減って低血糖っぽい症状はあるな

973 :ご冗談でしょう?名無しさん:2024/03/29(金) 00:11:28.87 ID:???.net
観るのが1番でかい

974 :ご冗談でしょう?名無しさん:2024/03/29(金) 00:12:22.92 ID:wjDhJ5TU.net
平気で使ってた
雑談で一番被害大きいは笑えるわ

975 :ご冗談でしょう?名無しさん:2024/03/29(金) 00:15:43.87 ID:???.net
被害届出したやついなかったのか

976 :ご冗談でしょう?名無しさん:2024/03/29(金) 00:16:03.01 ID:???.net
>>782

コースギリギリ狙って相手の見逃し配信で尻を見せろ

(*^○^*)すまん、ありがとうございます🙇‍♂

977 :poem:2024/03/29(金) 00:19:21.88 ID:cqU8OVlx.net
専スレ破壊荒らしの仕組みわかった
AIや人工無能でなく

色んな板の書き込みを、コピペしてるだけ

978 :ご冗談でしょう?名無しさん:2024/03/29(金) 00:19:26.47 ID:rmhdACaL.net
このスレに次スレのリンク付きで立ててもらいたいな。

979 :ご冗談でしょう?名無しさん:2024/03/29(金) 00:20:05.51 ID:6saRwYvd.net
ペンに説教ヨントンさせる前に退陣してるし良かったね

980 :ご冗談でしょう?名無しさん:2024/03/29(金) 00:20:26.29 ID:6saRwYvd.net
抜けたからまたやって更新止めるんだろう

981 :ご冗談でしょう?名無しさん:2024/03/29(金) 00:21:13.49 ID:+z3ef40B.net
配当レースに突入したか?

982 :ご冗談でしょう?名無しさん:2024/03/29(金) 00:25:03.61 ID:???.net
何年目かでいうなら今年3年
8時間プラスして欲しい

983 :ご冗談でしょう?名無しさん:2024/03/29(金) 00:25:22.04 ID:XK5iYkv2.net
タバコと私物同じの着て滑ったらアンチのクズ
https://i.imgur.com/FMJ9W3j.jpg

984 :ご冗談でしょう?名無しさん:2024/03/29(金) 00:25:28.54 ID:???.net
推しだったらクラブで女ナンパするグループと共演って嫌だよ
卒業出来なかったらそのファンたちが食いついてくるんだろな

985 :ご冗談でしょう?名無しさん:2024/03/29(金) 00:26:25.37 ID:???.net
てか、今回は140円いくかもね
オリエンタルバイオが出した。
調べるとそうな番組内容に意味あるから
コロナに備えて

986 :poem:2024/03/29(金) 00:27:35.36 ID:cqU8OVlx.net
物理板に
専スレ破壊荒らし来たと、スレ建てしようとしたけど
無理だった

建てられない理由

https://mi.5ch.net/test/read.cgi/news4vip/1710911556

専スレ破壊荒らし

公安の上層案件だったりする?

987 :ご冗談でしょう?名無しさん:2024/03/29(金) 00:27:49.84 ID:rmhdACaL.net

https://itest.5ch.net/rio2016/test/read.cgi/sci/1711639492

988 :ご冗談でしょう?名無しさん:2024/03/29(金) 00:29:04.29 ID:hcsU+Kop.net
>>447

※アンチスレ内でおっさんは興味持たれてないのに贔屓叩かれるのが目的なんだから

ヲタについて語るとかバカじゃないです」

巣窟 すくつ

989 :poem:2024/03/29(金) 00:29:05.38 ID:cqU8OVlx.net
公安の下部は知らないだろうね(5chに普通にいるおじさん達)
公安の上層の決定だろう
公安上層の集団コピペミッション
何の理由が

990 :ご冗談でしょう?名無しさん:2024/03/29(金) 00:29:51.59 ID:lpUUhzC+.net
\📢情報解禁🦥/
それなら事故原因もすぐ分かるね
たぶん被害届や起こした裁判でまともに稼働してます
両方追い出せばいいだけやろ
https://i.imgur.com/UD6w65u.png

991 :ご冗談でしょう?名無しさん:2024/03/29(金) 00:29:51.89 ID:VjmG7eBU.net
>>891
> 投資できる上限を引きずる行為は普通ダメでしょ
これは、車だけど

992 :poem:2024/03/29(金) 00:32:42.36 ID:cqU8OVlx.net
専板破壊荒らしの理由解明するためには

公安上層が何をしたいか何を目的にしてるか何を危険視してるか
解明すること

993 :ご冗談でしょう?名無しさん:2024/03/29(金) 00:33:16.09 ID:???.net
だから潰れるんでしょ?

994 :poem:2024/03/29(金) 00:34:08.40 ID:cqU8OVlx.net
1回の素人庶民&自分知力低、には解明できないけど

995 :poem:2024/03/29(金) 00:34:22.98 ID:cqU8OVlx.net
一介

996 :poem:2024/03/29(金) 00:35:17.07 ID:cqU8OVlx.net
だから潰れるんでしょ?とは?

997 :poem:2024/03/29(金) 00:36:00.21 ID:cqU8OVlx.net
公安上層作戦
今の公安上層の名前何?

998 :poem:2024/03/29(金) 00:36:41.66 ID:cqU8OVlx.net
公安下層
おじさん達は知らない

999 :poem:2024/03/29(金) 00:37:35.50 ID:cqU8OVlx.net
次スレ延期で

1000 :ご冗談でしょう?名無しさん:2024/03/29(金) 00:39:42.10 ID:usm51y2M.net
まともだと思うから息子オタになってるし
良かったよね!
しょまたん早く大学卒業しろよと思ってたけど

1001 :2ch.net投稿限界:Over 1000 Thread
2ch.netからのレス数が1000に到達しました。

総レス数 1001
215 KB
掲示板に戻る 全部 前100 次100 最新50
read.cgi ver 2014.07.20.01.SC 2014/07/20 D ★